Journal d'un avocat

Instantanés de la justice et du droit

Aller au contenu | Aller au menu | Aller à la recherche

Le passage à l'acte, en action

J'ai déjà expliqué que dans la mécanique psychologique qui aboutit au passage à l'acte, chez un être sain d'esprit s'entend, on rencontre toujours la combinaison de deux éléments, que j'appelle la certitude de l'impunité (c'est de Beccaria, en fait) et la justification morale, qui permet à celui qui va commettre un délit de se dire “ Ce que je fais n'est pas grave, en fait, ou du moins, c'est pour une bonne cause. ”

La certitude de l'impunité n'a pas à être avérée. D'ailleurs, la plupart des gibiers de prétoire avaient la certitude de ne pas être pris.

Ça ne signifie pas que la combinaison de ces deux éléments implique passage à l'acte. La plupart d'entre nous avons des inhibitions, ou une force de volonté, pour présenter ça sous un jour plus noble, suffisante pour que nous rejetions l'idée à peine germée.

Tenez quelques exemples de justification morale :

Le voleur de sac à main : “ J'ai plus besoin d'argent qu'elle, moi c'est pour m'acheter à manger ”.

Le voleur de voiture, après avoir vérifié que la rue est vide (certitude de l'impunité) : “ De toutes façons, il est assuré. ”

Le vendeur de cannabis : “ Le cannabis, c'est pas vraiment de la drogue ; et puis c'est ça ou bosser à McDo pour gagner dix fois moins ”. Variante “ Je fais que dépanner, c'est juste pour me payer ma consommation, je suis pas un dealer ”.

Le téléchargeur pirate : “ De toutes façons, j'aurais pas acheté ce CD/DVD, et j'ai pas les moyens d'aller au ciné, alors ils perdent rien. Puis de toutes façons, ils sont déjà pété de thunes, eux. ”

Cependant, aucun d'entre eux, quelles que soient les bonnes raisons qu'il se donne, ne passerait à l'acte s'il avait la certitude d'être sanctionné (c'est là le but de la loi HADŒPI : faire cesser le sentiment d'impunité des téléchargeurs pirates).

C'est là une différence essentielle avec certaines formes de délinquance idéologique : non seulement le délinquant se fiche d'échapper à la sanction, mais il en tirera une satisfaction de martyr et fera de son procès une tribune médiatique. D'où une plus grande sévérité, non pas à cause de l'idéologie, mais du fait que la réitération est à craindre vu l'absence manifeste de regrets.

Un lecteur, qui signe Dante Timelos et fait la promotion d'un guide juridique en ligne pour vous permettre de vous défendre tout seul au pénal (et il a bien raison, quand on a droit à un avocat gratuit, autant refuser), me fournit une intéressante démonstration par l'exemple. Sous mon vademecum des comparutions immédiates chapitre 2, il donne ce fort mauvais conseil :

Concernant d'abord les garanties de représentation, dont le détail est assez bien expliqué ici, petit conseil réservé aux proches des personnes poursuivies: concrètement, les juges n’ont guère le temps et les moyens de vérifier certaines affirmations de ceux qui passent en comparution immédiate, pas plus que les documents apportés par ceux qui sont à l’extérieur... Bien penser ceci dit que tout doit être cohérent.

Ces précisions n'ont aucun intérêt si les documents sont vrais ; j'en déduis qu'il suggère à mot couvert de produire des faux si on n'a pas de vrais sous la main. Rien que ça.

Vous aurez relevé ici la certitude de l'impunité : “ les juges n’ont guère le temps et les moyens de vérifier certaines affirmations de ceux qui passent en comparution immédiate, pas plus que les documents apportés par ceux qui sont à l’extérieur ”.

Je lui ai fait remarquer que forger un faux est puni de trois ans de prison, et le produire en justice, de la même peine.

Voici sa réponse, qui contient la justification morale :

C'est pas vraiment un faux…

Cher maitre, en réponse à mon commentaire (Numéro 16) vous parlez de "faux"... comme vous y allez. Un document qu'on ne peut pas vérifier n'est pas un "faux", c'est juste un document qu'on ne peut pas vérifier, rien de plus...

… et puis c'est de bonne guerre…

La CI est une justice d'abattage, ce qui signifie que les dossiers sont légers voire vides, bâtis à la hâte avec quelques PV policiers, et que les juges n'ont pas les moyens de vérifier quoi que ce soit. Il ne s'agit alors que de retourner un peu à l'avantage des personnes poursuivies cet état de fait déplorable.

… et un peu de relativisme absolu : qu'est-ce que la vérité, après tout ?

Quant à la "vérité", personne n'y croit et surtout pas la police et le procureur qui bâtissent l'accusation. Alors...

Voilà. Un délinquant, le plus souvent, c'est ça. Ce n'est pas un être pervers et méchant, toute la volonté et l'intelligence tendues vers le but de commettre un crime (ça c'est le méchant des séries TV françaises). C'est quelqu'un qui se trouve une justification pour passer à l'acte et se convainc qu'il ne sera pas pris. Jusqu'à ce qu'on vous l'amène menotté avant une CI.

Et une petite histoire pour finir. Un prévenu, poursuivi pour vol et escroquerie (il a volé une formule de chèque la carte American Express de quelqu'un chez qui il avait été invité et l'avait utilisé pour acheter un iPod). Pensant impressionner favorablement le tribunal, il se dit étudiant en droit (très mauvaise idée : si vous êtes étudiant en droit, taisez-le) et a fait amener par son père son certificat de scolarité de la prestigieuse faculté de Framboisy-II. Son avocat présente la pièce au procureur, sans y avoir jeté un coup d'œil.

Le procureur regarde la pièce, fronce les sourcils, et demande s'il peut la garder pour le moment. L'avocat accepte bien volontiers.

Arrive le tour du brillant juriste. À peine l'examen de la personnalité a-t-il commencé que le procureur prend la parole.

— Vous êtes étudiant en droit ?

— Oui, m'sieur.

— C'est bien le certificat de scolarité que vous avez produit ?

— Oui m'sieur.

— (Sourire avenant) Monsieur, vous savez que j'ai fait la même fac que vous ? Vous voyez qu'elle peut mener à de nobles fonctions…

— Je sais m'sieur. C'est une bêtise, je sais pas ce qui m'a pris, je ne le referai jamais.

— Cependant, je m'interroge. Les certificats de scolarité, de mon temps, pas si lointain, n'étaient pas du tout comme ça…

— Ils ont changé, m'sieur.

— Vraiment ? Au point de ne plus porter le cachet de la faculté ? J'ajoute que le doyen n'avait pas l'habitude de faire de grossières fautes d'orthographe dans les certificats de scolarité. Je garde cette pièce, et vais ouvrir une enquête préliminaire pour m'assurer auprès de la faculté que vous y êtes bien inscrit.

—Le président : Vous avez quelque chose à dire, sur ce point ?

Et le prévenu de reconnaître, tout penaud, qu'il avait fait croire à son père qu'il était inscrit à la fac de droit et avait fabriqué ce faux certificat pour lui.

Je ne vous dis pas la tête de l'avocat et du père dans la salle (non, ce n'était pas moi ; ni l'un ni l'autre). Et l'impression sur le tribunal : le voleur escroc est aussi un faussaire à l'égard de son père et de la justice.

Résultat, le faux étudiant s'est pris un an de prison avec sursis (plutôt lourd pour les faits : préjudice de 60 euros environs) et une nouvelle citation à venir pour faux et usage de faux. Et son père lui a, je le crains, coupé les vivres.

Au-delà du “ c'est pas beau de faire ça ”, objectivement, c'est totalement idiot. Ça ne résout pas le problème, au contraire, ça l'aggrave. (Quoi ? Qu'est-ce que j'ai dit ?)

Commentaires

1. Le vendredi 20 mars 2009 à 19:59 par *Celeborn

J'aime beaucoup cette analyse du sentiment d'impunité, qui me semble très juste (et que chacun, je pense, a expérimenté en son for intérieur). Il faudra un jour que je revienne sur le mot "morale" : depuis que j'ai lu chez vous à de nombreuses reprises la distinction forte que vous faites entre droit et morale, ça me turlupine.

2. Le vendredi 20 mars 2009 à 19:59 par CaO2

Ouloulouloulou !! Il a eu la monnaie de sa pièce le jeune en question.

Article très divertissant merci beaucoup Maitre.

3. Le vendredi 20 mars 2009 à 20:06 par henriparisien

Jolie histoire.

Un prévenu, poursuivi pour vol et escroquerie (il a volé une formule de chèque de quelqu'un chez qui il avait été invité et l'avait utilisé pour acheter un iPod).

Il y a vraiment une escroquerie dans le fait de voler un chèque et de l'utiliser ? J'avais cru à vous lire que l'escroquerie "consiste en effet à tromper une personne par des manœuvres frauduleuses (ce qui suppose une certaine élaboration, pas un simple mensonge) pour la convaincre de remettre une chose, rendre un service, s'obliger ou au contraire libérer quelqu'un de son obligation."

Dans ce cas, la manœuvre ne parait pas très élaboré.

Eolas:
Vous me faites douter… C'était bien escroquerie. Donc vous avez raison, ça devait être une carte de crédit.

4. Le vendredi 20 mars 2009 à 20:10 par thomas

Maitre Eolas cachant une batte de baseball derrière son dos se trouve a coté d'un troll assommé, portant une longue bosse sur le sommet du cuir chevelu. Un texte “Commentaire modéré par troll détector™” surplombe l'image.

5. Le vendredi 20 mars 2009 à 20:10 par clgroup

L'anecdote me fait penser à une petite histoire racontée par mon prof d'institutions administratives pour nous dire que l'on devait, en tant qu'étudiants en droit, être plus irréprochables que les autres citoyens. Alors qu'il était, lui-même étudiant en droit, ce professeur s'est rendu à une audience correctionnelle. Alors que se présentait un témoin à charge, la personne assise à côté de mon prof a crié "Balance". S'en suit le dialogue suivant: Le président: Monsieur, vous pouvez répéter . Le jeune homme: Je retire ce que j'ai dit. Le président: Monsieur, approchez-vous, décline votre identité Le jeune homme: je m'appelle X Y, je suis étudiant en droit. Autant dire qu'il en a pris "pour son grade". Mais l'affaire n'a pas été plus loin.

6. Le vendredi 20 mars 2009 à 20:17 par jijin

en fait, pour le téléchargement, je pense que le sentiments d'impunité ne vient pas tant de là

il serait plutot de définir le téléchargement comme un usage : "je le fais en attendant que la disponibilité légale (et je ne dis pas l'offre) des contenu arrive au niveau du dl illégal" on parle de l'offre légale depuis longtemps, on parlait avant de la vente de chansons à l'unité (et pas en album), on parlait de vente en mp3 sans drms, etc... c'est en partie arrivé, beaucoup trop tard pour ne pas laisser s'instaurer un usage de la disponibilité "promise" par internet et tout ceux qui ont défendu cette modernité regardez une pub pour un smartphone : "tout ce que vous voulez à portée de mains"

nous avons les moyens, les contenants, mais pas les contenus le téléchargement ne me semble ainsi pas tant une certitude de l'impunité mais un usage en attendant ce qui n'arrive pas, ou trop tard pour que cet usage ne se soit pas fondu dans l'habitude

7. Le vendredi 20 mars 2009 à 20:27 par Etudiante

Ne jamais dire que nous étudions le droit ? Est-ce parce que, nous, pouvons dans les faits être plus aisément considérés comme au fait du droit ? Parce qu'on est plus sévère envers ses "semblables" ? Parce que nous serons amené à exercer le droit, et donc nous devons être encore plus irréprochables ?

Eolas:
Non, c'est que si nul n'est censé ignorer la loi, un étudiant en droit est censé la connaître et comprendre parfaitement la portée de son geste ; en outre, un tel étudiant a vocation à se diriger vers des professions où l'honnêteté est primordiale.

Et c'est vraiment palpable ça ? cette réaction ?

PS : Bon, et je me retiendrais encore plus d'envoyer des coups de Code dans la tête de certains idiots... C'est tentant pourtant.

8. Le vendredi 20 mars 2009 à 20:39 par Gascogne

Concernant le sentiment d'impunité, que je rattache systématiquement à Beccaria, j'avais l'habitude de dire à mes étudiants : vous qui grillez toujours les feux oranges (ne mentez pas, je suis juge d'instruction), le feriez vous si vous aviez en vue une voiture de police sur la carrefour ?
Concernant le très mauvais conseil donné en commentaire, soyez assurés, comme a pu l'indiquer Me Eolas, qu'après des années d'audience, les magistrats ont un sixième sens concernant le mensonge. C'est un très gros risque à prendre...

9. Le vendredi 20 mars 2009 à 20:42 par Avatar

Pardonnez mon ignorance, mais qu'est ce que la fac de Framboisy II? S'agit-il d'Assas ? (et dans ce cas pourquoi?)

Eolas:
Une fac imaginaire pour qu'il n'y ait pas de jaloux.

10. Le vendredi 20 mars 2009 à 20:43 par David M.

Quelle idée de se prévaloir d'une fac de province aussi...

11. Le vendredi 20 mars 2009 à 20:47 par Pihoute

Bonjour Maître,

Pardon de réagir hors de propos, mais j'ai publié un article sur le Post restituant l'intervention de Benoît XVI dans son contexte (http://www.lepost.fr/article/2009/03/20/1464352_ce-qu-a-vraiment-dit-le-pape.html), et un commentateur-contempteur de l'espèce hargneuse a cité votre post à ce sujet. Comme vos commentaires y sont fermés, je vous le dis ici : du fond du coeur, merci.

12. Le vendredi 20 mars 2009 à 20:53 par David Minne

Billet magnifique. Brillant certes, mais avec également cet art du contrepoint maîtrisé, un brin provocateur contre certaines idées – si malheureusement répandues – du temps, qui fait tout le sel de ce blog. Merci.

13. Le vendredi 20 mars 2009 à 21:13 par Vincent

Wow noon ! Benoit-seize(64) sort de ce corps ! ha ha ha...

Sinon j'ai cotoye des petits delinquants de quartier toute ma jeunesse, et je t'assure que "Cependant, aucun d'entre eux (...) ne passerait à l'acte s'il avait la certitude d'être sanctionné" n'est pas tout a fait vrai. Certains n'ont aucune impugnite et font des choses terribles pour d'autres raisons (le jeu, la gloire ?), en sachant qu'il vont se faire prendre. Tu me suis ? Comme par exemple tabasser quelqu'un pour lui "regler son compte", meme si ca doit se faire devant les policiers et que c'est "paye" avec 3 mois de prison ferme ... Rassure toi, ce n'est pas moi !

14. Le vendredi 20 mars 2009 à 21:21 par marifline

ça c'est le méchant des séries TV françaises)

Extrapolons un peu : donc vous considérez que les "séries TV" françaises sont mauvaises ( personnellement je ne vois aucune œuvre de fiction télévisuelle française qui mérite ce titre ). Je suppose que vous préférez les séries TV américaines, certaines d'entre elles ont menée des réflexions très intéressantes sur la justice et ce billet me fait plus particulièrement penser à la série Dexter qui est diffusée sur Canal + : celle dont le héros est un serial killer qui a un besoin impulsif de tuer, mais qui le fait en suivant un code établi par son père adoptif ( destiné à le garder en vie et à lui éviter la prison) et qui lui impose de ne tuer que d'autres meurtriers. Par ailleurs il ne ressent aucuns sentiments ( à part quand il tue). Hum tout ça pour dire que dans la saison 3 il se fait un ami, qui est procureur et qui approuve ce qu'il fait ( un peu trop d'ailleurs) et qui commence à vouloir tuer lui aussi, mais n'a pas la même objectivité que Dexter.

Je ne veux spoiler personne donc je m'arrête ici. Mais cela me ramène à un autre sujet évoqué dans votre billet qui est le téléchargement pirate ( puisque bien sure la saison 3 n'a pas été diffusée sur Canal +...). Effectivement je continuerai à télécharger, probablement jusqu'à ce que je reçoive un avertissement, sinon retour vers le streaming, et sinon et bin retour vers des ami(es) qui téléchargent impunément...

Je dirai qu'en plus des justifications morales et de l'impunité, le téléchargement/visionnage des séries US en particulier est pour moi du pur bon sens : il faudrait attendre un an , pour avoir une diffusion à horaire fixe, et personnellement je regarde dans la mesure du possible avec des sous titres en anglais qui ne sont pas disponibles sur Canal +. En ce qui concerne les sériés diffusées sur d'autres chaines c'est bien pire : TF1 à un ordre de diffusion des épisodes qui défie le bon sens, et que dire de la VF qui, une fois que l'on a gouté à la VO, ne peut qu'irriter...J'avoue que c'est devenu une habitude et une solution de facilité et de confort qui revient à me dire que ne pas regarder ces séries serait comme ne pas ramasser un billet sur le trottoir. L'illégalité est beaucoup trop abstraite pour avoir un effet.

15. Le vendredi 20 mars 2009 à 21:26 par Giudice

Si je comprends bien, ca veut dire que pour passer à l'acte il faut une justification morale, fut-elle fausse. Donc il n'existe personne dont la morale soit différente de la votre (de la notre, disons)? Ou bien il n'existe personne qui se contrefiche de la morale?

J'avais toujours imaginé que certains criminels (pas délinquants) ne pouvait pas avoir de morale. Le tueur à gage se dirait donc, avant de tuer, un truc genre "Je lui rend surement un service, sa vie et pourrie et sa femme le trompe"?

Eolas:
J'ai défendu peu de tueurs à gage, je manque d'expérience. Je parierais pour un “ Si ce n'est pas moi, quelqu'un d'autre le fera ; donc il mourra et je garderai ma vie minable”.

16. Le vendredi 20 mars 2009 à 21:32 par Anticonstitutionnellement

Il me semble que tout dépend de ce que l'on place derrière l'idée de sain d'esprit. En effet, pour avoir assisté à de nombreuses audiences correctionnelles, je ne crois pas que la plupart des individus passés à l'acte aient auparavant eu ce sentiment d'impunité et de justification morale. J'avais plutôt l'impression que c'étaient des êtres qui, au moment de la réalisation de l'infraction, avait perdu certains repères. Je m'explique. Que dire du concubin qui, un soir de beuverie, va passer ses nerfs sur sa femme. Recherche-t-il une justification morale avant de cogner ? Pense-t-il à son impunité ? Je ne crois pas. L'acte, pour moi, relève plus de la pulsion à laquelle s'abandonne le délinquant du fait d'une levée des inhibitions. Le concept - si tant est qu'on puisse parler d'un concept - ne vaut, toujours à mon très humble avis, que dans les hypothèses d'infractions préméditées. Et il ne me semble pas qu'elles constituent le gros du travail des tribunaux correctionnels... Mais peut-être je me trompe. Par ailleurs, pour les infractions préméditées, ne devrait-on pas plutôt parler d'espoir légitime - au sens statistique - d'impunité que de certitude ?

17. Le vendredi 20 mars 2009 à 21:46 par Mathaf Hacker

Et si le passage à l'acte relève du libre arbitre ? Je sais que c'est une connerie, mais j'ai absolument besoin de le faire en toute conscience...

18. Le vendredi 20 mars 2009 à 21:51 par yellowrose

@14 marifline

Au sujet des séries US. Je vous suis très bien sur ce sujet.

Personnellement j'ai une autre motivation: un enfant aux US, les autres en France, il nous faut un quotidien bâti sur les faits ordinaires communs, pour pouvoir partager les émotions. Je déteste le zonage du monde, qui me case dans un silo des arriérés, et re-créé des frontières là ou il n'en existe plus.

19. Le vendredi 20 mars 2009 à 21:53 par Moi

La certitude de l'impunité ne me semble pas indispensable : il suffit d'estimer que le risque d'être pris ou que la sanction encourue ne sont pas suffisant pour contrebalancer les bénéfices de l'infraction.

De plus vous semblez penser que c'est nécessairement une erreur de penser ainsi (ou de penser qu'une action illégale puisse être légitime) alors que malheureusement il existe des lois qu'il est avantageux (ou juste) de violer.

20. Le vendredi 20 mars 2009 à 22:08 par Shylock

En ce qui concerne le téléchargement, je reste assez partagé. En effet, aucun impératif moral me l'interdit. N'ayant pas les moyens d'acquérir des DVD à 20 euros, mon éventuel téléchargement ne ferait pas perdre un sou à cette industrie. Mais l'illégalité de ce procédé m'intimidait tout de même, mais le risque valait la chandelle, d'autant plus que les films dont le téléchargement m'intéresse, sont souvent vieux et introuvables. Mais un peu moins intimidé depuis le projet HADOPI... D'ailleurs cher Maître, qu'en est-il de la conformité de HADOPI avec la CEDH ???

21. Le vendredi 20 mars 2009 à 22:21 par Escualdifargo

Oui alors moi je trouve que l'escroquerie peut aussi tenir avec le vol d'une formule de chèque si :

- le chèque était déjà signé par le titulaire du compte;

et

- le vilain escroc a produit un faux document au moment du paiement (fausse carte d'identité au nom de la victime avec photo de l'auteur, par exemple);

Si le chèque n'était pas signé et que c'est l'auteur de "l'escroquerie" qui l'a fait avant de l'émettre, on parlera pas d'escroquerie mais de contrefaçon et usage de chèque contrefait. En tout cas c'est comme ça que je vois les choses et pis c'est tout !

22. Le vendredi 20 mars 2009 à 22:21 par Luc Montesquieu

Si vous aimez le fantastique et le criminel, je vous conseille vigoureusement la lecture du manga DeathNote et vous verrez comment un tueur en série peut fonctionner... Vous verrez, il y a tout... le sentiment d'impunité et la justification morale...

Les amateurs d'animés me feront remarquer qu'il existe une version DVD et les plus avisés d'entre-eux ajouteront qu'il est disponible en streaming... Si si... Mais je n'en dis pas plus, un autre Luc pourrait nous lire...

23. Le vendredi 20 mars 2009 à 22:29 par Bebertii

Bonsoir @Moi : "il existe des loi qu'il est avantageux (ou juste) de violer" Avantageux, ok, je comprends. Mais juste, cela fait appel à votre point de vue. La morale peut être vue de 2 façons : interne à chaque être humain et de ce fait les lois représentent une espèce de consensus des personnes les énonçant (les plus forts, les plus influents). Rien ne justifie dans ce cas d'accepter la morale des autres, ni qu'elle est supérieure, si ce n'est que les autres sont plus forts.

Elle peut être vue comme universelle. Elle existe au dehors de l'esprit humain et au travers des lois, on essaie de s'en approcher. Si l'on choisit cette définition, une loi ne peut pas être injuste ! J'espère me tromper, mais n'est-ce pas celle que l'on nous vend ?

Ayant entendu des étudiants en droit en parler, il me semble que c'est ce qui sépare les jusnaturalistes des positivistes.

Etant scientifique et par conséquent positiviste, j'aurais tendance à prendre le point de vue de la morale interne à chaque homme. A partir de là, une loi n'est "juste" que si on l'accepte...

24. Le vendredi 20 mars 2009 à 22:30 par kji

j'apprécie de voir ainsi un débat de commentaire cité à titre d'exemple dans ce 'post'. quant au problème de l'internet et des téléchargements, il mériterait une autre approche que celui de la répression à tout prix.

25. Le vendredi 20 mars 2009 à 22:34 par atonnerre

@4Thomas

Est-ce que Thomas est puni pour son intervention de l'autre jour - où vous aviez l'air fâché-fâché contre lui ? Est-ce qu'il est mis à l'index (comme d'aucuns disent que notre souverain Pontife met le préservatif) ?

26. Le vendredi 20 mars 2009 à 23:14 par Rizgar Amin

Excellent billet, vraiment.
Comme ce vademecum des "compas". Maître Eolas, au sommet de son art blogistique, mêle vulgarisation pour le grand public et formation continue pour les professionnels, et même pour les amateurs qui se croient plus forts que les pros. Le tout dans un style rare n'omettant jamais la petite touche d'humour sarcastique (ah, le "et il a bien raison, quand on a droit à un avocat gratuit, autant refuser" :-) ).
Holà, mais je me crois en train de rédiger une quatrième de couverture, moi ! :-)

27. Le vendredi 20 mars 2009 à 23:57 par Skippy

Sans commentaire sur le fond, je voulais juste signaler une petite erreur de logique dans la première partie du billet : « la certitude d'être sanctionné » n'est pas le contraire de « la certitude de l'impunité ». Le contraire de la première est « l'incertitude d'être sanctionné », et celui de la seconde « l'incertitude de l'impunité ».

Ça peut sembler pointilleux de ma part, mais ça ne l'est pas : à mon avis la certitude de l'impunité est bien moins fréquente que l'incertitude de la sanction. Tout est histoire de probabilité. Estimer les chances de passer au travers des mailles du filet, et adapter sa situation en fonction : prendre ou ne pas prendre le risque d'être pris et potentiellement d'être sanctionné.

Par ailleurs, la certitude de l'impunité enlèverait beaucoup au plaisir de la transgression, la poussée d'adrénaline que tout le monde a connu au moins une fois (que ce soit pour une infraction pénale ou familiale…).

28. Le vendredi 20 mars 2009 à 23:59 par Paul

@ Maître Eolas : Vous supposez toujours que le criminel est, quelque part, irrationnel (en tout cas, qu'il est doté d'une rationalité propre et limitée - celle qui permet de supposer qu'un vol n'est "pas grave".)

Que pensez vous des théories économiques (au sein d'économie comportementale) des crimes ? Qu'on peut ainsi commettre un crime rationnellement, en pesant le pour et le contre, c'est à dire en fait une analyse coût / bénéfice ?

29. Le samedi 21 mars 2009 à 00:21 par Laurent

Sympathique billet qui nous confirme qu'au delà de la technique des lois, ce métier s'intéresse aussi (avant tout ?) à l'humain et à la complexité de ses motivations. Peut-être pas pour caractériser la réalité de la faute, mais probablement pour essayer d'évaluer la peine ?...?

La petite anecdote de l'étudiant en droit me procure un sourire... et une interrogation : la vie privée de cet étudiant a été exposée à son père alors qu'il ne le souhaitait probablement pas. Alors, ok pour dire qu'il a sur le coup de plus gros ennuis que ça, mais ne peut-on protéger les prévenus sur ce genre de problématique ???

30. Le samedi 21 mars 2009 à 00:28 par Alicanthe

Votre exemple montre surtout que quand on est assez bête pour faire des fautes d'orthographe dans un faux, il faut laisser la malhonnêteté aux gens plus intelligents.

Quant à votre démonstration, elle ressemble à un sermon de jeune prêtre frais émoulu du séminaire. Vous avez juste oublié de faire référence à la mauvaise conscience qui, sans aucun doute, poursuit le délinquant jusque dans ses draps.

Au risque de vous décevoir, il existe des multitudes de délinquants impunis, à commencer par le citoyen qui a ramassé un billet dans la rue et l'a gardé, ou par celui qui un jour a pénétré dans un village à 101 km/h parce qu'il n'a pas vu le panneau. Il existe même des trafiquants de drogue qui ont coulé une vie longue et heureuse dans le luxe et la luxure.

Si l'on met de côté toute morale de grenouille de bénitier, se pose la vraie question : est-il souvent rationnel (au sens de la maximisation du profit attendu) de commettre des délits, comme les faux évoqués par votre lecteur.

La réponse est évidemment oui. Par exemple quand on est en situation irrégulière et qu'on cherche du travail. Ou quand il s'agit de mettre en balance la quasi-certitude d'aller en prison en l'absence de garantie de représentation avec un risque modéré de se faire prendre pour usage de faux.

On ne saurait encourager ces détestables pratiques (pour éviter de vous causer des soucis), mais il faut reconnaître que bien mal acquis profite parfois.

31. Le samedi 21 mars 2009 à 00:42 par Xiawi

Tenez quelques exemples de justification morale (...) Le sans-papier: je bosse ici, je paye mes impôts et je n'aspire qu'à une vie normale

Je sais bien que le problème ici n'est ni le prévenu ni le juge mais le législateur (et ceux qui l'ont élu...). Néanmoins, dans des cas tels que celui-ci, j'estime effectivement qu'Antigone ne méritait pas de si mal finir...

32. Le samedi 21 mars 2009 à 00:47 par Yves D

@ Eolas:

A propos de votre conclusion, je dirais que l'on ne peut pas dépasser ce problème du mensonge suite à une infraction avec seulement les conseils d'un avocat, qui est nécessaire, mais s'il n'y a pas prise de conscience, si les délinquants ne s'amendent pas, on ne peut pas dépasser le risque de récidive avec l'augmentation des peines. Au contraire, les jugements trop sévères augmentent le problème.

Bon allez, c'est le WE ... il fallait bien sûr remplacer "mensonge suite à une infraction" par "téléchargement illégal", "augmentation des peines" par "loi Hadopi", et "jugement trop sévères" par "suppression de la connexion" pour retrouver une phrase devenue célèbre ces derniers jours ;-)

33. Le samedi 21 mars 2009 à 00:50 par marifline

@22 Luc Montesquieu Death Note est un excellent manga que j'ai découvert sur le net, et dont j'ai ensuite acheté tout les tomes ! ( comme quoi :)) Le genre que l'on peut conseiller aux personnes qui ont de mauvais a priori sur les mangas : les dessins sont raffinés et réalistes et l'intrigue très élaborée et franchement addictive.

Vous avez raison, il illustre tout à fait le fonctionnement de l'esprit criminel. Avec le pouvoir d'éliminer qui l'on veut et des ambitions de "justice" on en vient rapidement à tuer des innocents à la pelle.

34. Le samedi 21 mars 2009 à 01:04 par Bruno Kant

@Gascogne en 8, "après des années d'audience, les magistrats ont un sixième sens concernant le mensonge"

Comment définissez vous le mensonge?

35. Le samedi 21 mars 2009 à 01:20 par Luc Montesquieu

@ Paul et Alicanthe

Oui, comme disait Gabin... C'est vrai, oui, c'est vrai ! On peut commettre une infraction en en étant pleinement conscient et en maximisant sa satisfaction sous contrainte... Le juriste n'est-il pas Badman ?

Alors, prenons le cas de Crétin, Crétin est un jeune automobiliste qui a parcouru le code de la route et a lu son article R413-14. Crétin arrive en agglomération, il est un peu pressé, sa copine l'attend pour une soirée Harry Potter, il voit le panneau, comprend bien qu'il doit rouler à 50 Km/h maximum mais il se dit qu'il est pressé... Alors, que va-t-il faire ? Personnellement, j'aurais conseillé à Crétin de respecter la vitesse maximale autorisée... Et bien Crétin va se souvenir de cet article R413-14 du code de la route et va analyser la situation juridique comme suit...

"Si je roule à 50, je serai en retard, je ne risque rien mais je vais me faire engueuler par ma copine, en plus, son père, un certain Eolas, est maitre de cérémonie ce soir, c'est la soirée où il ne faut pas être en retard... Alors, j'appuie sur le champignon... Maintenant, à quelle vitesse vais-je rouler ? Le code de la route dispose que pour tout conducteur d'un véhicule à moteur, le fait de de dépasser de moins de 50 km/h la vitesse maximale autorisée est puni de l'amende de 135€... Le nombre de points, je m'en moque, au pire, je ferai un stage de récupération de points... Alors, quitte à risquer 135€, autant rouler à 99 km/h pour 50km/h"

D'accord, je vois certains me dire que c'est risqué... C'est vrai, il y a un risque mais le fait que la peine soit la même en ville pour 1km/h de dépassement que pour 50 pose un véritable problème de sécurité routière...

36. Le samedi 21 mars 2009 à 01:24 par Santé publique

Maître vous êtes comme le bon vin, jour après jour vous gagnez en valeur... Je ne me lasse pas de vos billets. Je tique juste sur une petite chose: c'est "résout" et non "résoud" (dernière phrase). Un mnémotechnique: les verbes en -dre font ds ds d, sauf les verbes en -indre et -soudre qui perdent leur "d" et font s s t. Meilleures salutations

37. Le samedi 21 mars 2009 à 01:35 par Bziaou

@ Alicanthe (30)

Mais ce que vous dites est-il contradictoire avec ce que dit Maître Eolas? '' Au risque de vous décevoir, il existe des multitudes de délinquants impunis, à commencer par le citoyen qui a ramassé un billet dans la rue et l'a gardé, ou par celui qui un jour a pénétré dans un village à 101 km/h parce qu'il n'a pas vu le panneau. Il existe même des trafiquants de drogue qui ont coulé une vie longue et heureuse dans le luxe et la luxure. '' Oui, il existe une impunité. D'où le sentiment d'impunité qui va naitre chez certains et faciliter le passage à l'acte! Tout ceci est lié: Eolas ne se pose pas dans une posture moraliste, il essaie de montrer le mécanisme psychologique qui y mène. Et votre réplique l'illustre bien: "tout le monde sait qu'il y a des gens impunis". Donc je peux tenter aussi...

''Si l'on met de côté toute morale de grenouille de bénitier, se pose la vraie question : est-il souvent rationnel (au sens de la maximisation du profit attendu) de commettre des délits, comme les faux évoqués par votre lecteur. La réponse est évidemment oui.''

Oui, d'ailleurs l'économie du droit nous enseigne qu'une sanction n'est efficace que si elle conduit le potentiel délinquant à se demander si, rationnellement, le délit vaut le coup d'être tenté, en termes de répression et de chances d'être pris. La réponse va souvent être oui, du moins dans son esprit (la réalité fera qu'il sera peut être pris en flagrant délit).

Ce que vous mettez en évidence, c'est exactement les deux motifs que retient Eolas: la certitude d'être impuni (de toute façon, les automobilistes ne sont pas pris, ni les dealers, dont tout va bien) et la motivation (rationnellement, mieux vaut tenter le coup).

38. Le samedi 21 mars 2009 à 01:42 par Bruno Kant

Ce qu'on peut lire là bas est au moins aussi affreux qu'ici:

"Les enfants en dessous de seize ans et les parents proches du prévenu (père, mère, frère, sœurs, enfants, grands-parents, époux même divorcés) témoignent sans prêter serment. Ils pourront toutefois être autorisés à prêter serment s’il y a accord de toutes les parties (art. 447 à 449 du CPP).

En cas de faux témoignage avéré, ce qui n’est pas facile à prouver, le président peut ordonner au témoin de demeurer à la disposition du tribunal et le faire garder par la force publique, puis après la lecture du jugement le présenter au parquet pour d’éventuelles poursuites (art. 457 du CPP)."

39. Le samedi 21 mars 2009 à 02:06 par sayan

un peu de relativisme absolu : qu'est-ce que la vérité, après tout ? Toutes les vérités seraient-elles bonnes à dire, même pour un avocat ?

40. Le samedi 21 mars 2009 à 02:20 par Yves D

Droit Canon n° 2360 (version de 1917, abolie en 1983) :

§ 1 Ceux qui fabriquent ou falsifient des lettres, décrets ou rescrits du Siège apostolique ou se servent sciemment des mêmes lettres, décrets ou rescrits, contractent une excommunication ‘latae sententiae’ spécialement réservée au Saint-Siège.

41. Le samedi 21 mars 2009 à 02:57 par cyber-cola

Je suis effaré. Je n'ai rien compris à votre billet sinon qu'il ne serait qu'un exercice d'équilibre pour ne pas cautionner un commentaire qui appelait à un acte illégal.

Vous nous expliquez qu'un délinquant agit généralement avec le sentiment d'impunité et de respect de la morale. La belle affaire! Tout le monde agit avec ces deux mêmes sentiments. Même pour effectuer des actes légaux. Quand je vais acheter du pain, et que je le paye avant de l'emporter, j'agis avec un sentiment d'impunité et de respects de la morale (avec un peu d'appétit aussi).

Après cela, savoir si c'est vraiment moral ou si cela restera impuni ne dépend pas de moi. La morale et la punition sont des sports collectifs sur lesquels l'individu a peu de prise. C'est le reste de la société qui décide. Avec des valeurs mouvantes.

En attendant je suis bien obligé de me fabriquer une logique sur ce qui sera puni ou pas et une morale qui me permet de vivre. Et j'y crois fortement, certainement comme vous et comme tous les délinquant.

A moins que votre propos ne vise, comme vous l'aviez fait dans un précédent billet sur l'indépendance entre la morale et la légalité, à nous expliquer que ce n'est pas parce qu'un acte est impuni qu'il est légal. Que ces deux notions n'ont aucun lien de causalité entre elles.

42. Le samedi 21 mars 2009 à 03:10 par Jilian

"Cependant, aucun d'entre eux, quelles que soient les bonnes raisons qu'il se donne, ne passerait à l'acte s'il avait la certitude d'être sanctionné" C'est certes l'état d'esprit de la loi, mais un rapide examen de, euh, toute l'histoire humaine montre assez clairement que la dissuasion n'a jamais réussi à supprimer le crime L'objection à cette déduction est que les sanctions et vérifications n'ont jamais été assez fortes : le totalitarisme ou rien ?

43. Le samedi 21 mars 2009 à 03:24 par Alex

Ce qui est étonnant, c'est la bêtise assumée. Et mise en avant comme un mode de vie. Dans le genre, le commentaire de Cyber Cola est fabuleux. L'esprit de contradiction contre... rien... Il n'a même pas compris avec quoi il n'est pas d'accord (il l'avoue lui-même, et le prouve dès la ligne en dessous). Mais en tout cas, il n'est pas d'accord, ah ça, non ! T'as raison ! Rebelle-toi Cyber Cola !

44. Le samedi 21 mars 2009 à 05:41 par PrometheeFeu

Regle 1: Ne vous faites pas attraper.

Regle 2: Ne faites rien si vous ne pouvez pas en accepter les conséquences parce que vous avez peu de chance de respecter la regle 1.

Ou comme le disent les policiers Americains (ces laxistes qui laissent les avocats acceder au dossier des le debut de la procedure) "Don't do the crime if you can't do the crime."

@cyber-cola:

En effet. C'est pour cela que l'on cherche a vous offrir le sentiment d'impunite quand vous achete du pain et pas quand vous piquez un sac...

45. Le samedi 21 mars 2009 à 05:49 par NMONNET

Je vole un sac. Mémère a un sac en moins.

Je télécharge le dernier Obispo. Obispo a toujours autant de daubes. Mais c'est interdit.

Je vends un godemichet en Alabama. C'est aussi très interdit.

Je copie un vieux film de Disney. C'est super mal, de la contrefaçon, mal mal mal.

Si Disney avait fait ce film à partir d'une œuvre existante datant de quelques dizaines d'années, supposons, hein, ils n'auraient pas eu à payer de droits d'auteurs, parcequ'à l'époque la durée en étant beaucoup plus courte. Mais comme ils ont plein de pognon ils ont pu faire changer la loi, pour qu'elle s'applique à leur produit dérivé et lui seul. Et ils n'ont pas eu besoin de se justifier, eux, comme de vulgaires hadopilleurs!

Pareil si les brevets logiciels sont un jour (probablement très proche) autorisés par l'OEB, nous les auteurs et utilisateurs de logiciels libres deviendront automatiquement contrevenants. Et on va devoir se trouver des excuses.

46. Le samedi 21 mars 2009 à 09:00 par proc hure

Il est toujours risqué de verser au dossier des juges (avec communication préalable au parquet) des pièces douteuses en croyant que tout ça va passer comme lettre à la poste.

Merci donc, Maître, d'avoir attité l'attention de vos lecteurs sur ces tours de passe-passe, car il n'est pas agréable d'être pris pour un coing.

A Alicanthe en 30,

Je ne crois pas que ramasser un billet dans la rue (sans avoir usé de stratagème ou de violences contre le possesseur), soit un acte délictueux.

Cet acte ne s'analyse ni en un vol, qui est la soustraction frauduleuse de la chose d'autrui (en l'absence de fraude et même de soustraction), ni en une escroquerie ni en un abus de confiance.

Et sur le plan de la morale et sauf exception (ex:le billet tombe de la poche de la personne devant moi) je ne pense pas que ce soit répréhensible.

47. Le samedi 21 mars 2009 à 09:00 par Jonas

Bonjour Maître,

Je ne me permettrais qu'un court commentaire sous cet intéressant article, mais il sera pour le "téléchargement".

Au delà du sentiment d'impunité, il est possible de penser que, pour une grande majorité des internautes, il peut se poser, simplement, la question de l'illégalité.

D'abord, bien que la propagande informe volontier les internautes sur le caractère délictueux du téléchargement, elle n'atteint pas tout le monde. Je pense qu'il se trouve encore des internautes qui ne se sont pas demandé si leurs actes appelaient une quelconque application d'un quelconque règle de droit.

Viennent ensuite ceux qui se sont posé la question au moment où les internautes étaient attraits devant les juridictions répressives... Ceux-là pourraient volontier penser que c'est légal, puisque, d'une part, le seul "téléchargement" a été reconnu a plusieurs reprises et que, d'autre part, la lecture des décisions sanctionnant des internautes enseigne que n'ont été sanctionné que des internautes reconnaissant avoir diffuser (ce qui est de la contrefaçon) des oeuvres de l'esprit...

Enfin, les fins juristes (ou ceux qui pensent l'être) ont connaissance de l'arrêt de la Cour de Cassation estimant qu'il n'y a pas de copie privée lorsque la source est illicite, mais pensent surement, pour un certain nombre en tout cas et après lecture dudit arrêt, cette décision est critiquable et sera surement remise en cause dans un futur plus ou moins proche.

Il ne s'agit donc pas nécessairement d'un sentiment d'impunité, mais de l'absence de conscience de commettre, ou, au contraire de la certitude de ne pas commettre, une infraction.

Bien Cordialement

Jonas

48. Le samedi 21 mars 2009 à 09:46 par Gascogne

@ Bruno Kant (34) : le beau sujet de dissertation que voilà.

Le mensonge, c'est :
- travestir consciemment la vérité : je ne suis jamais venu dans votre riant département, et je ne m'explique pas les flaques de sang contenant mon ADN que les services de police ont retrouvées dans la maison cambriolée.
- omettre quelques menus détails, en espérant que ça passe : j'étais seul sur ce braquage.
- enjoliver les choses pour se présenter sous son meilleur jour.
- la mythomanie pure et dure : je vous assure que j'étais agent de la DST avant de devenir gardien de camping.
- celui qui ne vise pas le juge ou la victime : impossible pour certains mineurs de reconnaître en présence de leurs parents les faits qu'ils avaient pourtant parfaitement reconnus en garde à vue ou seuls devant le juge.
- et celui qui est sans doute le cauchemar des magistrats : le déni. Refoulement inconscient de faits bien trop graves pour être assumés par la conscience. Les faits ont bien été commis, mais l'auteur est lui même persuadé de ne pas les avoir commis.
Mais le mensonge peut encore prendre bien d'autres formes...

49. Le samedi 21 mars 2009 à 09:52 par ramon

In cauda venenum

On peut déjà imaginer la nouvelle entrée qui figurera dans le Larousse 2010 :

"Se faire l'avocat du Pape"

Inventée par Me Eolas, cette action brillante et hardie s'exécute en deux temps 1. Prendre à rebrousse-poil son public habituel pour le sidérer 2. Crétiniser son client en tentant d'expliquer ce qu'il a bien pu vouloir dire avec ses mots à lui.

50. Le samedi 21 mars 2009 à 10:11 par Le Chevalier Bayard

" Passage à l'acte et périmètre du droit"

@ Gascogne

Peut-on dire que la stratégie de défense des avocats (auxiliaires de justice)d' Yvan Colonna est basée sur le mensonge en l'absence de preuve de la partie civile.

51. Le samedi 21 mars 2009 à 10:36 par El Re

Pas sûr de ce que vous entendez par "couper les vivres", mais en ce qui me concerne il est certain que je serai toujours prête à aider ma fille, d'une manière ou d'une autre, si j'en ai les moyens, car elle reste ma fille.

52. Le samedi 21 mars 2009 à 11:10 par Juris

Que pensez-vous de ce fameux adage en droit de la concurrence communautaire: Pas vu, pas pris... Pris, pendu! Pour ma part, je le trouve source d'inspiration intarissable pour les futurs juristes d'entreprises

53. Le samedi 21 mars 2009 à 11:11 par hervegg

Très joli billet: bonne analyse sur le sentiment d’impunité.... et chute pleine d’esprit. Félicitation maitre

54. Le samedi 21 mars 2009 à 11:25 par Ancilevien

@ 46. Le samedi 21 mars 2009 à 09:00, par proc hure

Je ne crois pas que ramasser un billet dans la rue (sans avoir usé de stratagème ou de violences contre le possesseur), soit un acte délictueux.

Eolas nous a dit il y a pas longtemps que c'était illégal, j'ai cherché où sans succès, mais allez retrouver une histoire de billet (de banque) au milieu de pleins de billets (de blog).

55. Le samedi 21 mars 2009 à 12:11 par Skippy

@proc hure en 46 : Je ne crois pas que ramasser un billet dans la rue (sans avoir usé de stratagème ou de violences contre le possesseur), soit un acte délictueux.

Ne s'agit-il pas d'un objet trouvé, et n'est-on pas censé le rapporter au commissariat ?

56. Le samedi 21 mars 2009 à 12:20 par Skippy

@Ancilevien en 54 : Une petite astuce : notre ami (ou pas) google possède une option remarquable, qui permet de rechercher une expression ou des mots sur un site. Ainsi, la recherche :

site:http://www.maitre-eolas.fr/ "billet dans la rue"

ne renvoie que deux résultats, à savoir la page sur laquelle nous discutons et le billet que vous recherchez (qui est en fait de fantômette).

Notre cher maître y écrit, en réponse à la question « ramasser un billet est un vol? le vol est la soustraction frauduleuse de la chose d'autrui. Quand je ramasse un billet par terre, il n'y a pas de soustraction frauduleuse il me semble, non? » (commentaire #1) :

il y a vol, à moins que son propriétaire ne l'ai laissé là intentionnellement à destination du premier quidam qui passe. Le vol est l'appropriation (qui consiste à se comporter comme le propriétaire de la chose) frauduleuse (sachant qu'on n'est pas le propriétaire). Techniquement, ramasser le billet ne constitue pas encore le vol faute de certitude sur l'appropriation, la volonté de se comporter en propriétaire de la chose (vous pouvez le ramasser pour l'amener au commissariat, ou pour le brandir en criant : « qui a perdu ce billet ?»). Il sera constitué quand vous vous comporterez en propriétaire de ce billet : soit que vous le dépensiez, soit que vous le conserviez un temps suffisant pour révéler votre absence d'intention de rechercher son véritable propriétaire. Vous deviendrez légitimement propriétaire de ce billet au bout de cinq ans, durée de la prescription acquisitive de mauvaise foi en matière mobilière (le vol sera quant à lui prescrit au bout de trois ans). (commentaire 10)

57. Le samedi 21 mars 2009 à 12:36 par Cavok

@ Gascogne sous 48 :

Du mensonge.

J'ai récemment suivi à la télé, suite à l'information relayée par M°Eolas, les cinq épisodes du procès en assises diffusé dans le journal de 13 heures de France 2.

J' ai vu un accusé y admettre son crime, ceci malgré sa gravité, sa violence, son inhumanité. Je l'ai vu s'excuser aussi, maladroitement, mais il l'a fait. Et regretter aussi.

En face, j'ai vu des parties civiles qui, faisant manifestement peu de cas de cette attitude positive, n'hésitait pas à faire témoigner qui on sait. Et c'était discutable, compte-tenu de l'absence d'éléments matériels concernant le caractère raciste du crime.

Réquisition du procureur : 25 ans. Verdict 25 ans. Je ne suis pas de la maison, ça m'a parut beaucoup.

Mais peut-être que le 25 ans requis prenaient déjà en compte l'attitude positive de l'accusé ? Peut-être aussi un jury populaire est-il plus sévère qu'un jury professionnel ?

Mais s'il y avait eu déni, mutisme, mensonges, affront fait à la famille, c'était combien alors ? Ça paie de ne pas mentir ou quoi ?

58. Le samedi 21 mars 2009 à 12:38 par Ancilevien

@56. Le samedi 21 mars 2009 à 12:20, par Skippy

Je connais le "site:", mais je n'ai pas trouvé la bonne phrase à rechercher.

Tout l'art d'obtenir les bonnes réponses et dans l'art de poser les bonnes questions.

Parfois je ne suis pas artiste. Merci pour la complétion de réponse néanmoins.

59. Le samedi 21 mars 2009 à 12:39 par zorro

Je ne comprends pas la morale de la petite histoire.
Il y a un avocat qui n'a pas fait correctement son travail.
Il y a un juge qui fait la justice (au sens du droit).
Il y a un délinquant qui va faire 1 an avec sursis pour 60 euros.
Est ce que pour vous, tout cela est censé representer une justice (au sens de l'institution) qui fonctionne bien ?

Eolas:
Non. C'est censé démontrer que produire des faux en justice n'est pas une bonne idée.

60. Le samedi 21 mars 2009 à 12:56 par Adèle

On ne peut alors pas imaginer une action illégale mais légitime ? (accueillir des Juifs en fuite chez soi sous Vichy par exemple) ?

61. Le samedi 21 mars 2009 à 13:59 par Gaël Martin

orthographe Ce que vous avez dit ? ou plutôt ce que vous n'avez pas écrit ? C'est : « Ça ne résout pas le problème... » (avec un T, bien sûr)

62. Le samedi 21 mars 2009 à 14:35 par dwarfpower

Le téléchargeur pirate : “ De toutes façons, j'aurais pas acheté ce CD/DVD, et j'ai pas les moyens d'aller au ciné, alors ils perdent rien. Puis de toutes façons, ils sont déjà pété de thunes, eux. ”

Cher maitre, quelques contestations de cette affirmation.

La loi prevoit une exception au monopole d'exploitation de l'auteur, prévu par l'article 122-5 comme vous le savez. Cette exception permet la reproduction sous quelques réserves: - usage strictement privé du copiste - respect du test en trois étape (une nouveaute de 2006 celle là )

aucune autre condition prévue par le texte.

Reste une question en suspend our ce qui concerne le téléchargement sur internet, celui de la licéité de la source dans un acte de reproduction non autorisé sous couvert d'exception copie privée. Pas de jurisprudence sur le sujet, à part une décision de la cours de cassation qui avait cassé un jugement de la cours de Montpellier car elle n'avait pas répondu à la question de la partie civile sur la nécessité d'une source légale pour que l'exception copie privée puisse jouer. La cours n'avait pas donné d'indication sur le sens à suivre et avait renvoyer en appel à Aix en Provence pour fournir une réponse sur le sujet.... Sur la licéité donc, on a un texte muet, un avis de la cours de cassation non interprétable sur le sujet, et des magistrat d'appel qui , sommé par la cours de cassation de fournir une réponse, sachant que le problème est central à un usage très répandu dans la société, et sur un sujet où la doctrine n'est pas unanime, a choisi de ne pas répondre.

Pour la peine on a un législateur qui en 2006, travaillant sur la question, sachant que le statut juridique de la source n'était pas défini et qu'il était crucial, ayant évoqué la question lors de ses discussions en séance, a aussi choisi de ne pas clarifier le texte sur le sujet. Ceci posé, un copieur pour bénéficier de l'exception, doit vérifier que son acte rentre dans le cadre du test en trois étapes. Il doit donc vérifier : - qu'il n'empêche pas l'exploitation normale de l'œuvre - que les préjudices qu'il fait subir à l'auteur ou aux ayant-droit ne sont pas disproportionnés.

En conséquence, vérifier que l'acte de reproduction ne se substitut pas à un achat est l'obligation légale qui repose sur le reproducteur, ou du moins est un des moyens qu'à le reproducteur de remplir ses obligations légales. Vous ne pouvez donc pas le lui reprocher et transformer cette vérification en une pseudo justification morale d'un acte répréhensible.

63. Le samedi 21 mars 2009 à 14:50 par Olive

Maître, je vous prie d'excuser la "sortie" qui va suivre, assez éloignée de l'objet initial de votre post, mais je me sens obligé de rebondir sur certains commentaires. Je suis profondément agacé par le fait que tout au long des derniers débats relatifs au téléchargement et donc, entre autres choses, à l'accès à la culture, personne ne se soit souvenu de l'existence de lieux où cet accès est pour ainsi dire gratuit. Quand je dis personne j'entends: ni les législateurs, ni ceux qui défendent le téléchargement par l'argument d'une absence d'alternative à celui-ci, au titre que la culture "classique" serait jugée non rentable pour la Fnac et consorts.

Un petit rappel me semble donc s'imposer à l'attention des pourfendeurs des majors et des institutions-laquais-du-grand-capital (ironie inside). Savez vous que moyennant une somme modique, allant de 0 et 50€ par an, avec tarifs réduits pour les jeunes, les retraités, les chômeurs, les familles nombreuses, les avocats (ah, en fait non, pas eux), souvent à moins de 20 minutes à pied de chez vous si vous vivez en ville (mes excuses aux ruraux qui lisent ce blog) il existe dans presque chaque commune d'importance des lieux où vous pourrez :

  • emprunter plusieurs DVD par jour si vous êtes cinéphile,
  • généralement autant de CD si vous êtes mélomane,
  • les partitions qui vont avec si vous êtes musicien et/ou musicologue
  • le roman du film ou la biographie de votre compositeur / réalisateur / musicien préféré, si vous êtes vraiment fan.
  • et même réserver le tout par internet si vous n'aimez pas perdre votre temps.

Et là, je le sais, certains trépignent d'impatience en se demandant où trouver cet endroit magique dont on lui aurait odieusement dissimulé l'existence. Je suis bon prince, c'est la médiathèque municipale. Ne me remerciez pas, c'est mon métier.

Certes nous travaillons encore avec des supports matériels et certains trouvent visiblement que c'est sale (quoiqu'on numérise à tour de bras). D'un autre côté si vous aimez les raretés il est nettement plus rapide de faire un détour en sortant du bureau pour emprunter un DVD que d'attendre 3 jours la fin d'un téléchargement mis à disposition par un seul péquin pour découvrir que, manque de pot, le fichier est incomplet ou corrompu. Tout ça étant parfaitement légal, vous agissez en toute impunité.

Pour ce qui est de la certitude d'accomplir un acte moral: vous donnerez un sentiment de travail accompli à des bibliothécaires souvent passionnés (les quelques vieilles harpies à chignon qui rôdent encore dans nos rayonnages ne sont conservées qu'à des fins historiques et se font de toute façon de plus en plus rares). Ca nous rend heureux de nous sentir utiles. Et si vous ne trouvez pas ce que vous cherchez dites vous que nous sommes toujours ouvert aux suggestions, ça nous évite de trop nous creuser la tête pour nos acquisitions.

64. Le samedi 21 mars 2009 à 14:56 par Dam

ils sont déjà pété de thunes, eux

pété avec un "s" peut-être.

65. Le samedi 21 mars 2009 à 15:05 par Bruno Kant

@Gascogne en 48 "le beau sujet de dissertation que voilà. ... et celui qui est sans doute le cauchemar des magistrats : le déni. ... Mais le mensonge peut encore prendre bien d'autres formes..."

J'adore Marie-Thérése dans Tatie Danielle: "mais madame, j'vous jure que j'ai jamais couché avec un garçon!" C'est plus qu'un sujet de dissertation, Laurent Lèguevaque en a fait tout un plaidoyer.

Pour en revenir au billet initial et à la morale (si chère à son auteur) plutôt qu'au droit et à ses racines, ce matin, dans ma boite aux lettres, je trouvais Beccaria préfacé par Badinter ainsi que Charles Dickens entre normes et déviance (de la société victorienne). C'est un étrange hasard qui semble vouloir que de tels thèmes soient récurrents dans la littérature, certains blogs, les prétoires et divers débats d'actualité (tel que la suppression du juge d'instruction).

On trouve aussi Beccaria et d'autres fondamentaux ou objets d'études aux éditions du Boucher, au format PDF. Cette semaine, j'y retournais après avoir lu une brève citation de Foucault dans Normes et Déviances.

Je viens de commander Face à la police / Face à la justice dont notre hôte fait ici la promotion. L'ouvrage pourrait être assez fantastique pour peu qu'on l'éclaire avec une catégorie très ancienne de la jurisprudence de la cour de cassation ou même avec celle de quelques petits juges.

66. Le samedi 21 mars 2009 à 15:41 par Mussipont

@ Bruno Kant : "mais madame, j'vous jure que j'ai jamais couché avec un garçon!"

C'est dans "La vie est un long fleuve tranquille" du même Chatiliez. :)

67. Le samedi 21 mars 2009 à 16:06 par Bruno Kant

@Mussipont, vous avez raison :-)

http://www.youtube.com/watch?v=E7XV6z_0RPA

68. Le samedi 21 mars 2009 à 16:07 par Babar

"il a bien raison, quand on a droit à un avocat gratuit, autant refuser :-) ".

Il y aurait des avocats exercant pro-bono en France ? Mmh je croyais que cela etait reserve aux cabinets anglo-saxons dans les pays democratiques comme les Etats Unis ou le Royaume Uni..

Peut avez voulu dire quand on a peut obtenir un avocat paye 192 euros par le ministere de la justice au titre de l'aide juridictionelle totale si on a de faibles revenus ? mhh si on est catholique c'est bien parce qu'il nous aide a confesser tout nos peches..et a montrer de la contrition devant le pretre , le juge de la comparution immediate...

Mais le procureur de la Republique n'oriente pas uniquement les dossiers vers la comparution immediate des mis en cause qui ont des faibles revenus et appartiennent a des minorites visibles non ? Tenez l'exemple du fils de notre president..ah moi mais non la plainte a ete classe sans suite et c'etait une citation directe..

En tout cas c'est bon a savoir cette tolerance zero pour les "faux et usage de faux" du procureur de la Republique!

Cela doit faire trembler les agents de police quand ils redigent leur proces verbaux d'audition sachant qu'il risque 15 ans de prison en cas de "faux en ecriture publique par un agent depositaire de l'autorite publique" et la cour d'assise...

69. Le samedi 21 mars 2009 à 16:09 par Ferdydurke

@ Gascogne

Comment un magistrat considère-t-il la mythomanie? Un mythomane est-il considéré comme un menteur sincère qui croit vraiment que ce qu'il dit est la réalité? J'emploie le mot réalité à dessein dans le sens "la réalité qui est la sienne", donc totalement subjective. Je croirais bien en l'existence de la vérité mais j'attends pour cela que quelqu'un soit capable de décrire un fait ou une chose en toute objectivité (oui, Me Eolas, il y a Dieu... je sais mais n'aggravez pas le problème, je vous prie ;-) )

Le cas du mythomane me semble donc assez proche de celui de la personne qui est dans le déni tel que vous le définissez. Un cas pathologique, en quelque sorte. Comment un magistrat l'aborde-t-il?

70. Le samedi 21 mars 2009 à 16:18 par Babar

"il a bien raison, quand on a droit à un avocat gratuit, autant refuser :-) ".

Il y aurait des avocats exercant pro-bono en France ? Mmh je croyais que cela etait reserve aux cabinets anglo-saxons dans les pays democratiques comme les Etats Unis ou le Royaume Uni..

Peut avez voulu dire quand on a peut obtenir un avocat paye 192 euros par le ministere de la justice au titre de l'aide juridictionelle totale si on a de faibles revenus ? mhh si on est catholique c'est bien parce qu'il nous aide a confesser tout nos peches..et a montrer de la contrition devant le pretre , le juge de la comparution immediate...

Mais le procureur de la Republique n'oriente pas uniquement les dossiers vers la comparution immediate des mis en cause qui ont des faibles revenus et appartiennent a des minorites visibles non ? Tenez l'exemple du fils de notre president..ah moi mais non la plainte a ete classe sans suite et c'etait une citation directe..

En tout cas c'est bon a savoir cette tolerance zero pour les "faux et usage de faux" du procureur de la Republique!

Cela doit faire trembler les agents de police quand ils redigent leur proces verbaux d'audition sachant qu'il risque 15 ans de prison en cas de "faux en ecriture publique par un agent depositaire de l'autorite publique" et la cour d'assise...

71. Le samedi 21 mars 2009 à 16:23 par Diogène

@ Bruno Kant "J'adore Marie-Thérése dans Tatie Danielle: "mais madame, j'vous jure que j'ai jamais couché avec un garçon!" "

Plutôt "La vie est un long fleuve tranquille " me semble-t-il ?

A+ Diogène

72. Le samedi 21 mars 2009 à 16:29 par PrometheeFeu

Je rappel a tout ceux qui justifient le téléchargement illégal par l'absence de préjudice mesurable l'un des aspects importants de ce blog: Eolas est avocat pas législateur. (Eolas 2012?) Donc il nous expose (en général) des arguments lex lata. Pour les discussion lex ferenda, il faudra aller chercher ailleurs... Mais bon, cela dit, vous avez raison.

73. Le samedi 21 mars 2009 à 17:12 par Machin

Donc il nous expose (en général) des arguments lex lata.

Du latex!! encore !!

74. Le samedi 21 mars 2009 à 18:11 par Dante Timélos

Cher maître,

Puis-je ajouter quelques exemples de mon cru à votre longue liste de « passages à l’acte » engendrés par un « sentiment d’impunité » et des « justifications morales » ?

Eolas:
Je parie que vous n'allez pas attendre ma réponse pour le faire…

Prenons un juge. On amène devant lui un pauvre type, qui a volé/vendu du cannabis/escroqué les assedics (rayez la mention inutile). Qu’est-ce qui justifie que notre juge va passer à l’acte, c’est à dire envoyer notre gars en taule pour lui apprendre que quand on est pauvre on le reste et on va bosser en silence, et que la vie facile, ce n’est pas pour lui mais pour ceux qui sont plus élevés dans la hiérarchie sociale ? Et bien une « justification morale » (notre ordre social est juste et démocratique, donc il n’y a aucune raison de ne pas s’en satisfaire comme tout bon travailleur) et un « sentiment d’impunité » (le temps où les fonctionnaires de l’Etat craignaient une quelconque vengeance prolétarienne est révolu depuis longtemps…)

Eolas:
La révolution est donc révolue ? On tourne en rond, ma parole.

Prenons à présent un avocat. Qu’est-ce qui lui permet d’affirmer que « certaines formes de délinquance idéologique » méritent « une plus grande sévérité » ? Et bien, sans doute quelque justification morale (ces gens là ayant l’outrecuidance de refuser de se renier, on les garde plus longtemps au trou, comme les anciens d’AD par exemple) et un certain sentiment d’impunité (notre avocat ne signe pas de son vrai nom…)

Eolas:
Tandis que vous, Dante Timélos, c'est votre vrai nom ?

Sur le fond, cher maître, je me permets également de préciser que :

- un certificat non vérifiable n’est pas un faux. Pourquoi ? Et bien parce qu’un faux c’est un document qui est reconnu tel, et quoi que vous disiez cela n’arrive que très rarement.
Votre exemple nous apprend seulement, comme le fait remarquer un de vos lecteurs, que quand on est un abruti on ne s’en sort pas très bien avec la justice. C’est hélas vrai, les gens malins ont beaucoup plus de chance de s’en tirer que les autres devant un tribunal. J’ai d’ailleurs toujours trouvé cela terriblement injuste, mais c’est ainsi (notre monde est cruel, dans l’ensemble).

Eolas:
Juste pour ceux qui lisent ce commentaire : non, un faux est une altération de la vérité dans un écrit ou tout autre support d'expression de la pensée qui a pour objet ou qui peut avoir pour effet d'établir la preuve d'un droit ou d'un fait ayant des conséquences juridiques. Peu importe qu'il soit ou non déclaré tel. Les divagations de l'auteur de ce commentaire reviennent à dire qu'un vol n'est un vol que quand il est déclaré tel par un tribunal, ou la conduite en état d'ivresse n'est constituée que si on souffle dans le ballon. Bref, le classique pas vu, pas pris.

- la « vérité » ! Je ne peux imaginer qu’un avocat, même débutant, y croie une seconde. Ou est la « vérité » dans le procès Colonna ? Où est la « vérité » quand la police rafle des manifestants au hasard et les amène devant le juge en prétendant qu’ils ont commis tel ou tel délit ? La vérité judiciaire n’est pas la vérité, nous le savons tous. Ou plutôt : la vérité de la justice, c’est son rôle de garant de l’ordre social. Peu importe que les gens qu’elle condamne aient ou n’aient pas commis les actes incriminés : il suffit qu’ils soient « capables du fait » pour que soit protégée la « société » (le capitalisme en fait...)

Eolas:
Ne parlez pas à la place des avocats, vous n'avez visiblement pas la moindre idée de ce que c'est.

- notre guide ne visait pas, à l’origine, à « se passer d’un avocat gratuit » (même s’il y en a certains dont il vaudrait mieux se passer), mais seulement à organiser la défense collective pour éviter de laisser le prévenu isolé face à la machine judiciaire. A l’origine il s’adressait aux manifestants et militants interpellés, mais il peut aussi concerner n’importe quel prévenu.

- dernier conseil : en garde à vue on se tait. Du début à la fin. C’est le B.A.BA quand on a affaire à la police. Réservez vos déclarations aux magistrats.

Eolas:
Se taire en garde à vue, c'est comme l'appel à la chasteté contre le SIDA. C'est le plus efficace mais c'est généralement au-dessus des forces humaines.

75. Le samedi 21 mars 2009 à 18:15 par dwarfpower

@PromotheeFeu

justement, lex lata, donc licéité de la source indéfinie et copieur devant mesuré le préjudice avant de s'adonner à la copie. c'était mon propos...

76. Le samedi 21 mars 2009 à 18:43 par Bonjour chez vous

@35 Luc Montesquieu Si la vitesse est limitée en ville à 5O km/h, ce n'est pas un caprice du législateur. Si Crétin circule en ville à 99 km/h, il risque de tuer un passant, un cycliste, ... Il prend alors un risque bien supérieur à une simple amende et quelques points ...

Eolas:
Une voiture qui roule à 49 km/h en ville peut parfaitement tuer un passant ou cycliste (la majorité des morts en ville, notamment à paris, ont lieu en dessous de la vitesse maximale autorisée… Le risque augmente avec la vitesse, terriblement. Mais la vigilance et la prudence s'impose quelle que soit la vitesse.

77. Le samedi 21 mars 2009 à 19:07 par zorro

>Eolas "C'est censé démontrer que produire des faux en justice n'est pas une bonne idée" Une foule de choses ne sont pas de bonnes idées, ce qui m'échappe dans l'affaire que vous citez c'est dans quelle mesure la condamnation de 1 an avec sursis est elle juste au regard des faits reprochés : vol de 60 euros. Si la justice sert à réparer le préjudice subi, je ne comprends pas l'intérêt d'un tel verdict.

Eolas:
C'est parce que la justice ne sert pas à réparer le préjudice (la preuve : la victime du vol ne s'est pas constituée partie civile). La justice pénale vise à sanctionner un comportement interdit par la loi, à prévenir la réitération de l'infraction et permettre le reclassement du condamné. Ici, le prévenu prétendu avoir compris la leçon, et produit un faux pour essayer de tromper la justice. Donc il n'a pas compris la leçon. D'où grosse peine au-dessus de la tête pour prévenir la réitération. Avec un sursis pour ne pas l'empêcher de se réinsérer.

78. Le samedi 21 mars 2009 à 19:16 par dante Timelos

Maitre, je vous est fait parvenir un commentaire, mais celui-ci n'est pas paru... Etant mis en cause dans votre billet, il me semble que j'ai le droit de dire quelque mot pour ma défense.

Eolas:
N'attendez pas de moi que je surveille le chenil de Troll Detector™ toutes les 5 minutes un jour de tournoi des 6 nations. Apprenez la patience et allez applaudir l'Irlande pour son Grand Slam.

79. Le samedi 21 mars 2009 à 19:54 par Bonjour chez vous

Je suis surpris par cette défense récurrente des téléchargements illégaux par plusieurs intervenants.

Pour motiver ces téléchargements, il ne suffit pas d’invoquer le caractère négligeable du préjudice ou la quasi-impunité de fait.

L’absence de risque de contravention pour un automobiliste qui oublie son clignotant, ne peut pas justifier la suppression de cette obligation dans le code de la route.

Les téléchargements illégaux posent le problème du financement des œuvres à terme.

Si vous téléchargez, c’est illégal et il vous appartient d’assumez vos actes, sans chercher des excuses bidons.

Eolas:
Non, télécharger n'est pas illégal (vous avez téléchargé cette page pour la lire). C'est télécharger une œuvre protégée sans l'autorisation des ayant droit qui l'est. Je chipote, mais la tendance actuelle de frapper du sceau de l'infamie le terme de téléchargement est préoccupant.

Quand à considérer qu’une loi est « illégale », cela me semble osé dans un état de droit.

Eolas:
Précisément non : c'est l'essence d'un État de droit que de pouvoir contester la loi en invoquant une norme supérieure qui protège le citoyen de l'atteinte de la loi : la Constitution et les droits fondamentaux qu'elle protège. C'est cela qui a permis aux noirs américains de contester les lois ségrégationnistes adoptée pourtant démocratiquement aux États-Unis.

80. Le samedi 21 mars 2009 à 20:05 par Marcel

Maître, si je partage votre opinion sur le fait qu'un avocat gratuit soit toujours bon à prendre pour sa défense, je ne trouve pas inutile pour autant d'apprendre à se défendre "seul". Pourquoi ? Parce que si certains avocats font très bien leur travail et avec beaucoup de conscience professionnelle, même vis-à-vis de leurs clients à l'aide juridictionnelle, d'autres traitent malheureusement nombre de dossiers largement par dessus la jambe, et il n'est pas inutile de savoir faire une partie de leur travail à leur place.

Combien d'avocats potassent réellement à fond le dossier de chacun de leurs clients et repèrent en temps et en heure les failles éventuelles du dossier ? Peut-être beaucoup mais certainement pas tous.

Eolas:
Tout ceux qui espèrent pouvoir s'acheter à manger.

81. Le samedi 21 mars 2009 à 20:09 par ff

Cet article est-il une allégorie, où il faudrait remplacer la production d'un faux par le préservatif et la Loi par la morale catholique ?

Eolas:
C'est effectivement une allégorie, mais elle vous a échappé. Remplacez le faux par un deltaplane, la loi par une citrouille verte de 10 mètres de large, le procureur par un arrosoir volant, et vous aurez une scène surréaliste.

82. Le samedi 21 mars 2009 à 20:33 par Personne

« (c'est là le but de la loi HADŒPI : faire cesser le sentiment d'impunité des téléchargeurs pirates). »

Vous n'avez apparemment pas compris grand chose à cette loi, elle n'est pas faites pour sanctionner les « pirates », mais le délit imaginaire de non sécurisation de son accès Internet, il suffit d'être accuser pour en être coupable, c'est très pratique puisque c'est l'ayant droit qui fourni la preuve. Mais sur son fonctionnement elle renforce au contraire le sentiment d'impunité pour ceux qui seraient effectivement coupable car elle assure une absence de sanction avant le 3ème repérage, peut-être même une immunité temporaire contre le délit de contrefaçon.

Soit elle sanctionne des innocents, soit elle pardonne des coupables, une loi parfaite.

Eolas:
Non, je n'ai rien compris à cette loi. Mais ce qui me console, c'est que vous non plus. La loi ne crée pas de délit de non sécurisation, c'est une faute civile.

83. Le samedi 21 mars 2009 à 20:50 par PrometheeFeu

@79: Les droits d'auteurs sont un monopole qui permet a l'auteur de vendre a un prix largement supérieur a un prix concurrentiel. Le préjudice est largement théorique. Il existe, mais il y a des gens qui téléchargent de la musique aujourd'hui, qui avant Internet se contentaient de ne jamais acheter de la musique. Quand on parle de ces gens, le préjudice qu'ils causent est de 0. Il y a aussi des gens qui continuent a acheter 1 ou 2 CDs par mois et qui maintenant en plus téléchargent. Ces gens aussi ne causent aucun préjudice. La reponse logique a ce problem est la license global. Ce n'est pas une utopie, c'est ce que toute les theories economiques proposent depuis longtemps. C'est la meme raison pour laquelle vous ne payez pas a l'unite pour la plage ou les rues en ville.

84. Le samedi 21 mars 2009 à 21:12 par Tigrou_bis

J'ai éclaté de rire sur la dernière phrase... Merci Maître !

Et je profite de ce commentaire, pour remercier Olive (#63) de sa mise au point !

85. Le samedi 21 mars 2009 à 21:16 par zerbinette

"il se dit étudiant en droit (très mauvaise idée : si vous êtes étudiant en droit, taisez-le)"
Evidemment ! Tout le monde sait que les études de droit sont suivies en majorité par les voyous pour apprendre la meilleure façon de contourner la loi ! ;-)

86. Le samedi 21 mars 2009 à 22:08 par Vince

@ Eolas

Si vous arrivez à me fournir légalement un documentaire que je cherche pour des raisons professionnelles, dont la propriété et les droits de diffusion appartiennent au service public télévisé, qu'ils ne peuvent eux-mêmes me fournir ni par leur propre intermédiaire, ni par le biais de l'INA, ni par le biais de quiconque... moi je veux bien prendre.

Ce n'est qu'un exemple parmi les milliers d'autres qui sont à citer. Ce n'est pas parce que 99% des oeuvres à disposition légalement ne sont pas téléchargées ou achetées qu'il faut pour autant ne pas continuer à diversifier. On me rétorque le biais de frais de copie unitaire. Je doute fortement toutefois qu'un documentaire de 2005 soit conservé sur pellicules. Donc certainement numérisé, à moins que les régies de FranceTélévisions en soient encore aux technologies de 1964. Donc les frais de "copie" autres que support physique sont négligeables. On me rétorque l'existence des médiathèques... Heu... si une médiathèque pouvait contenir l'ensemble des données multimédia... livres, vidéos, cd, ça se saurait, alors que pour l'exemple, plus de 3000 livres sont édités par jour dans le monde. (Qui voudra lire ça ? Justement... qu'en savez-vous ?) Avec Internet, on n'en est plus à être passif et se voir "proposer" du contenu selon les goûts et envies d'un bibliothécaire ou disquaire. On va le chercher ! C'était la parenthèse agacée... Je vous présente toutefois mes salutations, malgré les divergences de points de vue.

87. Le samedi 21 mars 2009 à 22:34 par Kristian

"@52. Le samedi 21 mars 2009 à 11:10, par Juris : Que pensez-vous de ce fameux adage en droit de la concurrence communautaire: Pas vu, pas pris... Pris, pendu!"

Au CPH, sur les congés payés, on a : "pas pris, pas dûs"...

88. Le samedi 21 mars 2009 à 23:01 par Lucas Clermont

En somme les conseilleurs ne sont pas les payeurs ; de là sans doute qu'il vaut mieux les payer pour mieux assurer sa défense.

Vous avez cette perle qui complète ou précise le "sentiment d'impunité", je cite cette phrase de votre source d'inspiration du jour : «un certificat non vérifiable n’est pas un faux. Pourquoi ?... les gens malins ont beaucoup plus de chance de s’en tirer que les autres devant un tribunal»

89. Le samedi 21 mars 2009 à 23:43 par Yves

A Adèle, en 60: Un acte illégal peut être légitime si on parle de moral. Un chauffard a écrasé ma petite soeur, il est légitime que je veuille lui en coller une, voire, lui faire la peau. Mais ce n'est pas légal.

Un acte légitime peut être illégal. Seulement, c'est de droit que se préoccupe un avocat, et la cour autour de lui. A tout le moins, ils essaient.

90. Le samedi 21 mars 2009 à 23:46 par Domble

Bonsoir, En lisant vos réponses au commentaire, on peut lire que dans votre histoires, l'accuse a écopé d'une sentence plus lourde pour avoir produit un faux devant le tribunal, je m'interroge un peu sur cette pratique qui me semble douteuse.

Je vole une voiture et je me fais prendre, arrive devant le tribunal, je me totalement irrespectueux de toutes règles instaurée dans celui-ci: j'insulte mon entourage, je fume, je suis en état d'ébriété et je sucre mon thé, je conçois que le juge me sanctionner pour l'ensemble de ces faits (et espérons pour moi que dans cette histoire il soit contre la peine de mort, sinon le thé sucre ne passera jamais), mais alourdira-t-il ma peine pour le vol de voiture? Si je roulais sans permis ce serais encore compréhensible car dans l'absolu ce fait est lie au vol de voiture mais pour un crime/délit non lie au délit de base, je ne vois pas ce qui permet a un juge d'alourdir la sentence.

Après n'aillant pas fait de droit, je peux très largement me fourvoyer, mais j'espère que vos lumières m'éclairerons sur un sujet que vous maitrisez

Eolas:
Voyez ma réponse au commentaire 77 ci-dessus.

91. Le samedi 21 mars 2009 à 23:52 par Patrice Lazareff

@Eolas: « Non, télécharger n'est pas illégal (vous avez téléchargé cette page pour la lire). C'est télécharger une œuvre protégée sans l'autorisation des ayant droit qui l'est. Je chipote, mais la tendance actuelle de frapper du sceau de l'infamie le terme de téléchargement est préoccupant. »

En effet, et je me permets de chipoter davantage. Le français souffre de n'utiliser qu'un seul mot - télécharger - pour deux actions distinctes, indépendantes et n'entraînant pas les mêmes effets juridiques.

Ainsi, télécharger, au sens de copier depuis internet vers votre ordinateur (download en anglais) peut être illégal suivant la licéité de la source, mais il est impossible d'en rapporter la preuve puisque vous pouvez très bien avoir enregistré sur une web-radio légale la chanson que l'on vous reproche d'avoir sur votre disque dur, c'est à dire avoir effectué vous-même une copie pour votre usage personnel, ce que permet l'exception pour copie privée.

Mais ce que la loi Internet et création tente d'empêcher n'est pas la copie mais la diffusion, ou mise à disposition, qui est le téléchargement dit montant (upload en anglais) qui, pour être légal, vous impose d'être titulaire d'un droit pour se faire, soit parce que vous êtes auteur ou ayant-droit, soit parce que vous êtes titulaire d'une licence vous y autorisant - c'est le principe de la licence globale.

Il est exact que la plupart des logiciels d'échange de fichiers rendent ces deux formes de téléchargement concomittantes, mais cela peut être modifié dans les préférences des logiciels, ce qui ruine d'avance tout effet de cette loi si elle est adoptée. Au final, cela ne servira qu'à faire disparaître les artistes français des échanges entre internautes, mais n'empêchera nullement les gens de continuer à obtenir des chansons gratuitement.

Pardon d'avoir été un peu long, et si vous lisez jusque ici, je serais heureux que vous précisiez cette autre de vos déclarations: « Se taire en garde à vue, c'est comme l'appel à la chasteté contre le SIDA. C'est le plus efficace mais c'est généralement au-dessus des forces humaines. » En quoi est-ce plus efficace ?

Merci et bon dimanche :)

92. Le dimanche 22 mars 2009 à 00:09 par Bruno Kant

@Eolas en 74... "Ne parlez pas à la place des avocats, vous n'avez visiblement pas la moindre idée de ce que c'est."

Comme des banquiers: globalement, très bien.

J'ai adoré les propos tenus par l'un des derniers qui est intervenu en faveur des 6 membres de l'Arche de Zoé. Quelques temps plus tard, leur tout nouveau site Internet disparaissait de la toile...

Ils militent encore: http://www.archedezoe.fr/

93. Le dimanche 22 mars 2009 à 00:39 par X.

Je connais (professionnellement), un avocat qui produisait régulièrement des faux en justice. Il n'a jamais fait l'objet de poursuites disciplinaires ou pénales et cela bien que le juge administratif ait détecté de nombreux faux et les a obligeamment transmis à la "maison d'en face" (les habitués de la maison savent que c'est une pratique qui se limite aux cas qui nous chatouillent bien fort)... en l'espèce fausses-factures EDF qui se voulaient justifiant d'une résidence continue en France depuis plus de 10 ans (je sais cela commence à dater) et cela même si les factures de 1994 étaient en euros et que le bâtiment dont il était question n'avait pas encore de permis de construire...

Après de longues années de pratiques de ce genre, "on a appris" que, dans la plupart des cas, les clients n'étaient pas au courant, enfin au parfum...

Le commissaire pas encore rapporteur se faisait alors une joie de préciser tout le bien qu'il pensait des faux et du risque pénal encouru lorsque les clients étaient présents (et cela même si après l'audience l'avocat parlait de confusion malheureuse de son secrétariat)...

Mais un jour il a produit le faux de trop. Celui qui laissait entendre que son client avait de la famille proche en France... et Leonid X se trouva donc beau-frère de Vladimir Y. à l'audience (instant magique mais je ne suis plus trop sur des prénoms...).

Sauf que les X. et les Y. avaient chacun une petite affaire d'import export de jeunes majeures russes qui souhaitaient passer 6 mois de stage en France pour y parfaire leur culture des us et coutumes nocturnes des français et que ces (mé)faits étaient en cours d'enquête par un juge d'instruction, chacun ayant dénoncé la petite entreprise d'en face en sus de quelques pruneaux échangés au pays... la police comptant principalement les points entre deux reconduites de ses dames au Pays.

L'avocat a depuis une retraite paisible dans un coin de Bretagne ou il vit reclus... loin de tout ce qui peut lui faire penser à la sainte et éternelle Russie...

Son associé, qui avait été à la même école (Framboisy au soleil III), continue ses pratiques (mais a sollicité une protection policière qui lui a été refusée: il n'a pas dit pourquoi)... il a souvent des clients d'origine corse alors on espère... tout comme le Procureur (c'est un membre éminent du conseil de l'ordre).

Mais vous trouverez dans un certain nombre de jugements la formule "Considérant que les documents versés au dossier, et dont l'origine et l'intégrité ne sont pas établis, ne sauraient par eux-mêmes et en l'état justifier de..."

94. Le dimanche 22 mars 2009 à 01:10 par Dyonis

A la suite de vos derniers articles tres instructifs, une question me vient a l'esprit:

Si un jour je me retrouve en CI, j'aurais un soucis.

Sachant que:
Je gagne trop bien ma vie pour avoir droit a un avocat gratuit
Je n'ai pas achete le livre permettant de se passer d'un avocat gratuit.
Je ne connais aucun avocat ni de pres (famille, amis...) ni de loin (coordonées d'un cabinet quelconque).
Comment faire pour etre aidé ? Serais-ce l'avocat de permanance qui me viendrait en aide moyennant payement à reception de facture ?

A moins que ce ne soit les policiers qui auraient la gentillesse de me preter l'annuaire avec lequel ils m'ont frappé toute la nuit??

Eolas:
Le droit à l'assistance gratuite d'un avocat de permanence est indépendant des conditions de ressources (vous croyez qu'on a le temps de les vérifier ?). Théoriquement, on peut vous demander de rembourser l'indemnité de l'avocat, soit 200 euros environs. C'est une affaire.

95. Le dimanche 22 mars 2009 à 01:49 par jean guimauve

quid de la jurisprudence Gachou ?

96. Le dimanche 22 mars 2009 à 01:54 par nuancière

Le téléchargeur pirate : “ De toutes façons, j'aurais pas acheté ce CD/DVD, et j'ai pas les moyens d'aller au ciné, alors ils perdent rien. Puis de toutes façons, ils sont déjà pété de thunes, eux. ”

Chacun y va de son petit couplet, voici le mien :

le téléchargeur de musique est aussi souvent un passionné, pas forcément jeune, ni certainement pauvre, et qui achète plein mais vraiment de disques... Il y a eu des études là dessus (et j'en connais quelques specimens). Il lui arrive aussi de télécharger des choses introuvables dans le commerce (ou les médiathèques!...), des groupes obscurcissimes comme je les appelle (souvent anciens mais pas seulement), et vers lequel tout fan, de rock en l'occurrence, vraiment passionné, fini par tendre un jour... C'est compliqué tout ça... c'est pourquoi ce ne devrait pas être la seule affaire des juristes, si fins soient-ils ;) Si vous saviez les sommes considérables que j'ai mises à trouver des pépites après avoir commencé à découvrir tel groupe de façon illégale... juste pour approfondir... et en parler autour de moi. Certains artistes restent ainsi dans les mémoires grâce à l'action de passionnés. Mais c'est un autre monde... une autre histoire... d'où les innombrables malentendus existants sur la question.

97. Le dimanche 22 mars 2009 à 01:56 par Barbecane

Ah encore ce fameux utilisateur d'internet appelé "pirate" lorsqu'il procède à un téléchargement illégal. Pour cela il lui aura fallu acquérir légalement: - de l'électricité - un ordinateur - une connexion haut débit - et à l'occasion des CD DVD vierges

Tout ça pour échanger des mails, des pdf et podcaster Jean Marc Sylvestre...

Nulle malice, aucune hypocrisie dans tout cela. Et voilà notre pirate accablé par la nécessité de se bâtir un sentiment d'impunité, de sorte qu'être mis ou/et se mettre en situation illégale soit conditionné par la probabilité de n'être pas puni.

Voilà tout le système de la propriété intellectuelle à repenser et nous devons nous satisfaire d'une loi inique. Un résidu législatif partant en quête d'IP... Alors que déjà le problème n'est plus là, les échanges sont fractionnés et distribués en multi réseaux. Seuls les utilisateurs les plus novices se feront choper. Comme tata Simone sur Emule mettant 2 mois à télécharger un épisode de Dexter qui s'avérera être un fake porno...

Elle n'est pas applicable qu'elle est obsolète depuis longtemps. Les ayant droits n'en seront jamais satisfaits et les législateurs se vanteront auprès des moins sachant, uniquement. (On compte déjà sur Jean Pierre Pernaut pour le SAV) Il n'y a depuis longtemps plus les moyens de la coercition. Ce n'est pas la loi qui changera quelque chose. A part pour quelques malchanceux qui serviront d'exemple et qui serviront de faire valoir à l'hypocrisie du législateur.

Définir le téléchargement illégal par le préjudice financier porté à un ayant droit est vain. Pas besoin d'alibi moral ou de sentiment d'impunité pour justifier un usage courant au prétexte qu'il est interdit. Pas besoin de tant. Vraiment pas.

La morale et l'impunité, bien sur. Celles qui conduisirent par exemple l'OMC à condamner l'Europe (au nom du droit international) pour avoir refusé d'importer le boeuf aux hormones américain? Ou encore celles qui appliquent le concept d'"abonnement" aux semences et en interdisent l'échange ou la réutilisation naturelle?

Je m'éloigne du sujet? Pas tant que cela. Rien qu'en écrivant cela je suis suspect de piratage... Ouille mon IP... Heureusement que j'ai pris la connexion de ma belle-mère...

98. Le dimanche 22 mars 2009 à 02:17 par Esurnir

@nuanciere: une autre justification moral mouais, ca reste illégal.

99. Le dimanche 22 mars 2009 à 08:31 par dwarfpower

@91

comme je l'ai écrit plus haut, la nécessité d'une source légale n'est pas une des conditions de l'exception copie privée. Elle n'est pas non plus une condition exigée par la jurisprudence. Vous allez donc vite en besogne lorsque vous affirmez télécharger, au sens de copier depuis internet vers votre ordinateur (download en anglais) peut être illégal suivant la licéité de la source.

Eolas:
Certes, mais la jurisprudence exigeant que la copie soit réalisée par le copiste lui-même et dans un cadre privé, toute mise à disposition du public via un réseau P2P ou autre est exclusif de l'exception de copie privée. Sauf spectaculaire revirement de jurisprudence,vous considérer comme acquis que le téléchargement d'œuvres protégées est illégal.

Par contre votre affirmation de la licéité de la capture de streaming d'une source légale, elle est problématique, parce que la plupart des sources légales de streaming protègent leurs flux et que le mécanisme de protection bénéficie de la même protection que l'oeuvre. La contourner constitue un acte assimilable à de contrefaçon.

Enfin, HADOPI ne modifie pas le droit en matière de protection des oeuvres: elle ne va pas changer quoique ce soit sur vos droits à télécharger ou à diffuser, elle va juste prévoir des procédures pour les agents assermentés pour qu'ils puissent déclencher des procédures aboutissant à des sanctions administratives. Le cadre d'action de ces agents n'est lui pas modifier, et ils peuvent constater tout aussi bien des actes de téléchargement (download) que des actes de diffusion ( upload ). M Olivennes avait parlé peu après la signature des accords de l'Elysée de technique de pot de miel, qui permettent de repérer les téléchargeurs(downloadeur). Cela va entrainer des années de procédure et finir en cassation pour savoir si une telle preuve est recevable, mais va sans doute être fait.

100. Le dimanche 22 mars 2009 à 10:07 par jijin

chez Olive, commentaire 63

les bibliothèques ont hélas et ce beaucoup trop souvent qu'un catalogue commun, voire banale...

Quasiment aucune micro-édition (les maculas quelqu'un en a vu ?), pas assez de bouquin rares et pointus sur des sujets avec peu de public, peu de livres d'anthologies, pas trop de livres polémiques (jamais vu walter benjamin dans un rayonnage histoire pour ma part)... ces institutions ne prennent pas de risques, ne vont pas fouiner, se restreignent à d'affreuses centrales d'achat !

Elles ne valent pas beaucoup mieux que les majors, dans le fond, si ce n'est pour le prix d'accès, certes...

101. Le dimanche 22 mars 2009 à 10:39 par Dante Timélos

Les divagations de l'auteur de ce commentaire reviennent à dire qu'un vol n'est un vol que quand il est déclaré tel par un tribunal, ou la conduite en état d'ivresse n'est constituée que si on souffle dans le ballon

C'est pourtant bien ce qui se passe. Seule une vision très idéologique des choses peut vous faire croire que "le droit" existerait en dehors des organes d'oppression (prison, police, justice) qui le font naître. Sans Etat et tribunal, pas de droit. Ce qui ne signifie pas "pas de règles", soit dit en passant. Le droit, c'est la norme dont l'Etat s'est arrogé le monopole.

Eolas:
Je vous assure que si on me vole mon vélo, je n'aurai pas besoin qu'un jugement vienne me l'indiquer pour réaliser que je vais devoir rentrer à pied. Et si on m'assassine, je n'attendrai pas l'arrêt criminel de la cour d'assises pour mourir. Tel est mon mépris pour l'État.

Autre chose: je plaide coupable pour l'énorme faute d'orthographe de mon commentaire N°78... Je ne l'ai vue qu'une fois à l'écran, c'est à dire trop tard!

102. Le dimanche 22 mars 2009 à 10:44 par ariane

Se taire en garde à vue, euh....

je suis étonnée (enfin, à moitié) de lire que Dante Timélos donne comme cpnseil : "en garde à vue on se tait. Du début à la fin. C’est le B.A.BA quand on a affaire à la police. Réservez vos déclarations aux magistrats"

Mais peut être encore plus étonnée de la réponse du maître des lieux : "Se taire en garde à vue, c'est comme l'appel à la chasteté contre le SIDA. C'est le plus efficace mais c'est généralement au-dessus des forces humaines. "

Je crois qu'effectivement, il y a quelques cas où, ma foi, autant se taire en garde à vue, voire ensuite en cas d'ouverture d'information devant le magistrat instructeur lors de l'interrogatoire de première comparution. Mais en tant qu'ex juge d'instruction, je dis que dans la plupart des cas, ce n'est VRAIMENT pas une bonne idée!!!!! et je suis vraiment sciée, Me Eolas, que vous présentiez le silence en garde à vue comme le plus efficace!

Si la personne se tait en garde à vue, ça veut dire pas de vérifications possibles de sa version... alors si par exemple vous êtes inncocent des faits pour lesquels vous êtes placé en GAV, imaginons ainsi qu'on vous accuse d'avoir tué quelqu'un à Lyon et qu'au moment des faits vous étiez en voyage d'affaires à Rennes avec votre patron et 4 collègues, je crois qu'il serait plus prudent de le préciser aux enquêteurs tout de suite, pour que cela puisse être vérifié au plus tôt. Si vous taisez cet élément là par contre, et que vous le sortez comme une fleur à 21h au magistrat instructeur devant lequel vous êtes présenté, ce dernier risque de se dire :
1) bizarre qu'il l'ait pas dit plus tôt quand même!
2) enfin, admettons que comme il le prétende il n'ait fait que suivre les conseils lus sur le blog de Me Eolas.... ça ne change rien au fait qu'il est 21h, que les enquêteurs sont chez eux, que je dois terminer cet interrogatoire de première comparution avant d'entendre ou de faire entendre ces témoins, donc... soit cet homme était effectivement à Rennes et n'a pas pu commettre les faits, mais ça l'audition des témoins sera déterminante pour le savoir et je peux pas la faire maintenant; soit il se fout de moi, tente un coup de bluff, et va se carapater à l'autre bout du monde si je le laisse repartir libre ou sous contrôle judiciaire ce soir. La décision qui risque fort d'être prise sur le moment est une demande de détention provisoire...
et même si on prend le cas d'une personne qui aurait bien commis les faits pour lesquels elle est placée en garde à vue, elle peut souvent avoir intérêt à parler en garde à vue, il y a bien des cas où cela a évité une détention provisoire! ("oui, c'est vrai, j'ai violé ma fille quand elle avait 10 ans, maintenant elle a en 20, elle habite à 600km de chez moi, si vous saviez comme je regrette, je n'ai jamais recommencé une telle chose, je suis atteint d'une grave maladie, je vis seul avec ma femme à la campagne, aucun enfant ne vient jamais à la maison" => tout ça, vaut peut être mieux le dire tout de suite en GAV, non? )

Eolas:
Je ferai un jour un billet sur la question. Notez que je dis en garde à vue. Devant un magistrat du siège, je conseille au contraire d'être volubile. Tout simplement parce qu'un magistrat du siège n'a pas le droit d'adresser la parole à quiconque sans qu'il n'ait eu droit à un avocat ayant eu accès au dossier. En attendant, si vous parlez anglais, voici des éléments de réponse.

103. Le dimanche 22 mars 2009 à 10:47 par Soe

Tiens, mon navigateur s'étant arrêt sur ce passage :

"La CI est une justice d'abattage, ce qui signifie que les dossiers sont légers voire vides, bâtis à la hâte avec quelques PV policiers, et que les juges n'ont pas les moyens de vérifier quoi que ce soit. Il ne s'agit alors que de retourner un peu à l'avantage des personnes poursuivies cet état de fait déplorable."

J'ai cru que vous parliez du droit des étrangers et de l'asile ... :o)

Eolas:
Pour le droit d'asile, certainement pas, sauf procédure en rétention. Pour le droit des étrangers, c'est plus pervers. Le droit pénal, quand même un peu protecteur, est écarté. La situation des étrangers reconduits est pire que celle des prévenus en CI hormis bien sûr la peine encourue.

applicable mutatis mutandis ? ô, bien sûr que non, n'est-ce pas maître ?

104. Le dimanche 22 mars 2009 à 11:15 par Leslie/Elsie

A Monsieur Dante Timelos :

Je ne suis pas juriste du tout, mais si demain on me vole mon porte monnaie, je n'attendrai pas qu'on le dise pour estimer qu'il ya eu vol. La seule chose qui m'incombe étant de le prouver.

Par ailleurs le droit n'est pas un organe d'opression, à la base. Il pose justement les limites de ce qui est permis dans le cadre de la société afin que chacun puisse y vivre à peu près libre et en paix. C'est le droit qui, justement, empêche que le simple ordre d'un monarque absolu vous envoie à vie croupir dans une geôle.

Votre discours me semble par trop entaché d'idéologie, justement. Car il passe sous silence que, tous les jours, des gens commettent des délits ou des crimes qui portent atteinte aux droits de leur semblables, et que la première fonction du droit est justement de contenir ces dérives. Vous tentez de faire croire que nous vivons encore dans une situation semblable à celle des pauvres paysans sous Louis XIV. Que tous les gens qui comparaissent sont des victimes. Ce qui est faux.

Je ne nie pas l'existence d'abus. Je ne dis même pas que la loi soit toujours juste car elle est une création humaine et la perfection n'est pas humaine. mais l'absence de loi ne produit, en général, que du mal. Il est d'ailleurs significatif qu'aucune société humaine, même la plus isolée et la plus proche de la nature, ne se soit passée de règles pour vivre ensemble. S'il ya des choses à perfectionner, il est du devoir du législateur de se retrousser ses manches. Et le citoyen, effectivement, doit garder les yeux ouverts afin de ne pas élire des pignoufs qui mettront ses droits en danger.

Je vous accorde que le citoyen ne fait pas toujours bien son boulot, que le législateur n'est pas toujours très malin. Mais c'est le moins pire qu'on ait trouvé. Pas le meilleur : le moins pire. Car quoique vous fassiez, vous ne pourrez pas empêcher l'existence des gens malintentionnés.

Enfin je trouve qu'inciter les gens à mentir : oui, je dis bien MENTIR, car je n'irai pas chercher une nuance qui me parait être de l'ordre de la sodomie de diptère pour décrire la chose, c'est leur faire grand tort : Comment ensuite la Cour les croira t'elle ? Si seuls les plus malins s'en tirent à ce jeu dangereux, est-ce à dire que les innocents affligés de bêtise devront payer plein pot à cause de ce genre de bon conseil ?

Vous ne cessez de parler de l'insjustice sociale, monsieur, mais semblez fort peu soucieux du type d'injustices que vos conseils peu malins, justement, pourraient provoquer... Malheur aux simples d'esprit : la prison sera pour eux...

105. Le dimanche 22 mars 2009 à 11:39 par Patrice Lazareff

@98 DwarfPower

Pour la licéité de la source, je faisais référence au même arrêt de la Cour de cassation que vous dans votre commentaire précédent, que j'ai sans doute mal interprété - tant mieux.

Je ne conteste pas la possibilité de la contrefaçon au sens strict mais j'affirme qu'il est impossible d'en rapporter la preuve. Car après tout, un ordinateur est aujourd'hui presque le seul outil utilisé comme magnétophone dans les plus grands studios d'enregistrement, fini les bandes magnétiques, et le fichier MP3 que l'on me reprocherait d'avoir sur mon disque dur pourrait tout autant provenir d'une capture de la radio hertzienne, quelle différence avec Deezer ou Dailymotion ?

Etant un être humain, c'est à dire un dispositif non pas numérique, ni même électrique mais biologique, je peux enregistrer ce que j'entends et filmer ce que je vois - c'est ce que les industries culturelles appellent «la faille analogique».

Le "pot de miel" est en effet la seule possibilité de surveiller une activité de download, car à force de menaces les flux entre internautes étant entièrement chiffrés les FAI ne peuvent savoir ce qui est en train d'être échangé via leurs tuyaux rendus opaque.

Question: quelles sont les conditions de la légalité de cette pratique du pot de miel ? (je pense aux protections qui encadrent le constat d'adultère)

À vous lire

106. Le dimanche 22 mars 2009 à 12:09 par dwarfpower

@104 patrice Lazareff

En matière de contrefaçon, le doute profite à l'accusation, vous êtes présumé contrefaisant sauf si vous apportez la preuve que la copie dont vous disposer est effectivement légale. La jurisprudence va ussi dans ce sens.

Pour le côté légal du pot de miel, j'avais cru comprendre que les parties ne sont pas tenues par l'obligation de loyauté qui ne s'impose qu'à l'autorité public. Mais je n'ai jamais lu d'avis pertinent sur le sujet. Mais dans un cadre ou le constat d'un agent assermenté, peut on dire qu'il s'agit d'une partie et non de l'autorite publique ? Sans compté que dans le cadre du pot de miel on peut soutenir que la source est legale car autorise, a moins de considerer que l'agent a commis une contrefaçon.... difficile sujet

107. Le dimanche 22 mars 2009 à 12:25 par adrien

@Eolas sous 78 Oui vous avez bien raison, il faut chaudement féliciter les guerriers de la verte Erin... D'autant plus qu'aller battre le Pays de Galles à Cardiff alors qu'ils étaient encore en lice pour la victoire finale (certes au goal average, ce qui ne se fait que depuis peu) sachant que deux victoires de rang, ils ne l'ont pas fait depuis 30 ans, mais surtout, qu'ils visaient encore la triple couronne.

Une pensée cependant pour les fois irlandais, qui quatre jours à peine après la Saint Patrick doivent remettre ça...

108. Le dimanche 22 mars 2009 à 12:43 par Bruno Kant

@Leslie/Elsie en 103 "la première fonction du droit est justement de contenir ces dérives."

Vous répondre nécessiterait un développement un peu plus long qu'un article ou un petit commentaire dans un blog. Il existe déjà des ouvrages qui ont dépeint de quelle manière peut s'exercer le pouvoir. En agissant sur ceux qui ont été pris la main dans le sac, ceux qui passent au travers des mailles du filet (et ils sont à mon avis nombreux) perçoivent malgré tout un message qui leurs fait entendre quelle pourrait être la peine encourue pour telle ou telle faute, crime ou délit. Ce n'est donc pas en punissant toutes les dérives individuelles que vous contenez le peuple dans son ensemble.

"je n'attendrai pas qu'on le dise pour estimer qu'il ya eu vol. La seule chose qui m'incombe étant de le prouver."

Dans un précédent billet, un juge répondait que seul l'Etat (le magistrat qui siègera) pourra définitivement juger et établir qu'il y a eu vol. Que ferez vous si les éléments de preuve que vous avancerez seront insuffisants à le convaincre?

Supposons que vous portiez plainte contre Y (et non X, un inconnu) pour le vol de votre portefeuille. Y peut très bien répondre qu'il a trouvé votre portefeuille dans le caniveau et qu'il était vide. Si Y est "malin", il répondra à la police qu'il ne comprend pas le sens de leur interrogatoire (et pour une broutille, la police n'insistera pas). Selon ce que vous même et ce Y affirmez, il y a de très fortes chances que votre plainte soit classée sans suite, comme des millions d'autres par an. Y bénéficiera de la présomption d'innocence et vous même aurez beau dire malgré tout qu'il y a eu vol, vous vous la mettrez sur l'oreille.

109. Le dimanche 22 mars 2009 à 12:49 par Alicanthe

"Se taire en garde à vue, c'est comme l'appel à la chasteté contre le SIDA. C'est le plus efficace mais c'est généralement au-dessus des forces humaines. "

Et pour éviter les coups de soleil, il vaut mieux vivre dans sa cave, pour éviter la grippe il vaut mieux vivre au fond d'un bois, pour éviter les accidents de voiture, il vaut mieux marcher, etc.

Il y a des gens, qui pour des raisons morales ou religieuses, accordent du prix à la chasteté ou à la fidélité, et ces gens-là sont relativement à l'abri du Sida, nul ne le conteste. Il y a aussi des gens qui souhaitent avoir une activité sexuelle plus variée ou plus libre. Ils en ont le droit et ils ont les moyens d'avoir une protection relativement satisfaisante grâce au préservatif.

Sauf s'il se trouve un Pape pour leur raconter que cet objet aggrave le problème, causant très probablement quelques milliers de morts par ses paroles.

Sur un autre sujet, je rappelle aux lecteurs que pour qu'il y ait vol, d'après ce qu'écrit Eolas habituellement, il faut qu'il y ait condamnation définitive de l'auteur, qui n'est pas coupable jusque là.

110. Le dimanche 22 mars 2009 à 13:51 par Leslie/Elsie

A Bruno Kant : je maintiens ce que je dis. Si on me vole mon porte monnaie, je sais qu'il a été volé. La police va constater le vol, et si on arrive devant la justice, c'est qu'on présuppose qu'il ya bien eu vol. Sauf à m'accuser de mensonge (Ce qui se prouve aussi). Bien sur qu'on va me demander des informations. Mais pour avoir eu une fois l'occasion, hélas, de déposer plainte contre X, il me semble qu'on ne remet pas en cause le fait d'avoir été victime d'un acte délictueux (Bon, là, le type, on l'a jamais retrouvé).

Maintenant, savoir si l'accusé est bien le coupable est une autre paire de manche... La seule chose que pourra dire la justice, ce n'est pas qu'il n'y a pas eu vol mais que la personne accusée n'est pas le voleur. Grosse nuance.

111. Le dimanche 22 mars 2009 à 14:47 par Léopold

Je ne suis pas avocat et je n'ai pas lu Beccaria, mais il me semble que c'est psychologiquement faux. Je croyais aussi qu'on ne pouvait pas commettre ce qu'on savait être mal, et qu'il était donc nécessaire d'inventer une justification ; mais certains actes ne le permettent tout simplement pas (je pense par exemple à des affaires d'industriels cachant que leur production est empoisonnée, ou empoisonnant celle d'un client). Bon, je peux pondre une justification implausible, du genre "Ça va tuer plein de gens, mais avec l'argent que ça me rapportera je pourrai en sauver encore plus", mais si on les accepte, la théorie devient infalsifiable.

Eolas:
Si vous parliez d'une délinquance qui existe en masse, plutôt que sur des rares affaires ponctuelles, ce serait plus facile de raisonner. Un raisonnement général ne se bâtit pas sur l'exception.

Pour le côté "certitude de l'impunité", c'est un peu plus incertain. Apparemment, le test proposé ("Cependant, aucun d'entre eux, quelles que soient les bonnes raisons qu'il se donne, ne passerait à l'acte s'il avait la certitude d'être sanctionné.") est "si les gens croient vraiment à leurs justifications, alors ils passent à l'acte même s'ils sont sûrs de se faire prendre". C'est très douteux. Sous une dictature, il y a vraiment d'excellentes raisons, disons, d'imprimer un journal d'opposition clandestin, et on peut difficilement douter de la sincérité de ceux qui le pensent ; pourtant, très peu de gens le feront s'ils sont certains d'être arrêtés et condamnés. Il me semble évident qu'il y a une simple évalution de risque, comme pour n'importe quelle activité, et que la certitude ou l'illégalité n'ont pas a priori de raison d'intervenir.

Eolas:
Non, même sous une dictature, ceux qui publient un journal d'opposition espèrent ne pas être pris. ne serait-ce que pour sortir l'édition suivante, et accessoirement, ne pas se retrouver avec des cathodes sur les parties génitales.

D'après mon expérience personnelle, les "pirates" croient sincèrement à leurs justifications ; les vendeurs de cannabis se considèrent vraiment comme des dealers, mais ne trouvent pas ça plus mal que d'être buraliste ; et personne ne blâmerait un voleur qui n'a pas de quoi manger.

Eolas:
D'après la mienne, oui, les pirates sont sincères, ce qui ne les empêche pas d'être hors la loi, les vendeurs de cannabis évitent de se poser la question, et si, on blâme le voleur qui n'a pas de quoi manger.

112. Le dimanche 22 mars 2009 à 14:56 par Sylvain

Bonjour,

"La CI est une justice d'abattage, ce qui signifie que les dossiers sont légers voire vides, bâtis à la hâte avec quelques PV policiers". Bah, voyons....

"Quant à la "vérité", personne n'y croit et surtout pas la police et le procureur qui bâtissent l'accusation. Alors...". (re) bah, voyons....

IN SYLVAM NON LIGNA FERAS INSANIUS !

113. Le dimanche 22 mars 2009 à 15:24 par RG

@110 Leslie/Elsie

Si on me vole mon porte monnaie, je sais qu'il a été volé. La police va constater le vol,

Sauf témoins ou preuve matérielle (vidéo) elle ne pourra que constater vos dires.

114. Le dimanche 22 mars 2009 à 16:11 par Nemesis

J'assure Dante Timelos de toute ma sympathie. Et je le remercie pour cette magnifique formule : "Le droit, c'est la norme dont l'Etat s'est arrogé le monopole." Je crains qu'on ne l'enseigne pas à l'ENM.

Eolas:
Non, Dieu merci. Les étudiants de l'ENM ont fait des études de droit ou Sciences-Po, ils savent ce que c'est que le droit et n'ont pas besoin de ce genre de lieu commun.

Car enfin est ce qu'ils "passaient à l'acte" les Justes qui cachaient des enfants juifs il y a si peu de temps ?

Eolas:

Point Godwin.

Est ce qu'ils "passent à l'acte" les courageux qui protestent quand la PAF bouscule (respectueux autant que prudent euphémisme) quelques africains reconduits au Sud ? Et quelle norme énonce-t-il le Juge qui condamne les passagers qui protestent ?

Que l'on me réponde à ces situations là d'abord. Elle est où la "vérité", il est où le "passage à l'acte", et quid de sa "justification morale"?

Tout ce qu'écrit Eolas postule que la loi est juste. C'est loin d'être le cas. Etre lâche serait-il un devoir ?

115. Le dimanche 22 mars 2009 à 16:23 par Couplus

Comme le disait un maître à penser de mon alma mater, c'est la capacité de pression par menace de sanction qui crée la norme et différencie celle-ci du voeu pieu.

Cela étant, reconnaissons une certaine logique au discours de Dante : une infraction n'existe, en droit pénal, que si elle est reconnue en tant que telle par un tribunal. A défaut, elle n'est qu'un désordre sociétaire, ceci en raison du principe de légalité du droit pénal et de la présomption d'innocence.

116. Le dimanche 22 mars 2009 à 16:29 par Valéry

Un commentaire hors sujet je le crains.

À la lecture de l'article suivant je me demandais dans quelles conditions les policiers sont autorisés à pénétrer au domicile des particuliers ?

http://www.liberation.fr/societe/0101557103-sommet-de-l-otan-des-strasbourgeois-sommes-de-retirer-le-drapeau-de-la-paix

Aux Etats-Unis on a une petite idée mais en France on n'en sais pas grand chose.

Sauriez-vous nous éclairer à ce propos un de ces jours s'il vous plaît ?

117. Le dimanche 22 mars 2009 à 16:45 par Bruno Kant

@Leslie/Elsie "Si on me vole mon porte monnaie, je sais qu'il a été volé."

D'abord remarquons bien que nous discutons d'une situation fictive.

"La police va constater le vol"

La police va simplement constater que vous arrivez sans porte monnaie et va - peut être - enregistrer vos déclarations.

118. Le dimanche 22 mars 2009 à 17:08 par Fantomette

Framboisy c'est chez moi.

119. Le dimanche 22 mars 2009 à 17:42 par Le Chevalier Bayard

@ Léopold

Pardonnez-moi, mais j'ai du mal à vous suivre.

Sachez, en tout cas, que le passage à l'acte c'est transgresser un interdit...Bécarria ou pas.

Très prosaïquement, c'est aussi l'impossibilité de refouler une pulsion irrésistible... certitude ou pas certitude d'une impunité.

Le sujet pathologique, paradoxalement, a souvent, pleinement conscience de "braver" un interdit sanctionné, précisément, par une punition !

La transgression, c'est alors aussi la jouissance de l'interdit sanctionné !

Et, le concept de "certitude de l'impunité", paraît alors, en effet, non pertinent ou, à tout le moins, discutable à suivre, je dirai, une "doxa psychologisante".

120. Le dimanche 22 mars 2009 à 18:48 par Bruno Kant

@Le Chevalier Bayard, en 119 "Sachez, en tout cas, que le passage à l'acte c'est transgresser un interdit...Bécarria ou pas."

De quels interdits, transgressions et pathologies parlez vous? C'est qui Beccaria, qu'est-ce qu'il démontre?

Moscou n'arrêtera pas la mère d'Elise

121. Le dimanche 22 mars 2009 à 19:52 par CaO2

Super vidéo, très instructive ET distrayante. Est-ce que c'est valable en france également ?

Eolas:
Sous certaines réserves, oui. Voici, tout aussi intéressante, la "réponse" du policier. Qui se résume à : "Il a raison". Au passage, je salue les universités américaines qui mettent en ligne les cours de leurs profs. L'internet, ça doit être ça.

122. Le dimanche 22 mars 2009 à 19:55 par Moi

Commentaire 23 :"Etant scientifique et par conséquent positiviste..."

La science est neutre de jugement de valeur, autrement dit elle se contente de présenter les faits comme ils sont. Dire que la loi est nécessairement juste, ou il y a pas d'autre source de légitimité que le droit positif, est un jugement de valeur ! Dire que l'on doit respecter même les mauvaises lois est un autre jugement de valeur.

Le positivisme juridique dans ce sens là (et non dans le sens restreint de l'étude du droit tel qu'il est) n'est pas une position scientifique (ce qui ne signifie pas nécessairement qu'il n'est pas valable mais simplement que c'est un problème normatif et non descriptif).

123. Le dimanche 22 mars 2009 à 20:12 par Le Chevalier Bayard

@ 120 Bruno Kant, merci de me rappeler que j'ai commis un " délit d'orthographe" au nom de Beccaria.

C'est la raison pour laquelle, je suppose, vous me soumettez, à la peine de son oeuvre majeure : " Des délits et des peines" ?

Très modestement, et à suivre Michel Foucault dans "Surveiller et punir -Naissance de la prison" (Gallimard 1975 collection tel), je dirai que Beccaria et je cite est : "Un grand réformateur faisant appel aux thèmes chers aux Lumières du 18ème siècle - l'égalité, l'humanité,la clémence, l'autonomie et l'universalité - et aux principes utilitaristes de la prévention et de la correction, pour développer une forme de châtiment plus mesurée et plus disciplinée. Becccaria joue un rôle crucial dans cette histoire : il l'est un des théoriciens des réformes des Lumières les plus cités - Le premier des (grands "réformateurs") qui annonça toutes "les règles qui autorisent, mieux, qui expriment la "douceur" comme une économie calculée du pouvoir de punir." (p.77, 84 et 103).

De ce point de vue on peut dire que l'ouvrage de Beccaria forme la clef de voûte de la généalogie de la disciplne de Michel Foucault dans son livre qu'il a su parfaitement saisir.

Enfin, pour conclure à la manière de Beccaria le message principal de l'ouvrage peut être celui-ci : "Comme les âmes s'adoucissent sous l'action de la société, la sensibilité croît. Et dans le processus, la sévérité des châtiments devraient diminuer, si la relation entre l'objet et la sensation se doit de rester constante" (chapitre 47).

Eolas:
Ajoutons que l'article 8 de la déclaration des droits de l'homme et du citoyen est un hommage à Beccaria.

124. Le dimanche 22 mars 2009 à 20:16 par proc hure

A Valery en 116

si vous ouvrez la porte aux policiers et les invitez à entrer, ils peuvent bien entendu le faire.

Ils peuvent s'asseoir après vous en avoir courtoisement demandé la permission et engager la conversation.

C'est comme ça que ça se passe dans l'immense majorité des cas (calmement, sauf chez les tempéraments soupe-au-lait, j'en devine quelqu'uns à la lecture des messages).

Si vous leur refusez l'entrée et hors situations particulières (appel de quelqu'un se trouvant chez vous, par exemple votre fils que vous avez pendu par les oreilles, crime flagrant) ils ne peuvent s'imposer à l'intérieur, sauf s'ils sont munis, entre autres, d'un ordre d'arrestation émanant du procureur ou d'un juge.

Le droit c'est une question de bon sens, mais j'en vois qui cherchent à tout compliquer.

C'est pas bien.

125. Le dimanche 22 mars 2009 à 20:31 par john

J'ai ri en lisant cet article. Mais plus particulièrement pendant la première partie nous expliquant le mécanisme psychologique du passage à l'acte et sa justification "morale"... Sachant que les textes de loi sont majoritairement pris en fonction de considérations morales qui peuvent être contradictoires d'un pays à l'autre, quel est donc celui qui est le plus "moral" pour s'appliquer?

Et mélanger le projet HADOPI dans le lot dont le but serait de faire cesser le sentiment d'impunité des pirantins, hum... c'est plus que négliger tout le débat qui existe à l'heure actuelle sur net et refuser d'admettre les évolutions de notre société. Il ne s'agit plus de morale mais structure de la société.

126. Le dimanche 22 mars 2009 à 20:41 par benoil

Cher Maître,

Au fil de vos billets, je ressens toujours la même impression : celle de la précision. Vous êtes d'une précision, d'un pointillisme merveilleusement efficaces. Exemple de cette précision; le commentaire que vous formulez en 79, sur la "définition" du téléchargement. Tout, avec vous, n'est que nuances et subtilité. Je poursuis un but depuis maintenant 3 ans, celui de devenir avocat. Mais je désespère de ne pas y parvenir si je n'acquiers pas votre finesse spirituelle et juridique. Cher Maître, consolez-moi !

Eolas:
Allons, allons… (tapote votre épaule doucement)

Avec tout mon respect.

127. Le dimanche 22 mars 2009 à 20:41 par dwarfpower

@ Eolas en réponse au 99

Bien sûr que la mise à disposition ne saurait être couverte par les exceptions pour copie. Par Contre je ne vois pas comment vous concluez à partir de cela qu'un téléchargement ( un download en français, à par les député je n'ai jamais entendu quelqu'un appeler un upload un téléchargement ... ) non autorisé est forcement illégal. Dès lors qu'il ne serait pas lié à une mise à disposition le cas serait à étudier ce qui n'a jamais été fait par une cour d'appel en france à ma connaissance. ( jamais eu d'évaluation du test en trois étapes non plus pour le compte ). Ca finira par arriver, mais ca va prendre du temps.

128. Le dimanche 22 mars 2009 à 20:45 par BearImprint

"Sentiment d'impunité" "Passage à l'acte" Je pensais à un article sur nos hommes politiques du coup. Et si les hommes de lois se doivent d'être irréprochables, les hommes politiques devraient être l'exemple à suivre. Les actes des autres ne doivent (devraient) pas dicter les nôtres mais quand on vous sert régulièrement du: "Fait ce que je dis, pas ce que je fait" cela à de quoi se faire demander si nos dirigeants ne nous prendraient pas pour des nouilles. Et puis, j'ai un peu de mal à me voir en dangereux criminel quand je roule à 134 km/h sur l'autoroute. Sentiment d'impunité de ne pas le faire devant un radar, surement.

129. Le dimanche 22 mars 2009 à 21:03 par heros

"Eolas: Le droit à l'assistance gratuite d'un avocat de permanence est indépendant des conditions de ressources (vous croyez qu'on a le temps de les vérifier ?). Théoriquement, on peut vous demander de rembourser l'indemnité de l'avocat, soit 200 euros environs. C'est une affaire."

Oui vous avez le droit a choisir votre propre avocat mais pour vous aider dans ce choix la police en garde a vue ne vous fournira aucune liste d'avocat...ni d'acces a un telephone ou a internet pour en choisir un.

Avant la comparution immediate le substitut du procureur de la Republique doit avertir l avocat de votre choix mais ne vous fournira également aucune liste ni d'acces a un telephone ou a internet pour en choisir un.

Donc dans ce cas il faut mieux prendre l avocat de permanence pour parler du beau temps pendant la garde a vue et ne jamais prendre l avocat de permanence pour la comparution immediate mais demander le renvoi de l audience pour que vous puissez prendre conseil:..

Eolas:
Si vous donnez le nom de votre avocat, la police le cherchera dans l'annuaire et contactera l'ordre des avocats dans les villes où, comme Paris, il y a une permanence 24/24. Si vous avez un vrai pénaliste, il vous fournira un numéro de téléphone mobile ou il est joignable 7j/7. Par contre, n'attendez pas de lui qu'il soit disponible à 3h00 du matin.

130. Le dimanche 22 mars 2009 à 21:21 par boby

Il faut toujours garder le silence vis a vis de la police ou du procureur et juge d'instruction.

Comme le dis la video on n a jamais vu qq s en sortir en temoignant de sa bonne foi. Il faut essayer de penser a quel temoin vous allez faire citer a votre proces ..

Et si la police tape a la porte, il ne faut pas lui ouvrir. S'il defonce illegalement votre porte au moins il ne pourront pas pretendre que vous leur avez ouvert ;) et puis l etat sera oblige de vous rembourser les degats..et oui les serruriers cela existent..

Pacte international relatif aux droits civils et politiques

14-3. Toute personne accusée d'une infraction pénale a droit, en pleine égalité, au moins aux garanties suivantes:

g) A ne pas être forcée de témoigner contre elle-même ou de s'avouer coupable.

131. Le dimanche 22 mars 2009 à 21:59 par Bruno Kant

@Le Chevalier Bayard en 123

Dans ma réponse en 120, vous deviez plutôt remarquer que je balayais l'oeuvre de Beccaria ainsi que les textes et pensées qui ont pu s'en inspirer. Lisez un peu de Schoppenhauer et d'Hegel, ne vous contentez pas simplement de citer du Foucault dont les textes ne prouvent rien non plus... ce ne sont jamais que des philosophies auxquelles on adhère ou qu'on réfute. Je pourrais vous présenter des personnes qui vont balayer Foucault et marteler du Tocqueville... alors Bécarria :-)

Dans le lien et la dépêche vers laquelle je renvoyais un peu plus haut, on nous fait savoir que la Russie pourrait ne pas avoir l'intention de reconnaitre que Elise, récemment "disparue", a été enlevée très brutalement à son père. Ce dernier allègue qu'il a été violemment frappé mais il est peut être tout simplement tombé de son vélo? Nous aurions là, peut être, une certaine variante d'expression du "déni" que Gascogne évoquait un peu plus haut.

Pourquoi les russes devraient-ils croire à la version du père, est-il seulement crédible? Lui même est manifestement en infraction et indésirable sur leur propre territoire, la Russie. Vous pourrez citez du Beccaria ainsi que du Foucault orthographié comme vous le souhaiterez. Je ne suis pas certain que la Russie daignera vous entendre. :-)

@boby en 130 "Il faut toujours garder le silence vis a vis de la police ou du procureur et juge d'instruction."

Il est aussi possible que ce père en ait trop dit à la presse? Le silence est d'or...

132. Le dimanche 22 mars 2009 à 22:22 par Bianca

Merci Maître, il faudrait vous cloner. On aurait autant d'Eolas que de justiciables, la justice tournerait sûrement mieux. Imaginons que je fabrique une fausse facture EDF pour justifier du domicile de quelqu'un, parce que je n'ai pas de justificatif (abonnement à un autre nom, par exemple), mais que le domicile de ce quelqu'un est réellement à l'adresse indiquée sur la fausse facture EDF. Est-ce répréhensible?

Eolas:
Il y a altération de la vérité, dans un acte destiné à faire la preuve d'un droit. Cependant, on peut discuter du préjudice que peut causer le faux, mais la jurisprudence est très peu exigeante sur la question.

133. Le dimanche 22 mars 2009 à 22:39 par Alicante

Vous pouvez être condamnée pour vrai et usage de vrai.

134. Le dimanche 22 mars 2009 à 23:19 par Bruno Kant

@Eolas en 132: "Il y a altération de la vérité, dans un acte destiné à faire la preuve d'un droit."

Il y a peu, je contestais encore une très ancienne décision de justice dans laquelle apparait, entre autres, que l'une de mes filles serait née en 2001. Or, selon l'Etat civil, son carnet de santé ainsi que divers autres fichiers et registres, elle est née en 2003. Que penser de cette décision de justice, du papier qui constitue, pour certains, la preuve de divers droits?

Eolas:
Qu'elle peut être rectifiée par une requête en erreur matérielle.

135. Le dimanche 22 mars 2009 à 23:41 par Le Chevalier Bayard

@ 131 Bruno Kant

Vous êtes taquin Bruno ! Ha! La dialectique éristique du ténèbreux "Schopi"... Ma foi, je la préfère pour des raisons d'efficacité à l'éthique de la discussion kantienne, même si Kant reste à mes yeux le plus grand quant à Hegel c'est à son idéalisme historique conceptuel du droit que je pense! Au fait,vous ai-je dis que Beccaria et Foucault n'étaient pas ma tasse de thé contrairement à Tocqueville ! Je vous rejoins on a les lectures que l'on mérite !

A mon tour,de la même manière, pourquoi pas à ce jeu, vous dire de ne pas simplement citer des auteurs et de balayer, comme vous dites, Habermas, Weber, Gény, Jhering, Kelsen, Delmas-Marty, Dworkin, Arendt, Rawls, Troper, Schmit, Aron et, un petit dernier pour la route, Nietzsche pour sa généalogie de la morale etc, etc, etc,...

Le lien avec Beccaria et les Russes est trop subtil pour moi !

Car, voyez-vous, je préfère le pédant au cuistre. Le cuistre est un homme qui exhibe ses connaissances avec assurance. L'assurance révèle en général des connaissance mal assurées; je n'ai jamais rencontré un vrai savant qui soit un cuistre. Le savant est modeste, le cuistre est cramoisi, bombé, homard.

Le pédant est pardonnable, lui, il exhibe ses connaissances avec passion !

136. Le lundi 23 mars 2009 à 00:18 par v_atekor

(c'est là le but de la loi HADŒPI : faire cesser le sentiment d'impunité des téléchargeurs pirates).

Certe, même chose que pour les radars automatiques. Ca c'est la théorie. Sauf qu'en pratique ça va être une autre affaire. Si on était à peu près sûr que les automobilistes de 2000 utiliseraient 5 ans après des automobiles pour circuler sur des routes allant d'un lieu à un autre, les choses risquent d'être moins systématiques avec des comportements mouvants, ou l'utilisateur aura un sentiment d'impunité en changeant, non pas son comportement, mais son logiciel - et pire- la manœuvre à toutes les chances d'être efficace, et la loi de se retourner contre les empêcheurs de télécharger en rond (notamment à propos de la confidentialité des échanges d'une communication chiffrée).

Si le câblage reste bien le même, son volumineux contenu peut donner des sueurs froides à plus d'un.

J'aimerai bien connaître une méthode systématique et efficace pour empêcher de pirater massivement les logiciels, histoire que les logiciels libres se diffusent plus vite! Oh, pardon, le projet de loi ne parle que de l'industrie du disque et du cinéma ...

137. Le lundi 23 mars 2009 à 01:06 par Bruno

@Le Chevalier Bayard en 135, "Vous êtes taquin Bruno !"

Tout à fait, j'adore susciter des réactions chez les autres ou même simplement lire leurs échanges ou écouter leurs débats. C'est souvent très enrichissant, surtout lorsque tout oppose les intervenants.

@Eolas en 134, je pensais bien à une vérité ou à une histoire altérée et non à ce que peuvent être, prises individuellement, de simples erreurs matérielles. Et je crois que vous vous empresseriez vous-même de faire rectifier si un élément particulier transformait totalement la perception d'une cour.

138. Le lundi 23 mars 2009 à 02:02 par Bruno

@Chevalier Bayard en 135, "Nietzsche pour sa généalogie de la morale etc, etc, etc,...Le lien avec Beccaria et les Russes est trop subtil pour moi !"

En Russie, nous serons à deux au moins, en errance, sur le terrain du droit, de la justice, de la morale et de la philosophie. S'agissant de nos morales et de certains conflits franco-russes qu'elle peut alimenter, j'en ai une petite idée pour avoir quelque peu approfondit le sujet à travers différentes affaires dites familiales.

Mais Beccaria aurait bien été reçu aussi: books.google

Je lirais cela plus tard. Sans notre échange, ne n'aurai pas pensé à utiliser les mots clefs qui m'ont mené là.

C'est un autre univers que le notre. Là bas, près d'un quart de la population masculine a déjà séjournée derrière les barreaux.

Actuellement la population carcérale de Russie compte 895.000 personnes. Les femmes représentent environ 5 % du nombre de détenus en Russie.

139. Le lundi 23 mars 2009 à 02:06 par colinate

Je vous lis relativement assidument, mais là je crois qu'il faut que je m'exprime.

Le problème, le plus grave a mon sens dans l'éducation des personnes qui font du droit s'est que les autres disciplines sont souvent philosophie ou science politique... mais pas psychologie, je pense que s'est une grave erreur de notre société (il y en a bien d'autre surtout en droit). Bien sûr on juge des faits. Quoique je dire une certaine vérité des faits. Les êtres humains sont ainsi faits que nul ne pourra jamais découvrir toute la vérité des faits et puis surtout quelle vérité ?

Exemple : donner un coup violent à sont concubin est presque toujours puni. Mais pourquoi en est ’ il (ou elle) arriver la ? Violence psychologique et dans se cas les preuves seront difficile a prouvaient et la loi aura ainsi fait que le vrai coupable dans l’histoire sera vainceur. Que l’on se comprenne bien, l’autre est bien sûr coupable de violence physique. Mais ou est la vraie victime ? (Le plainent, l’accusé...) Car la justice a rarement le temps d’approfondir ce genre de plainte.

Mais fait rentre des gens dans des stéréotypes de cette façon, s'est assez scandaleux. Bien sur vous pressier « sain d'esprit » (ça veut pas dire grand-chose d'ailleurs). Mais se type de mécanisme de devoir trouverais justification rationnelle a toute chose que l'on fait et une méthode de protection chez les névrotiques, il en et tout autrement cirez lez psychotique et les pervers par exemple. Dans les deux derniers cas que je viens de citer, il y a des degrés de folie et certaines personnes sont tout aussi saines qu'un névrotique. Alors bien sûr vous n'etre pas le seul a pensé ça, et puis vous etre avocat se qui est moins grave que certaine autre profession dans le droit. Mais je trouve dommage d'applique cette règle de facon et de l'ignore pour certain cas seulement. Le droit encore plus que la société aime bien faire rentre les êtres humains dans des cases, mais celle si est vraiment trop grande...

P.-S. Désolé pour les fautes je suis dysorthographique.

Eolas:
Ce n'est pas l'orthographe qui est dérangeante mais l'alignement de clichés que vous faites sur la matière juridique. Si vous me lisez assidûment, c'en est vexant. Le droit ne sert pas à faire entrer des êtres humains dans des cases mais à qualifier des faits et à y appliquer les conséquences que la loi attache à cette qualification. Point.. Ce que je dis n'est pas de la psychologie (science dans laquelle je suis ignare) mais de la criminologie, la science qui se penche sur le comportement criminel, et surtout sur les causes du passage à l'acte. Depuis deux siècles que cette science existe, elle a eu le temps de constater qu'il existe sinon des constantes du moins des points communs chez les délinquants et criminels. Le plus fréquente est le délinquant d'opportunité, celui que je décris ici. Et par sain d'esprit je parle d'une personne responsable pénalement, c'est à dire n'ayant pas agi sous l'emprise d'un trouble psychique ou neuropsychique ayant aboli son discernement. Vous comprendrez que sain d'esprit est plus rapide à taper et économise ma bande passante.

140. Le lundi 23 mars 2009 à 03:43 par Greg

Je vais jouer à ce jeux là aussi :)

Pour obtenir un visa US, il faut fournir un papier certifié indiquant la réception d'un salaire mensuel et suffisant au seuil définit par les américains pour vivre chez eux sans travailler. Ce certificat est facilement faisable par mon travail.

Sauf que... mon travail ne peut délivrer QUE des documents en français, et l'administration US ne peut recevoir QUE des documents en anglais.

Pour la traduction, il faut passer "évidemment" par un organisme agrémenté par les deux parties pour effectuer la traduction. (ne me demandez pas de citez des noms, ces organismes ne les connaissent pas eux mêmes.)

Au final, j'ai fais la traduction moi-même (elle est juste j'en suis sur), mais pourtant.. c'est un faux (m'en fiche.. je serai jamais contrôlé :P)

Toujours est il. Produire un "faux" document disant une chose vraie, est il toujours faux ? (réponse oui je sais) (j'en profite pour bénir à nouveau le "copie conforme" délivrable par moi-même depuis son changement d'il y a quelques années)

On parle d'interopérabilité de fichiers... celui qui la fera pour les administrations entre pays aura lui, tout mon respect...

Eolas:
Si la traduction est exacte, ce n'est pas un faux, faute d'altération de la vérité. Sera un faux la mention que cette traduction a été faite ne varietur par un de ces organismes agréé (et non agrémenté, à moins que l'ambassade des États-Unis ne soit aussi en charge de la décoration de ces officines).

141. Le lundi 23 mars 2009 à 05:48 par PrometheeFeu

@Greg: Vous etes un idiot... Je suis justement en train d'obtenir mon visa US et j'ai du faire traduire des document francais. Il m'a suffit de trouver un service qui faisait des traductions et les faisaient certifier par un notaire: 3 jours 20 dollars. Si ils vous attrapent, rejettent votre demande et vous expulsent: priceless. There are some things money can't buy. For everything else, there is Master Card.

142. Le lundi 23 mars 2009 à 09:25 par Bruno

@Eolas en 139 "...mais l'alignement de clichés que vous faites sur la matière juridique. Si vous me lisez assidûment, c'en est vexant. Le droit ne sert pas à faire entrer des êtres humains dans des cases mais à qualifier des faits et à y appliquer les conséquences que la loi attache à cette qualification. Point.."

Chez les JAF et les JPE, avec le renfort de travailleurs sociaux et de "psys", le droit sert aussi à dépeindre des personnalités, à les classer également, très souvent à exclure ou à faire disparaitre aussi. Par exemple, un rapport, et paf, magique, un enfant n'est plus là. C'est bordé par le droit.

Eolas:
Vous me faites penser à ces primitifs qui, voyant les blancs parler dans des radios et le lendemain, paf, magie, des avions parachutent du matériel, construisirent des boites vides avec des simulacres de boutons et de cadrants pour prier dedans les dieux de leur faire tomber du ciel ce dont ils avaient besoin. Vous en restez à la surface. C'est fréquent quand on prétend faire des généralités à partir d'un cas personnel qui nous aveugle.

Pour les autres, imaginez que le rapport constate que les enfants font état de visites nocturnes de leur père, que sa petite fille de 6 ans se plaigne que papa lui caresse le zizi, tandis qu'un psychologue constate chez le père une profonde immaturité affective illustré par le fait qu'il reconnaisse se masturber en présence de ses enfants, qui seraient trop jeunes pour comprendre. Paf, magie, les enfants disparaissent (de la vue du père, en fait, ils existent toujours, ils sont juste placés chez leur mère, de la famille ou en foyer). Et oui, c'est bordé par le droit. Le père sera évidemment scandalisé. Je vous laisse décider si vous devez l'être.

NB : Je n'insinue nullement que c'est ce qui est arrivé à l'auteur de ce commentaire. Je dis que ce qu'il raconte est partiel, partial, et peut être tout à fait normal.

143. Le lundi 23 mars 2009 à 09:41 par zorro

>Eolas : "C'est parce que la justice ne sert pas à réparer le préjudice (la preuve : la victime du vol ne s'est pas constituée partie civile). La justice pénale vise à sanctionner un comportement interdit par la loi, à prévenir la réitération de l'infraction et permettre le reclassement du condamné. Ici, le prévenu prétendu avoir compris la leçon, et produit un faux pour essayer de tromper la justice. Donc il n'a pas compris la leçon. D'où grosse peine au-dessus de la tête pour prévenir la réitération. Avec un sursis pour ne pas l'empêcher de se réinsérer."

Dans l'affaire qui nous occupe, de quels faits l'individu est accusé ? Il s'agit de vol et non pas d'usage de faux dans le but de tromper la justice. Il est probable que l'individu en question n'a pas souhaité en cette occasion malheureuse réveler à son père qu'il n'était pas inscrit en fac de droit. Sans doute, s'est il retrouvé pris au piège de sa propre manipulation, je ne pense pas qu'il ait cherché délibérement à tromper la justice puisqu'il semble évident que l'usage du faux n'avait pas pour mobile initial de tromper la justice mais simplement son père. Bref, verdict démesuré et inutile.

Eolas:
À se demander pourquoi on fait encore des audiences. Vous n'avez pas besoin de connaître le dossier ni d'être présent, il suffit qu'on vous dise ce qui s'est passé. Et quel aurait été selon vous, ô divin haruspice des comptoirs, une décision équilibrée et utile ?

144. Le lundi 23 mars 2009 à 09:49 par André Lavigne

Merci monsieur Eloas pour cet article intéressant. Je ne savais pas qu'il était si facile de produire des faux devant un tribunal. Dommage que l'histoire ne dise pas si le jeune voleur/faussaire fut finalement condamné et à quelle peine.

Eolas:
Il est très facile de produire un faux devant un tribunal. Il est très facile de vendre du cannabis. Il est très facile de frapper sa femme ou ses enfants. La délinquance est facile ; elle est même parfois la facilité.

145. Le lundi 23 mars 2009 à 09:49 par Petruk

@Ariane 102. je ne partage pas du tout le point de vue de dante Timelos. Par contre, en ce qui concerne votre cas, j'aurais tendance à penser que si la police m'arrête pour meurtre alors que je me sais innocent, la prudence est de mise.

Je ne vais pas griller bêtement ma principale carte. Une fois mon alibi connu, la police pourrait interroger de nouveau les témoins à charge en leur suggérant une heure différente espérant qu'ils modifient leur version. Il me semble que si la situation est grave à ce point, il vaut mieux attendre un peu que les versions des témoins se soient un peu décantées, quitte à passer un mauvais moment.

Si la police et le juge d'instruction font correctement leur boulot, ils n'auront pas besoin de mon alibi pour se rendre compte que je suis innocent ou qu'ils n'ont pas assez d'éléments. S'ils travaillent un peu rapidement, comme je n'ai pas envie de finir comme Richard Roman, il me semble qu'il vaut mieux procéder avec le maximum de circonspection.

146. Le lundi 23 mars 2009 à 10:41 par Barbecane

Ah si maintenant en commentaires, Eolas gratifie le lecteur de ce que doit être Internet, c'est parfait ! "Le passage à l'acte", de par le sentiment d'impunité ou la justification morale, serait ainsi une catégorie capable de subsumer tout acte non conforme à la loi. A l'appui, dans cette catégorie, le dealer, le voleur et le "pirate". Pourquoi pas le violeur (responsable), le sans papiers et l'assassin? A ce compte là tout le monde rentre dans le même panier (à salades). Ah mais pardon. Ca fait autrement plus humaniste de contester une politique d'immigration nationaliste que de récuser l'iniquité d'une loi issue d'un lobbying mercantile et d'un modèle obsolète. C'est autrement plus gratifiant de s'appesantir sur l'illicéité du deal de cannabis, plutôt que porter le problème en macro-économie où l'impact financier du narco-trafic est négocié entre Etats "producteurs" et "consommateurs". (France/Maroc...) "Dans" la loi pour en contrer les abus et ce qu'en conscience, l'on tient pour injuste. Ok. Merci de me réserver, dans le droit, le droit de contester le droit, en tant que citoyen. Car à la majorité de ces derniers, c'est bien l'action du législateur qui doit demeurer contestable et donc modifiable. Et quand ce n'est pas par le droit, cela doit être par la force, merci à la DDHC et à Thomas Hobbes. La soumission systématique au droit lui reconnaît le statut de dogme et lui confère une transcendance abusive. La régulation comportementale imprimée par l'action normative du droit ne peut se passer de son contraire. De même que l'association du droit à celui de justice ne saurait faire l'économie de recourir à des valeurs morales et de définir imparfaitement ce qui sépare le juste et l'injuste, le bien du mal, etc. Le concept de "pirate" pour celui qui, au regard d'une loi (nationale), télécharge des kilo-octets illégalement, induit une valeur morale à son action, comme l'indique la sémantique employée à définir la qualité du geste. Le terme ne s'appliquait au début qu'à des personnes qui, à l'instar de certains scientifiques, considéraient que les codes ou le génome ne devaient pas, au nom du principe de bien commun de l'humanité, être brevetés et accaparés à titre privé. Grande confusion entre les biens non marchands et la propriété intellectuelle qui s'ensuivit, problème de définition d'un contenu numérique. Vous êtes le premier à critiquer la loi lorsque vous la trouvez mal faite ou injuste. L'action de décryptage que vous menez est d'intérêt public. Alors de grâce, pourquoi simplifier à l'excès par des amalgames catégoriels, au prétexte de moucher à qui il revenait de l'être?

147. Le lundi 23 mars 2009 à 10:47 par mourkos

"Se taire en garde à vue, c'est comme l'appel à la chasteté contre le SIDA. C'est le plus efficace mais c'est généralement au-dessus des forces humaines. "

j'adore !

148. Le lundi 23 mars 2009 à 11:05 par Petruk

@Barbecane "La régulation comportementale imprimée par l'action normative du droit ne peut se passer de son contraire." J'ai "un peu" de mal à vous suivre...Suis-je le seul?

Je lance un QCM où il s'agit de trouver le contraire de la phrase de Barbecane et je propose :

  • l'action normative du droit imprimée par la régulation comportementale
  • La régulation normative imprimée par l'action comportementale du droit
  • l'action comportementale imprimée par la régulation normative du droit

149. Le lundi 23 mars 2009 à 11:20 par Simplet

@ Eolas en 142

"...que sa petite fille de 6 ans se plaigne que papa lui caresse le zizi..."

150. Le lundi 23 mars 2009 à 11:22 par Simplet

J'avais mis trois points d'interrogation après, qui ne sont pas passés. L'anatomie de cette petite fille me surprenait.

Eolas:
Je vous assure qu'une petite fille de six ans n'emploie pas le mot de vulve.

151. Le lundi 23 mars 2009 à 11:25 par Dante Timélos

Tout d'abord, tout le monde aura compris ici que je suis inspiré par un point de vue radicalement critique sur "notre moins pire des système", ce qui implique aussi un point de vue radicalement critique sur le droit. Je ne peux donc que heurter les conceptions des professionnels de la profession qui, sur ce site, défendent avec morgue leur petit pouvoir de spécialiste. Mais je ne souhaite pas développer ici cette polémique: je pense qu'elle serait hors sujet. Si les positions que je défends intéressent quelqu'un, qu'il aille voir ailleurs les textes où elles sont développées. (Et je ne donne pas plus de précision, pour ne plus être accusé de faire de la pub déguisée. Vous n'aurez qu'à chercher).

Juste un mot concernant un point précis, en réponse à Ariane (commentaire 102). J'ai bien dit qu'il fallait se taire "en garde à vue" et "réserver ses déclarations aux magistrats". Maître Eolas, dans sa réponse, ne dit pas autre chose. Le point central, c'est d'avoir une idée de ce qui est dans le dossier. Le problème, c'est que dans la procédure qui conduit à la CI on se retrouve devant le proc avant que l'avocat puisse voir le dossier. Mais bon ne pas oublier que les magistrats qui comptent vraiment en CI ce sont le président et ses assesseurs. Ce sont eux qui vont vous juger au bout du compte. A eux il faut absolument parler : et bien si possible... je suis le premier à déplorer que les gens qui s'expriment mal, qui ne sont pas les plus cultivés, soient les plus condamnés, mais bon, c'est ça aussi la justice de classe.

152. Le lundi 23 mars 2009 à 11:28 par Bruno Kant

@Eolas en 142 "construisirent des boites vides avec des simulacres de boutons et de cadrants ... Vous en restez à la surface. C'est fréquent quand on prétend faire des généralités à partir d'un cas personnel qui nous aveugle."

Depuis quelques temps et plus encore depuis la "disparition" de la petite Elise, il me démange de remuer un peu la merde et d'évoquer d'autres affaires, plus anciennes. Un cas particulier avait fait beaucoup de bruit par le passé, ce que Poutine en pensait avait été diffusé sur les ondes de la radio de Moscou; vous trouverez aussi trace d'échanges très officiels entre Poutine et Chirac, dans les archives de l'Elysee. Un petit billet sur ces thèmes pourrait être très drôle pour peu que j'évoque également la dépression de Sumo ainsi qu'une audition d'un dalmatien...

"Je dis que ce qu'il raconte est partiel, partial, et peut être tout à fait normal."

Ce que je raconte est simplement caricatural et correspond bien à la réalité de quelques cas particuliers. Par ailleurs, ce que je décris rejoint tout à fait les conclusions de chercheurs et de professionnels. Vous même descernez bien des prix Busiris à notre Garde des sceaux (à champagne)... serait-elle la seule que nous puissions critiquer?

"Pour les autres, imaginez que le rapport constate que les enfants font état de visites nocturnes de leur père.."

Là, vous versez vous même dans la carricature et les extrèmes, comme de nombreux autres professionnels de ces sphères que je suspecte parfois de défendre avant tout autre chose leurs chappelles respectives. Ceux qui se donnent la peine de parcourir mes références et mes notes - et c'est bien l'une des raisons pour lesquelles je les expose - constateront par eux même que je prend bien en considération la diversité des situations, que je ne me contente pas de me référer à quelques cas emblématiques.

A vous lire, tous les parents d'enfants placés pourraient être des sortes de Moniques et Michels (Fourniret)... Or seul environ 20% des enfants placés l'ont été en raison de défaillances parentales ou de maltraitance avérée et grave (ou supposée, suspectée; souvent, des procès puis jugements confirment).

"sa petite fille de 6 ans se plaigne que papa lui caresse le zizi"

Vous affirmez que vous êtes un expert du pénal, quelqu'un de compétent en criminologie (et je n'ai pas de raisons d'en douter). Peut être ne connaisez vous que de tels cas.

"Le père sera évidemment scandalisé. Je vous laisse décider si vous devez l'être."

Renseignez vous. Je connais également de nombreuses mères, grands-parents et latéraux, eux également scandalisés par certaines pratiques "professionnelles".

153. Le lundi 23 mars 2009 à 11:39 par Barbecane

Le jeu étant ce qu'il est vous en êtes déjá le gagnant.

154. Le lundi 23 mars 2009 à 11:59 par Ash

Concernant ce post, dont je ne saurais contester la démonstration, il me semble qu'un aspect important du mécanisme psychologique menant au passage à l'acte a été omis : la pression extérieure.

En effet, il me semble fortement improbable que notre prétendu étudiant produise un faux certificat sans demande de justification de son oisiveté répété par l'autorité paternelle, ou de justification de son affiliation à Framboisy II par l'autorité judiciaire. De même, le voleur de sac à main semble moins disposé à encourir les moult peines et tracas du délit sans l'amicale pression de son estomac (pression externe ou interne, je laisse le débats aux augustes gastro-entérologues qui passeraient par ici).

Il me semble que cet aspect conditionne bien souvent, lorsqu'il est combiné aux deux autres exposés par le maître des lieux le passage à l'acte, et est même souvent l'indicateur temporel du déclenchement (je vole / mens quand j'en ai besoin, et non pas dès lors que cela m'est justifié moralemen et que je me suis assuré de ne pas être pris).

155. Le lundi 23 mars 2009 à 12:01 par adrien

@Barbecane en 146

Heuuu... je crains de ne pas avoir compris le texte de la même façon que vous. Par "passage à l'acte", je pense qu'il faut comprendre "fait d'agir de façon contraire à la loi". Et même en toute connaissance de cause.

Et le maître d'étudier les différentes causes de ce passage à l'acte. Et cela me semble très pertinent. Je respecte le code de la route. Même sur une ligne droite au fin fond du gers, avec pas un flic à l'horizon, je reste à 90. Parce que je ne voit aucune justification au fait d'aller plus vite.

Quand petit je tondais les pelouses pour mes voisins, ou plus tard je donnais un cours de maths à quelqu'un, je ne déclarais pas ces sommes gagner au fisc. Tout simplement parce que bon, pour 20€, on va pas en faire un plat quand même.

J'ai pas lu beccaria ou qu'est-ce, mais ça m'a pas l'air faux quand même...

Après une autre question est-celle de la justesse desdites loi, et des façon de lutter contre les loi "injustes". Mais je ne crois pas que cela fut le thème initial de ce billet. C'est assurément un thème connexe, mais ne blâmons pas l'auteur de ne pas l'avoir explicitement formuler.

156. Le lundi 23 mars 2009 à 12:12 par ouaip

Je n'ai pas du tout apprécié votre petite blague avec le mot "aggrave"

Les meilleures blagues sont les plus courtes, et si on peut rire de tout, ça dépend de la façon dont c'est fait et de l'idéologie qui se cache derrière, et il s'agirait de ne pas rire au dépens de milliers de morts. Merci.

(Si je donne des leçons je ne me considère pas parfait pour autant)

Eolas:
Gardez vos leçons et surtout vos cadavres. Un mot n'a jamais tué personne, et celui là tout particulièrement.

157. Le lundi 23 mars 2009 à 12:24 par ouaip

Maitre Eolas cachant une batte de baseball derrière son dos se trouve a coté d'un troll assommé, portant une longue bosse sur le sommet du cuir chevelu. Un texte “Commentaire modéré par troll détector™” surplombe l'image.

158. Le lundi 23 mars 2009 à 12:26 par Krone

Maitre Eolas cachant une batte de baseball derrière son dos se trouve a coté d'un troll assommé, portant une longue bosse sur le sommet du cuir chevelu. Un texte “Commentaire modéré par troll détector™” surplombe l'image.

159. Le lundi 23 mars 2009 à 12:35 par Bruno Kant

@Eolas en 142,

Il me semblait vous avoir répondu. Y aurait-il eu un petit incident technique?

Eolas:
Si vous utilisez des gros mots, soyez patients.

160. Le lundi 23 mars 2009 à 12:38 par zorro

>Eolas "À se demander pourquoi on fait encore des audiences. Vous n'avez pas besoin de connaître le dossier ni d'être présent, il suffit qu'on vous dise ce qui s'est passé. Et quel aurait été selon vous, ô divin haruspice des comptoirs, une décision équilibrée et utile ?"

Je ne connais du dossier que ce que vous avez jugez bon de nous transmettre pour votre propre demonstration. Je veux bien reconnaitre que c'est tout à fait insuffisant pour juger, mais dans ce cas, reconnaissez également que votre démonstration n'est pas recevable.

Eolas:
Ma démonstration était : il ne faut pas produire de faux en justice ; pas : la justice ne rend que des décisions équilibrées et utiles. J'admets donc que la démonstration que je n'ai pas faite n'est pas recevable. On progresse à grands pas.

Mon opinion est d'ordre général : lorsque le préjudice est maigre (60 euros), il est important de juger les intentions de l'accusé, à savoir dans quelle mesure son comportement délictueux relève d'avantage d'un accident de parcours que d'une absence totale de conscience morale.

Eolas:
Précisément ; le fait qu'il fasse état devant le tribunal saisi de son affaire d'une fausse qualité étayée par la production d'un faux tend à faire opiner dans quel sens, selon vous ?

Or dans le cas que vous nous relatez, rien ne nous permet de juger de façon claire les intentions de l'accusé.

Eolas:
Chez Zorro, le bandeau devant les yeux… Il faut faire deux trous dedans au niveau des yeux.

Voilà pourquoi le verdict intransigeant (sans compter la citation à comparaitre pour faux et usage de faux, et les difficultés familiales à venir) apparaît davantage comme une volonté de punir lourdement qu'une volonté de contribuer à l'édification morale d'un individu manifestement égaré.

Eolas:
Si un juge s'avisait de vouloir procéder à l'édification morale de mon client, je dégainerais mon code pénal.

161. Le lundi 23 mars 2009 à 12:41 par Obsidian

Encore merci, cher Maître, pour cet excellent billet qui, comme tous les précédents, se lit comme un roman ! J'aimerais toutefois apporter mon point de vue en ce qui concerne le piratage : d'abord, je m'inquiète du raccourci qui est fait entre le mot « télécharger » seul et le fait de télécharger des œuvres contrefaites. Ça en dit long sur la perception qui est faite des réseaux informatiques par les profanes, y compris au moment de voter une loi.

Ensuite, on ne peut pas mettre côte à côte les voleurs de sacs à main et les « téléchargeurs » d'œuvres musicales, car il s'agit de vol dans le premier cas et fraude dans le second. C'est tout autant condamnable sur le principe, mais il est de fait que l'argument « je n'aurais pas acheté ce disque de toutes façons » est probablement vrai et que pour le reste, ça n'empêche pas le support physique d'être vendu à une autre personne. Le manque à gagner découle alors seulement du fait que l'acte incite d'autres personnes à faire de même et, au final, impliquent des personnes qui auraient acheté le disque.

Pour autant, je ne cautionne absolument pas le piratage, ni même ne l'excuse. Mais je trouve que ces temps-ci, on a tendance à asseoir certains projets de lois sur des amalgames douteux.

Eolas:
Je ne suis pas sûr que ce site soit le meilleur endroit pour venir y dire ce qui ici relève de l'évidence.

162. Le lundi 23 mars 2009 à 13:05 par Guillaume

Qu'en est-il des infractions commises en état d'ébriété (pour le passage à l'acte j'entends)? Je sais (crois savoir ?) que c'est une circonstance aggravante (en plus d'être une infraction en soit quand on est sur la voie publique ou au volant) et on sort du cadre "chez un être sain d'esprit". La justice considère-t-elle le passage à l'acte de la même façon ?

163. Le lundi 23 mars 2009 à 14:01 par Luc Montesquieu

@ Guillaume, commentaire 162

Je vais répondre à votre question en évoquant, une fois encore, le cas de Crétin... Vous savez, ce jeune automobiliste qui était pressé d'arriver à la fameuse soirée Harry Potter dont le maître des lieux était le maître de cérémonie. Comme vous le savez, Crétin est arrivé en voiture, ce que vous ignorez, c'est que Crétin a bu pendant cette soirée. Je ne doute pas un instant que le maître des lieux ait tenu à ce que chacun soit sobre, mais vous connaissez Crétin... Bref, il a bu et a une alcoolémie supérieure aux normes en vigueur. Récapitulons, il est venu en voiture, il savait, avant même de boire qu'il devait rentrer, par conséquent, il savait, avant même de boire qu'il commettrait une infraction au code de la route. Il était donc parfaitement sain d'esprit au moment où, pour la première fois de la soirée, il a levé son verre... puis le second... Ce n'est qu'à partir du 2nd verre que vous atteignez l'alcoolémie critique... Bref, il boit le second et d'autres encore... Au moment où il prend son 3ème verre, il SAIT qu'il commettra une infraction au code de la route s'il rentre immédiatement...

Mais Crétin sait autre chose, il n'a pas le vin joyeux et l'alcool le rend irascible... Les esprits s'échauffent, Eolas qui s'est rendu compte que notre jeune ami n'était pas dans son état normal tente de calmer le jeu lorsque, hélas, le jeune Crétin administre un coup au maître de cérémonie... Que penser de cet acte ? Certes, il ne l'aurait jamais fait s'il n'avait pas bu... Espérons le, mais il SAVAIT qu'il pouvait être dangereux s'il buvait et n'a pas pris les mesures diligentes qui s'imposaient... Il a commis des coups et blessures volontaires avec l'alcoolémie pour circonstance aggravante.

Mon illustration répond-elle à votre question ?

Maître Eolas, en affirmant que les circonstances atténuantes n'existent pas, me trompé-je ? Si oui dans quelle mesure ai-je tord ?

Eolas:
Vous avez bon sur le propos. Sur le droit, les circonstances aggravantes ont disparu en 1994 avec les peines minimales. L'ancien code pénal prévoyait des peines par intervalles : le vol était puni par exemple de 6 mois à trois ans, de mémoire. Pour aller en dessous de ce minimum, le juge devait retenir les circonstances atténuantes. Le nouveau code pénal ne fixe plus que le maximum. Le juge est en principe libre de descendre jusqu'à un minimum d'un jour, d'un an si c'est un crime, deux ans si c'est la perpétuité qui est encourue. Ce principe a duré 13 ans, jusqu'à ce que la loi sur les peines plancher revienne dessus.

164. Le lundi 23 mars 2009 à 14:11 par Bruno Kant

@Eolas en 159 "Si vous utilisez des gros mots, soyez patients. "

J'ai employé divers mots dont Busiris, sceaux, Sumo, Poutine et champagne... et je crois me souvenir que j'ai copié vos propres mots, "c4r3ss3 le z!z!". Je vais patienter.

165. Le lundi 23 mars 2009 à 14:14 par Natoussia

Cher Maître Eolas, je voulais rebondir sur le commentaire de Marcel (80), qui disait que la consultation d'un guide juridique pouvait être utile au béotien qui pourrait un jour se retrouver devant un juge. Vous avez balayé son argument d'une phrase lapidaire car il avait, fort maladroitement, justifié la nécessité de cette démarche en évoquant l'éventuelle négligence de l'avocat.

Je suis moi-même complètement étrangère au monde merveilleux des tribunaux, je n'en entrevois les délices que grâce à quelques excellents blogs, mais si un jour je devais passer devant un juge (je crache trois fois par terre en me grattant les ovaires), je vous assure que je me jetterais sur tous les guides du Droit pour les Nuls, Easy justice pour pécains lambdas et autres Martine et Oui-oui au tribunal que je trouverai, en plus bien sur de prendre un avocat. Et puis, en cas de comparution immédiate, je serai bien aise de les avoir tous lus et potassés préalablement. Car vous le dites vous-même, si l'avocat est le chef, le prévenu est le co-pilote, et même s'il n'a pas le volant, c'est bien s'il arrive au moins à lire la carte routière, et si en plus il a son permis, et donc connait les contraintes du pilote, ça facilite grandement le trajet... (oui, j'adore filer les métaphores moi aussi) Bref, mon avocat, si génial et doué qu'il soit dans le travail d'explication, n'apprécierait-il pas de s'adresser à quelqu'un qui ne lui pose pas dix mille questions débiles (c'est qui le monsieur avec la robe bizarre?) -question que j'aurais été tout à fait capable de poser avant de lire ces excellents blogs que je mentionnais- alors qu'il n'a qu'un temps très court pour préparer le dossier? En bref, ne vaut-il pas mieux faire la promotion de tout ce qui donne accès au plus de connaissances possibles, dans une langue compréhensible même aux blondes?

Eolas:
Intéressant. Et avant d'aller chez le médecin, vous lisez tout ce qui se publie en guide santé et bien-être ?

Cela dit, je n'ai rien contre le principe des guides juridiques. Mais faites attention à qui l'a écrit. Un diplôme en droit ne suffit pas, il faut vraiment la pratique des prétoires. NB : pour le guide juridique de Dante Timélos, je ne l'ai pas lu et ne puis donner d'opinion approbatrice ou désapprobatrice.

166. Le lundi 23 mars 2009 à 14:27 par IceCream

@Natoussia De vrais cours d'Education Civique dans les collèges, lycées seraient déjà une très bonne chose!!! et en plus gratuits et contrôlés (sur la justesse des propos j'entends). Pour le reste renseignez vous, mais n'attendez pas que l'ensemble de la population fasse de même, après tout c'est le métier des avocats, et autres juristes!!!

Au fait, les questions ne sont jamais débiles!!!

Eolas:
C'est déjà le cas (bon, ce ne sont que quelques heures chaque année, non couronnées d'un examen…) et à Paris, l'association Initiadroit envoie des avocats volontaires effectuer des interventions de quelques heures en collège et lycée. C'est gratuit, messieurs les proviseurs, et côté justesse des propos, on ne fait que difficilement mieux.

167. Le lundi 23 mars 2009 à 14:43 par Natoussia

@IceCream, c'est bien de refaire le monde et d'imaginer ce qui serait le mieux... Oui, effectivement, si on nous apprenait le droit à l'école, et si tous nos profs étaient en plus excellents, les élèves attentifs et les collèges et lycées pourvus de plus de moyens, ça serait encore mieux. Comme ca tout le monde serait content et avec un petit effort, Oui-oui et Martine n'iraient jamais au tribunal.

En attendant ce jour lumineux, je suis bien contente d'avoir à ma disposition un maximum de sources d'informations pour me renseigner sur les très nombreux domaines dans lesquels je suis ignarde.

@Maitre Eolas Si je vais chez le médecin pour un rhume, non. Pour un cancer ultra carabiné, oui. Pareil dans le cas présent, si ma liberté ou même mon honneur étaient en jeu, évidemment que je lirais tout ce que je peux... Pas vous?

Eolas:
J'irai plutôt voir le meilleur spécialiste et lui poserai toutes les questions que je me pose. Chacun son truc.

168. Le lundi 23 mars 2009 à 15:02 par IceCream

@Natoussia

apparemment vous prenez ça au sérieux!!! Bigre! je n'ai pas précisé le ton ironique de ma remarque... Désolé (voix de Denisot).

Pour vous faire patienter jusqu'au "jour lumineux" (dont vous ne bénificierait pas, à moins d'être encore au lycée) inscrivez vous dans des DU de droit ou au diplome donnant la capacité en droit (introductions aux matières juridiques dispensées généralement le soir afin d'être plus accessibles aux personnes actives).

Au moins vos sources seront d'une qualité certaine!!!!

169. Le lundi 23 mars 2009 à 15:10 par Koudou

Il me semble que vous oubliez un élément dans le passage à l'acte : le gain. Plus le gain potentiel est grand et plus on accepte le risque.

170. Le lundi 23 mars 2009 à 15:11 par Natoussia

@IceCream, ben oui, c'est sérieux la justice quand même ! J'adorerais m'inscrire en DU de droit, si seulement mes contraintes temporelles (nécessité de faire acte de figuration dans un bureau entre 9h et 18h pour gagner ma pitance) et géographique (j'habite au pays des ours mafieux qui boivent de la vodka et qui enlèvent les petites filles françaises) ne m'obligeaient pas à me contenter de ce que l'ami Internet peut me donner...

Eolas:
Si vous êtes vraiment motivée, le CNED propose des formations en droit de niveau universitaire. Привет aux ours.

171. Le lundi 23 mars 2009 à 15:11 par chihuahualibertaire

Est-ce si simple ? Si toutes les questions peuvent se résoudre aussi simplement pourquoi autant de discussions, la rationnalité juridique devrait suffire. Je vais réfléchir à mon prochain délit idéologique en attendant la condamnation.

172. Le lundi 23 mars 2009 à 15:31 par Natoussia

@Eolas ; Peut être que si je préparais mon entretien avec le meilleur spécialiste en lisant toute la doc que je trouve sur ma maladie, mes questions seraient plus précises, et je serais mieux renseignée... Et puis peut-être aussi que le meilleur spécialiste n'est pas le meilleur pédagogue et que j'ai besoin d'une petite aide pour mieux comprendre ce qui m'arrive... Pourquoi serait-ce incompatible? Pourquoi est-ce que la lecture de guides juridiques ou d'autres types d'information de type explicatives (comme par exemple des blogs) ne seraient-elles pas d'utiles compléments à la nécessaire consultation d'un avocat? C'est tout l'objet de mon commentaire.

Eolas:
Disons que le jour où vous serez malade, que votre voiture sera en panne, ainsi que votre téléviseur, que votre ordinateur aura planté et que des fissures apparaîtront dans votre plafond, ce sera la fortune des éditeurs.

173. Le lundi 23 mars 2009 à 15:58 par Guillaume

@ Luc Montesquieu 163

Je suis d'accord sur l'analyse mais je tique sur un point : "Mais Crétin sait autre chose, il n'a pas le vin joyeux et l'alcool le rend irascible... " A moins d'avoir des antécédents dans ce sens, comment l'affirmer ? Crétin a peut-être exceptionnellement "péter un cable" ce soir là, d'habitude quand il boit il dit une ou deux conneries qui font rire tout le mode puis s'endort dans un coin comme une loque.

174. Le lundi 23 mars 2009 à 16:09 par dwarfpower

@eolas reponse 172 et supra

Il y a une grosse différence entre le droit et la médecine. Votre corps ne vous demande a priori pas de comprendre son fonctionnement pour vivre, et le médecin vous dira ce que vous faites mal si vous faites n'importe quoi.

Si vous vous retrouver face à la justice sans l'avoir cherché il est mieux de connaitre les règles pour les respecter correctement. Et certains avocats ( je parle d'expérience) vous montrent la porte si vous cherchez à comprendre et posez des questions; genre "si vous voulez vous défendre seul, je vous en prie, bonne chance..."

175. Le lundi 23 mars 2009 à 16:12 par Pinouille

Il y a deux saveurs dans ce blog:
1- celle, subtile et complexe, que l'on goute à la lecture des billets de Maître Eolas (celui du 18 mars inclu)
2- celle, plus acide et coupable, dont on se délecte à la lecture des réponses affutées et souvent cinglantes de Maître Eolas aux quelques commentaires exotiques qui le font sortir du bois (je ne lis que ceux là)
Saurons nous un jour qui se cache derrière le masque? (musique de thriller en fond)

176. Le lundi 23 mars 2009 à 16:14 par Natoussia

@Eolas : Oui, sauf qu'a priori, une panne de voiture, de télé ou de plafond aurait moins de chance de boulverser ma vie qu'un procès ou un cancer, voyez...

177. Le lundi 23 mars 2009 à 16:37 par Un qui drague comme un mérou

Le propos est particulierement bien illustré par le cas de Sébastien Budin, l'homme au 130 000 € d'amende, qui commence à faire parler de lui...

178. Le lundi 23 mars 2009 à 16:37 par PrometheeFeu

@Eolas:

Je pense tout du moins qu'une connaissance en droit vous permettrait de savoir quand il faut aller chercher votre avocat. Les avocats ne sont pas tous gratuit et nous n'avons pas toujours le temps d'aller vous exposer tout ce qui nous embête pour savoir dans lequel de ces cas le droit est applicable. Avoir quelques connaissances de droit, vous permettra aussi d'êtres un meilleurs citoyen. Écrire la loi est le pouvoir le plus fort que l'on offre a nos représentants législateurs. Savoir ce qu'ils en font et les interpeller quand on est pas d'accord est peut êtres une bonne chose. Mais attention, un peu de connaissance peut êtres pire que rien. Par exemple, il me semble que vous êtes tenu d'obéir aux ordres d'un policier quel que soit leurs légalité (jurisprudence du Conseil d'État, corrigez moi si j'ai tort) dans le pays des droits de l'Homme. Donc savoir quels sont les limites de l'autorité légal d'un agent de police sans savoir cette jurisprudence peu vous causer des problèmes. Avec un peu de chance, cette jurisprudence sera bientôt considérée comme anti-constitutionnelle, mais en attendant..

179. Le lundi 23 mars 2009 à 16:38 par NormalienEnPuissance

J'assiste à une réelle recrudescence de la morale en droit, et votre blog en fait echo. C'est bien dommage, surtout quand le droit ne devrait pas être confondu avec la morale. J'espère seulement que ça n'est pas l'influence de l'un de vos récent billets qui en révèle ( un peu ) plus sur vous que fait ça ... :S

180. Le lundi 23 mars 2009 à 17:20 par Typhon

"ben quoi sérieux ? J'ai pas raison non? franchement heh, si c'est pour buter des méchants on peut le faire, nan ? sans déconner, si c'est des méchants"

Didier super

Typhon

181. Le lundi 23 mars 2009 à 17:26 par Elzevier et tournebroches

Je viens d'écouter la première vidéo que vous avez postée... J'espère que vous allez le faire ce billet pour expliquer pourquoi il ne faut pas parler à la police en garde à vue...

182. Le lundi 23 mars 2009 à 17:35 par Scoop

Le tribunal administratif de Seine-Saint-Denis s’implantera à Montreuil (ouverture prévue à l'automne 2009)

183. Le lundi 23 mars 2009 à 17:40 par La mère Denis

@Typhon en 179

C'est ben vrai, ça !

184. Le lundi 23 mars 2009 à 18:11 par adrien

@normalien en puissance

Certes non, le Droit ne doit pas être confondu avec la Morale, ce sont deux choses différend. Mais pour déterminer si notre pays est soumis à une forme d'oppression quelconque, nous ne pouvez qu'observer le Droit sous l'angle de la Morale.

Vichy était un Etat de Droit. Nous vivons aujourd'hui dans un Etat de droit. Seule la Morale, nous permet de dire que sous l'un nous étions opprimés, et pas sous l'autre....

185. Le lundi 23 mars 2009 à 18:11 par adrien

@normalien en puissance

Certes non, le Droit ne doit pas être confondu avec la Morale, ce sont deux choses différend. Mais pour déterminer si notre pays est soumis à une forme d'oppression quelconque, nous ne pouvez qu'observer le Droit sous l'angle de la Morale.

Vichy était un Etat de Droit. Nous vivons aujourd'hui dans un Etat de droit. Seule la Morale, nous permet de dire que sous l'un nous étions opprimés, et pas sous l'autre....

Eolas:
L'État Français de 1940-1944, un État de droit (fichons un peu la paix à la préfecture de l'Allier) ??? Vous pouvez me rappeler quels étaient les recours pouvant être exercés contre la loi du 3 octobre 1940 portant statut des juifs ? (JO 8 octobre 1940, p. 5324) ?

186. Le lundi 23 mars 2009 à 18:24 par La Biscotte

Sur le fait de lire des bouquins de droit pour se préparer à une audience :

Ca me rappelle ce type un jour en correctionnelle, cité pour un délit routier quelconque, et qui se défendait sans avocat.
Pour autant, il avait apparemment révisé son code de procédure pénale illustré, puisqu'au moment où je lui donnais la parole pour sa défense, après les réquisitions du MP, le type part sur un long exposé concernant toutes les irrégularités commises en garde-à-vue, me citant même de la jurisprudence de la Cour de cassation sur le sujet..

Bon, je l'ai laissé finir sa plaidoierie, et j'ai statué sur le dossier en le déclarant coupable, rejettant ses exceptions de nullité présentées un brin tardivement...

Faut le lire in extenso le code de procédure pénale...

187. Le lundi 23 mars 2009 à 18:24 par Greg

@141:PrometheeFeu

Amusé de voir votre jugement hâtif sur ma stupidité. Sachez juste que j'ai fait tamponné mon document par un professeur d'anglais de l'école où est rattaché mon laboratoire. Ainsi, je transfère la responsabilité en cas de problème. Cela m'a pris 15min (et un café offert au prof), là où cela vous aura pris 3 jours et 20 dollars... Suis-je toujours stupide ?

De plus, le maître des lieux le dit, il ne s'agit pas apparemment d'un faux bien que la traduction n'ai pas été faite par une organisme agrées (mais le tampon de cet organisme n'est pas obligatoire pour faire valider un document. Preuve est qu'il a été validé (par deux fois) par l'administration américaine.

Je présume donc qu'un faux document est un document ne disant pas la véritié (et ne répondant pas aux obligation légale (telle qu'un tampon)

Quand est il si je "fabrique" un certificat de scolarité identique en tout point à l'original (scanner + imprimante. Voyons cela comme une photocopie), mais que je suis réellement étudiant de l'école émettant ce certificat. Serait-ce toujours un faux ?

188. Le lundi 23 mars 2009 à 18:30 par Makura

A mon avis, demandez-vous si fabriquer une fausse carte d'identité française en tout point semblable à l'originale est répréhensible, indépendamment du fait que vous puissiez par ailleurs prétendre à en avoir une vraie, et vous aurez votre réponse...

189. Le lundi 23 mars 2009 à 18:38 par La mère Denis

186. Le lundi 23 mars 2009 à 18:24, par La Biscotte

''Bon, je l'ai laissé finir sa plaidoierie, et j'ai statué sur le dossier en le déclarant coupable, rejettant ses exceptions de nullité présentées un brin tardivement...

Faut le lire in extenso le code de procédure pénale...''

Sadique !

190. Le lundi 23 mars 2009 à 18:40 par Greg

@188 Makura :

Justement, en faire une photocopie (couleur très haute qualité) et écrire dessus "copie conforme" la rend légale et peut être utilisé à la place de l'originale...

191. Le lundi 23 mars 2009 à 19:02 par Moulinois

@Eolas en 185 "L'État Français de 1940-1944, un État de droit (fichons un peu la paix à la préfecture de l'Allier) ???"

On nous a déjà pris notre TGI, maintenant on en veut à notre prefecture !

192. Le lundi 23 mars 2009 à 19:24 par Bruno Kant

@Eolas, en 185 "quels étaient les recours pouvant être exercés contre la loi du 3 octobre 1940 portant statut des juifs ?"

Bonne question. De source www.vie-publique.fr:

1893 : Octroi du droit de vote aux femmes en Nouvelle-Zélande, premier pays au monde à l’accorder ; la Grande-Bretagne (pour les femmes de plus de 30 ans jusqu’en 1928) , la Suède et l’Allemagne l’accordent en 1918, le Canada et les Pays-Bas en 1919, les É tats-Unis en 1920 (quelques États l’avaient déjà adopté avant ).

21 avril 1944 : L’ordonnance d’Alger accorde le droit de vote aux femmes françaises.

27 octobre 1946 : Le préambule de la constitution proclame : "la loi garantit à la femme, dans tous les domaines, des droits égaux à ceux de l’homme" (art.3) .

193. Le lundi 23 mars 2009 à 20:22 par De minimis

En tant que primitif (parce que provincial) et vil flagorneur, je vous fais part, Maître, de mon admiration. La patience dont vous faites preuve me frappe (ou me touche, qualifiez comme bon vous semble). ... @ Dante Timelos. J'agitais un drapeau noir (parfois rouge et noir) il fut un temps. Mais, lorsque j'entrai en droit (en même temps qu'au monastère, d'ailleurs, puisqu'en province, ce sont les Dominicains qui enseignent le droit... une lubie à eux depuis Bernard Gui ) ma vision de la société et de l'État changea. Je compris en effet que le Législateur n'est pas mu par la seule obsession mesquine de nous pourrir l'existence et que parfois, au détour d'un Code, certains textes avaient le mérite d'assurer à chacun une tranquillité relative et, à un niveau plus global, de garantir une certaine paix sociale.

Quant à ceux et celles qui estiment qu'une nouvelle citation "offerte" au jeune et piètre faussaire est exagéré, il serait bon qu'ils fassent appel à leur bon sens: qui aime être pris pour une buse (pour ne pas utiliser un mot plus bref)?

194. Le lundi 23 mars 2009 à 20:52 par bencaalors

@186 la biscotte : ''Article 442 : Avant de procéder à l'audition des témoins, le président interroge le prévenu et reçoit ses déclarations.' Article 460 : L'instruction à l'audience terminée, la partie civile est entendue en sa demande, le ministère public prend ses réquisitions, le prévenu, et, s'il y a lieu, la personne civilement responsable, présentent leur défense. La partie civile et le ministère public peuvent répliquer. Le prévenu ou son avocat auront toujours la parole les derniers. Article 385 : Dans tous les cas, les exceptions de nullité doivent être présentées avant toute défense au fond.'''

Il a presente ses conclusions de nullites avant toute defense au fond et apres l'instruction comme le prevoi le code de procedure penale.

Pourquoi avez vous rejete ces conclusions de nullites ?

Si il ne s'etait pas presente en 1ere instance, il aurait pu presenter ses conclusions de nullite en appel bien que l'instruction ait deja ete faite en 1ere instance a l'audience parce qu'il ne s'etait pas defendu au fond.

195. Le lundi 23 mars 2009 à 20:58 par robert

Il y a quelque chose de pourri en Guyane et au Royaume d'Eolas

196. Le lundi 23 mars 2009 à 21:00 par Bruno

@Eolas, en 185 "quels étaient les recours pouvant être exercés contre la loi du 3 octobre 1940 portant statut des juifs ?"

Franchement, cette réponse là me choque. Même le Reich était un Etat de droit, j'en parlais encore il y a 10 jours, à la cour d'appel.

http://www.defense.gouv.fr, le décret NN

197. Le lundi 23 mars 2009 à 22:29 par Antigel

Bonsoir Maître!

Je suis toujours heureux de lire vos articles et commentaires. Mais dernièrement, j'ai été surpris et un peu déçu d'apprendre que vous étiez catholique pratiquant. Vous me semblerez moins libre que je ne le supposais!

Mais vous avez le temps d'approfondir la question, aussi je vous conseille, pour un début, de lire l'article: "Le Prophétisme, illusion pathologique?" à voir à:

http://www.libertyvox.com/phpBB/viewtopic.php?t=2437&start=0

La recherche de la Vérité (spirituelle) mérite qu'on y passe un peu de temps!

198. Le lundi 23 mars 2009 à 22:40 par La Biscotte

@bencaalors : non, l'expression "avant tte défense au fond" signifie avant les débats sur le fond. L'interrogatoire du prévenu est considéré comme une défense au fond.

La règle est donc de présenter ses exceptions de nullité dès que le président a terminé la lecture de l'acte de saisine.

Une exception préjudicielle ne peut donc pas être soulevée en appel, si il y a déjà eu des débats au fond en première instance.

C'est du reste parfaitement logique : imaginez un procès comme celui de la passerelle du queen mary, ou l'incendie du tunnel du mont blanc. On a des débats qui vont durer des jours voire des semaines.

Si la défense a une nullité a soulevé, il est logique qu'elle le fasse avant que ne débute les débats au fond. Car si la nullité est accueillie par le tribunal, tout s'écroule, et tous les débats au fond deviennent inutiles. Donc pour éviter de perdre du temps à étudier le fond du dossier si jamais la forme ne tient pas, il est normal d'examiner en premier les nullités.

199. Le lundi 23 mars 2009 à 22:42 par La Biscotte

ouhla ! il se fait tard...

"Si la défense a une nullité à soulevER, il est logique qu'elle le fasse avant que ne débutENT les débats au fond"

200. Le lundi 23 mars 2009 à 22:59 par marie

Jean-Louis Borloo déclare : "Non je ne suis pas alcoolique il s'agit de calomnie" :

http://frenchcarcan.com/2009/02/13/intoxicated-borloo/

201. Le lundi 23 mars 2009 à 23:14 par PrometheeFeu

@Greg: Vous avez raison. Je vais peut etres un peu vite en besogne, mais c'est bien vous qui disiez que vous avez fait un faux. Moi je vois ai fait confiance... (Allez voir les 2 videos qui circulent sur le blog. "Anything you say can be used against you" Si ca va dans votre sens, "hearsay!") :P

"§ 1003.33 Translation of documents. Any foreign language document offered by a party in a proceeding shall be accompanied by an English language translation and a certification signed by the translator that must be printed legibly or typed. Such certification must include a statement that the translator is competent to translate the document, and that the translation is true and accurate to the best of the translator's abilities."

@Nemesis: Ce que dit Eolas ne depend pas du tout de la justice de la loi. On ne viole les normes que quand on pense que l'on adhere a une norme superieure. C'est un fait psychologique bien connus. Aussi, "passer a l'acte" est un terme que l'on peu en effet appliquer aux "Justes" qui planquaient des enfants Juifs. Vous avez bien merite votre point Godwin.

202. Le lundi 23 mars 2009 à 23:32 par Clems

@biscotte

Il est interdit de lui demander si il a des nullités à soulever avant que ne commence l'examen des faits ?

Et si il vous demande de bien vouloir avant l'examen des faits d'examiner si la procédure utilisée est bien conforme ?

203. Le lundi 23 mars 2009 à 23:52 par david

réponse à Un qui drague comme un mérou #177

quel rapport ?

(bon ce monsieur devrait lire ce site car sortir encore des énormités comme " On m’a vendu les outils pour copier, stocker, visionner, écouter le tout sans limite ou presque. S’il est avéré que ce que j’ai fait est illégal, on m’a vendu tous les outils pour le faire et on m’a délibérément incité à le faire" ou "par souci d’économie mais aussi de bons sens, je vais donc télécharger sans relâche sur les réseaux peer to peer tel emule, kazaa, winmx ou direct connect." ou "De fait, l'Etat a taxé les supports informatiques sous prétexte de rémunérer les Majors, l'Etat a de ce fait légalisé le téléchargement en ligne"

bref, contrefacon, aide à la contrefacon, travail "au noir" sans déclaration en touchant les assedics, et il trouve "1an de sursis pour ça, cela parait abusif. (bon c'est clair qu'il est mal parti avec ce qu'il doit aux ayants droits

204. Le mardi 24 mars 2009 à 00:09 par bencaalors

@la biscotte :

vous ne pouvez donc pas reprocher a ce prevenu d'avoir compris que "defense au fond" voulait dire "defense au fond" et "non interrogatoire du prevenu et des temoins et requisition du procureur de la Republique".

En tout cette regle ne me semble pas vraiment etabli puisque la jurisprudence considere que si la personne n'est PAS comparante en 1ere instance et sans avocat elle peut deposer ses conclusions en appel quant bien meme il y a eu "interrogatoire des temoins" et "requisition du procureur de la Republique en 1ere instance".

''Article 459 du code de procedure penale: Le tribunal qui est tenu de répondre aux conclusions ainsi régulièrement déposées doit joindre au fond les incidents et exceptions dont il est saisi, et y statuer par un seul et même jugement en se prononçant en premier lieu sur l'exception et ensuite sur le fond. Il ne peut en être autrement qu'au cas d'impossibilité absolue, ou encore lorsqu'une décision immédiate sur l'incident ou sur l'exception est commandée par une disposition qui touche à l'ordre public.''

"Si la défense a une nullité a soulevé, il est logique qu'elle le fasse avant que ne débute les débats au fond. Car si la nullité est accueillie par le tribunal, tout s'écroule, et tous les débats au fond deviennent inutiles. Donc pour éviter de perdre du temps à étudier le fond du dossier si jamais la forme ne tient pas, il est normal d'examiner en premier les nullités"

De toute facon dans ces affaires il y a eu une purge lors de l'instruction..et dans les petites affaires la nullite sera jointe au fond selon l'article 459...

Si vous voulez que les deputes et senateur votent une loi pour qu'une audience speciale soit consacree aux nullites avant l'audience au fond, vous avez tout mon soutient mais il est plus que douteux que d'affirmer que defense = interrogatoire du prevenu.

205. Le mardi 24 mars 2009 à 03:11 par Népomucene

@Eolas, 185.

Et un point Goodwin, félicitations.

---

Sorti de ce clin d'oeil taquin, je vous remercie pour ce site en général, et chacun de vos posts en particulier. J'ai fréquenté la justice d'un peu trop près, souvent demandeur, parfois assigné ; j'y ai perdu ma candeur, mais ce site qui reste très près du sujet dessine par mosaïque pointilliste un schéma qui restaure un peu de ma confiance en les acteur de la justice.

206. Le mardi 24 mars 2009 à 07:37 par adrien

@PrometheeFeu en 201 Tout à fait, on voit donc que : premièrement ils justifiaient leur geste ("ce que je fais est juste, envoyer des enfants dans des camps est une monstruositée") deuxièmement ils espéraient bien ne pas être pris...

Il disait quoi Eolas déjà?

207. Le mardi 24 mars 2009 à 07:51 par papamobile

A Antigel en 197

ainsi Eolas ne serait pas libre car il serait catholique pratiquant.

je ne sais pas d'où vous tenez ça d'ailleurs?!

Est-il impossible d'écrire quoi que ce soit ou de dire quoi que ce soit concernant le pape, la religion et le spirituel sans faire l'objet de caricatures outrancières et d'a priori éculés.

Et si vous intéressiez sérieusement à la religion d'amour qu'est le catholicisme vous vous apercevriez qu'elle rend profondément libre.

208. Le mardi 24 mars 2009 à 07:54 par ortho pédique

A Biscotte en 186

vous avez voulu écrire "plaidoiries' sans doute et "rejetant".

La plume s'est égarée mais pas le jugement espérons-le.

209. Le mardi 24 mars 2009 à 08:16 par La Biscotte

@Bacaalors : Je puis vous assurer que la jurisprudence est très claire sur ce point, et toute exception de nullité soulevée après que les débats aient commencé sera systématiquement jugée irrecevable. Ainsi: La présentation des exceptions avant toute défense au fond conduit à ce que l'exception soit présentée par le prévenu avant son interrogatoire sur les faits. Cet interrogatoire, en effet, implique qu'il s'est engagé dans la défense au fond. À défaut de conclusions écrites, il doit alors, pour que l'exception soit régulièrement présentée, résulter des mentions de la décision elle-même que les juges ont été saisis oralement de l'exception (Cass. crim., 29 mars 1995 : Bull. crim., n° 137 ; Dr. pén. 1995, comm. n° 225, 1er arrêt, obs. J.-H. Robert. – 23 mai 1995 : Dr. pén. 1995, comm. n° 226, 2e arrêt).

A fortiori, une exception qui ne serait soulevée que devant la cour d'appel par un prévenu qui se serait défendu devant le tribunal, voire qui ne serait soulevée que devant la Cour de cassation serait irrecevable (Cass. crim., 18 janv. 1993 : Bull. crim., n° 22. – 17 févr. 1993 : Bull. crim., n° 79).

Pour ce qui est de l'exemple de la cour d'appel dont vous parlez, c'est différent puisque dans votre cas, la personne n'a pas comparu en première instance : évidemment que dans un tel cas elle peut présenter une nullité pour la première fois en appel, n'ayant jamais été encore entendue sur le fond.

Pour ce qui est de la jonction au fond de l'exception soulevée, il s'agit d'une mesure d'administration judiciaire, et il arrive souvent que le tribunal statue immédiatement sur l'exception soulevée.

Les gros dossiers ayant fait l'objet d'une information judiciaire bénéficient effectivement de la purge des nullités via l'ORTC, mais cela n'empêche nullement les avocats de la défense de présenter dès l'ouverture des débats des exceptions de nullité, fondées par exemple sur un vice affectant l'ORTC.

@Clems : les exceptions de nullité ne peuvent être soulevées d'office par le tribunal, quand bien même elles seraient d'ordre public ou substantielles (exception faite des problèmes de compétence).

210. Le mardi 24 mars 2009 à 08:18 par didier specq

Je ne comprends pas cette discussion sur les nullités. Elles doivent être soulevées "in limine litis", avant le début du procès, un point c'est tout, non?

211. Le mardi 24 mars 2009 à 10:03 par bencaalors

@La Biscotte

Merci pour ces précisions de jurisprudence..mais on peut conclure qu'il n'a pas eu un procès équitable en violation de l'article 6-1 de la Convention de sauvegarde des Droits de l'Homme et des Libertés Fondamentales. En effet, la Loi doit être prévisible et explicite..ce qui n'est pas le cas ici a mon avis.

212. Le mardi 24 mars 2009 à 10:39 par youp

Rien à voir : Que risque en france un lanceur de chaussure contre le président ? et j'ai souligné votre expression favorite "le vide juridique quasi-total" ici : http://www.rue89.com/2009/03/23/gestion-de-leau-a-qui-appartiennent-les-nuages Certaines lois sont aériennes, mais en existe-t-il sur la propriété des nuages ?

213. Le mardi 24 mars 2009 à 10:43 par Dante Timélos

La Biscotte, commentaire 186, écrit ceci:

"Sur le fait de lire des bouquins de droit pour se préparer à une audience : Ca me rappelle ce type un jour en correctionnelle, cité pour un délit routier quelconque, et qui se défendait sans avocat. Pour autant, il avait apparemment révisé son code de procédure pénale illustré (...) Bon, je l'ai laissé finir sa plaidoierie, et j'ai statué sur le dossier en le déclarant coupable (...)"

Cette anecdote (qui me rappelle étrangement une scène filmée par Depardon) est très révélatrice. Les gens de robe, juges comme avocats, sont souvent imbus de leur spécialité (le droit) et ils n'entendent pas que des néophytes viennent s'en mêler.

C'est pourquoi, dans notre guide, nous ne prétendons pas du tout qu'il faut se passer d'avocat. Même nul, un avocat sert à vous rendre crédible devant un juge.

Nous voulons seulement que les prévenus et les collectifs de défense qui se mettent en place pour les appuyer (c'est souvent le cas dans les délits "politiques" - je n'aime pas ce terme - qui touchent par exemple des manifestants interpellés) aient les moyens de comprendre la machine judiciaire: et ainsi de parler à égal avec l'avocat pour définir les choix de défense, au lieu de se les laisser imposer. Quand on a certaines convictions et qu'il y a des choses que l'on est pas prêt de dire devant un tribunal, c'est important.

Zut, je m'étais promis de ne plus intervenir sur ce blog! Je vais réciter deux Avés Maria et trois Paters pour ma pénitence...

214. Le mardi 24 mars 2009 à 10:52 par Un qui drague comme un mérou

@david > réponse à Un qui drague comme un mérou #177 > quel rapport ?

L'histoire de ce M. Budin qui a pris 130.000€ + 1 an avec sursis m'avait intriguée. J'ai lu quelques articles et surtout ses justifications candides. Il y a le sentiment d'impunité et la justification "morale". Son autobiographie vaut le détour. Je me demandais comment en arriver à ce stade en toute "innocence". Une prise de risque énorme pour un gain vraisemblablement mninable. Un mauvais délinquant donc. J'en étais là de mes réflexions puis je suis passé par ici comme je le fais régulièrement. J'ai trouvé que ce billet poursuivait heureusement ma lecture précédente.

215. Le mardi 24 mars 2009 à 11:15 par akhela

@ Dante Timelos : à priori vous avez des difficultés avec la lecture, La Biscotte ne l'a pas condamné parce qu'il s'était défendu seul, mais parce qu'il s'était mal défendu. La personne a soulevé des irrégularité de procédures après les réquisitions du parquet, càd TROP TARD. La Biscotte précise d'ailleurs bien dans son commentaire qu'on peut se défendre seul mais encore connaître TOUT le droit et pas seulement des passages.

216. Le mardi 24 mars 2009 à 11:25 par Antigel

à Papamobile (n° 206):

D'où je tiens ça?: des articles concernant la levée d'excommunication, le préservatif papal, l'avortement brésilien ou du forum associé. Peut-être me suis-je trompé, mais tant qu'il n'y a pas de démenti, l'interprétation est plausible. Il n'y a pas de mal à être catholique pratiquant, je l'ai été pendant longtemps et même militant à certaines occasions.

Actuellement, je pense que toute croyance limite la liberté. L'article proposé et les discussions associées peuvent aider à progresser vers des conceptions différentes.

Ici, il serait hors sujet de détailler. Posez vos questions sur le fil: "Le Prophétisme, illusion pathologique?" à une adresse que j'espère écrire correctement cette fois-ci pour que le lien soit direct. Voir:

[|http://www.libertyvox.com/phpBB/viewtopic.php?t=2437&start=0|fr]

217. Le mardi 24 mars 2009 à 11:28 par Pragmacrit

La justification morale me parait présentée dans cet article sous un jour négatif, comme de mauvaises excuses. Mauvaises au regard de quoi ? Du passage à l'acte qui serait "permis" par le sentiment d'impunité.
Parmi les différents exemples, il y en a un que je trouve très inadéquat : celui relatif au téléchargement. En effet, autant le vol du sac à main prive sa propriétaire légitime, autant un téléchargement illégal ne prive son auteur/diffuseur etc. que s'il remplace un achat. C'est loin d'être systématiquement le cas. En faire une généralité et légiférer en ce sens me semble faire peu de cas des réalités.

218. Le mardi 24 mars 2009 à 11:39 par D R Hache

Bonjour

fort interessante chronique, mais que dit elle de plus que la peur du gendarme ? N'oublie t elle pas de plus de gout voire la nessecité pour certains de la transgression et de l'exitation qui l'accompagne? si l'on en est à déconstruire les mecanismes psychologiques qui poussent à toute forme de délinquance, je pense qu'on peut plancher un peu sur ces theme contradictoire :

c'est interdit donc exitant. tout le monde le fait donc moi aussi.

ainsi le délinquant s'exterieorise de la masse de moutons qu'il méprise en violant leur régles, il croit enrichir son identité en se positionnant au dessus des règles qui régissent et donc définissent le corps social, mais dans le meme temps il a besoin de redefinir son appartenance à ce meme corps social par une justification du type "ca ne fait de mal à personne et de toute facon tout le monde le fait.

voire la chronique " rions un peu " http://ledrh.typepad.fr/mon_weblog/2008/05/rions-un-peu.html

bien cordialement

D R Hache.

219. Le mardi 24 mars 2009 à 11:49 par Dante Timélos

à 214:

non, je n'ai pas besoin de lunettes. J'ai bien évidemment vu le point de droit que La Biscotte avait utilisé pour écarter les conclusions du prévenu, et qui d'ailleurs a ensuite été longuement discuté par un autre internaute.

Mais je ne parle pas de ça, justement, mais du ton utilisé.

"Ah, mon petit monsieur (semble dire La Biscotte), vous voulez vous mêler de droit: mais il faut alors TOUT savoir sur la procédure, autrement dit (semble toujours dire La Biscotte) il faut faire au moins cinq ou six années d'études comme moi avant de pouvoir donner son avis".

Je ne sais pas si j'ai du mal avec la lecture, mais vous, 214, vous êtes sourd(e) aux sous-entendus. Pourtant, ce que dit La Biscotte a le mérite d'être clair...

220. Le mardi 24 mars 2009 à 11:49 par proc hure

A Dante timeleos en 212

vous écrivez: "même nul un avocat sert à vous rendre crédible devant le juge"

Et vous prétendez éclairer vos lecteurs sur le fonctionnement de la justice, si j'ai bien compris.

De manière générale la crédibilité n'a pas sa place dans le débat judiciaire et l'assistance d'un avocat ne change rien à l'affaire.

Ce qui compte ce sont les preuves (r)apportées, in concreto dans le dossier.

Vous pouvez apparaître totalement crédible, avec votre avocat, mais s'il n'y a rien au dossier, alors bernique.

Le juge d'instance va vous raser et donner gain de cause à l'adversaire.

C'est comme ça.

Allez de temps en temps assister aux audiences du tribunal d'instance.

Ou aux audiences correctionnelles courantes, dans tous les tribunaux de France (et de Navarra).

Les "crédibles", qu'il ne faut pas confondre avec les "de bonne foi", il s'en condamne dix toutes les heures.

On n'est pas chez les intellos mais chez les Saint Thomas, on fait dans le concret.

Désolé de le rappeler.

221. Le mardi 24 mars 2009 à 12:07 par Bruno Kant

@212. par Dante Timélos "Même nul, un avocat sert à vous rendre crédible devant un juge. ... et ainsi de parler à égal avec l'avocat pour définir les choix de défense, au lieu de se les laisser imposer"

Certains pourraient en avoir des boutons, mais tant pis. Je dirais que parfois, même s'il est "bon", il vaut mieux prendre l'avocat par la bride voir même en changer s'il ne convient pas. Ce n'est pas pire ni bien différent de randonnées dans la nature sur des montures un peu capricieuses. A ce jeu, j'ai pu constater que ce sont les juges qui finissent désarconnés.

"dans les délits "politiques" - je n'aime pas ce terme"

"un avocat sert à vous rendre crédible devant un juge."

Ca dépend du juge. S'il a décidé de ne pas vous entendre, il se contentera de vous écouter un peu, vous et votre ou vos avocats. Vous écouter lui sera toujours utile à mieux motiver ses décisions. Révisez le Code noir et la harangue baudot.

222. Le mardi 24 mars 2009 à 12:13 par Bruno Kant

@219, par proc hure "Ce qui compte ce sont les preuves (r)apportées, in concreto dans le dossier."

Ca dépend des prétoires.

223. Le mardi 24 mars 2009 à 12:17 par La Biscotte

@Dante timeleos : merci du joli procès d'intention que voilà..

Je n'ai jamais méprisé un prévenu à l'audience, et ceux qui se présentent sans avocat ont au contraire toute mon attention.

En poste dans un petit tribunal de province, je déploie régulièrement des trésors de pédagogie pour expliquer de la manière la plus claire possible la procédure à l'audience et la décision aux prévenus qui n'ont pas d'avocats.

Quant à l'anecdote dont j'ai fait état, je me souviens parfaitement de l'affaire et je n'ai absolument pas été méprisante : je n'ai pas rendu ma décision en disant " vous ne connaissez rien au droit, vous vous êtes plantés misérable insecte ayant voulu voler trop près du soleil"..

J'ai simplement rejeté l'exception en indiquant qu'elle était irrecevable car tardive, en expliquant au prévenu pourquoi.

Déduire de mon post que moi, comme tous les juges et avocats, sommes "imbus" de notre spécialité et n'admettons pas qu'on vienne s'y mêler, bonjour l'extrapolation !

Néophyte ou professionnel, il faut respecter les règles de procédure pénale, je me fiche pas mal de savoir si le prévenu a un doctorat en droit ou pas.

224. Le mardi 24 mars 2009 à 12:21 par Niko

Bonjour

Dans le cas du téléchargement, il y a (du point de vue du néophyte que je suis) une différence : le bien est immatériel. On ne prend pas le sac à main d'une petite vieille, on ne vend pas de substances, on n'emprunte pas une voiture. L'argument du "de toute façon je ne l'aurais pas acheté" est valable, au moins en partie ; si beaucoup de films ou d'albums auraient été acheté si le P2P n'existait pas, on télécharge beaucoup plus que ce qu'un budget culture raisonnable pourrait permettre. Ce qui ne méritait qu'une écoute en magasin il y a quelques années est téléchargé aujourd'hui sans vergogne, souvent pour n'être écouté que quelques fois, au mieux.

Les majors et consorts se basent sur les téléchargements pour établir le préjudice, mais qui pourrait affirmer que l'intégralité de ces téléchargements se seraient traduit par des entrées en salle ou des achats ? Dès lors, comment établir le préjudice, et par la même la faute ?

225. Le mardi 24 mars 2009 à 12:22 par Dante Timélos

Un procureur nous dit (commentaire 219): "ce qui compte ce sont les preuves (r)apportées, in concreto dans le dossier."

Les dossiers dans les trois quart des procès (je parle ici des procédures dites rapides, CI et autres...) ce sont:

- les déclarations des policiers (ce qu'ils disent vous avoir vu faire, ce qu'ils disent avoir trouvé sur vous)

- vos propres déclarations si vous avez été assez stupide pour parler en garde à vue, et vous être laissé piéger. Je précise que des "innocents" (ce terme n'a pas beaucoup de sens) se laissent souvent piéger

- qui vous êtes (socialement parlant)

- votre casier

- éventuellement (mais ce n'est pas le plus courant) un témoignage

J'ai assisté à des comparutions, mais pas dans le même rôle que vous, monsieur le procureur. Je sais de quoi je parle, car de là ou j'étais on voit ce que vous ne voyez pas, ou plutôt ne voulez pas voir...

226. Le mardi 24 mars 2009 à 12:31 par Dante Timélos

En réponse à 223:

Ferons-nous donc jouer la présomption d'innocence en faveur de La Biscotte? Ses propos ont sans doute été rapidement jugés, aussi vite, peut-être, que ceux d'un prévenu dans le box quand le juge consacre dix minutes à son affaire...

Mais, sur le fond, la morgue des spécialistes du droit est un fait. Le ton de nombre de commentaires sur ce blog en fait foi, tout comme l'expérience tirée des prétoires. Oui, il faut changer d'avocat le cas échéant: et surtout ne pas se laisser imposer des choix de défense.

227. Le mardi 24 mars 2009 à 13:02 par Manou

Très intéressant billet d'analyse. Faisant moi-même partie des gens qui répètent que "de toute façon je l'aurai pas acheté" je vais réflechir à la question. Est-ce juste une justification morale? Est-ce vrai. Je me propose de faire le tour de mon disque de dur et de voir si il n'y a pas des choses que j'aurai du acheter.

PS : j'aime beaucoup le petit dessin du chasseur de troll!

228. Le mardi 24 mars 2009 à 13:21 par Bruno Kant

@222. par La Biscotte "Je n'ai jamais méprisé un prévenu à l'audience, et ceux qui se présentent sans avocat ont au contraire toute mon attention."

Tout dépend de ce que vous entendez par "votre attention". Vous exercez un métier et une fonction très particulière qui peut faire ou défaire n'importe quoi. D'autre part, quelle que soit votre attention d'un moment, elle peut être perçue de manière assez nuancée. Dominique Simonnot qui a fréquenté beaucoup de prétoires a dépeint une sorte de loterie nationale...

Tout début 2007, je me suis retrouvé au tribunal d'instance parce qu'une commission avait refusé mon inscription sur la liste électorale (...) et je me suis défendu seul. Tout en "discutant" avec le juge et au moment de lui tendre mes justificatifs, je me suis apperçu qu'en partant de chez moi, je m'étais emparé non pas d'une fausse... mais de la mauvaise facture EDF!

Je suis rentré chez moi et en attendant son délibéré, je me suis documenté un peu plus sur ce qu'était la mort civile. Ce n'était pas insensé comme démarche:

Le tribunal reconnaît qu’il est... vivant / 10.01.2009

15 jours plus tard, j'apprennais que j'allais pouvoir voter.

229. Le mardi 24 mars 2009 à 13:46 par lapocompris

Un petit hors sujet, j'ai un énigme à résoudre :
mademoselle a=k o/o 2xi
J'ai trouvé la profession (Avocat au Barreau de Paris) et peut-être le prénom (Sophie ?) mais il me manque le nom.

230. Le mardi 24 mars 2009 à 14:03 par athenais

x

231. Le mardi 24 mars 2009 à 14:15 par Kiol

Pourquoi Sophie ?

Je pense qu'il s'agit de Anne DEMETZ

mademoselle faute de frappe pour mademoiselle ou bien "made moselle" d'ou de metz

232. Le mardi 24 mars 2009 à 14:28 par La Biscotte

@Bruno Kant :J'entends par là que je suis parfaitement consciente que mon métier et tout le cérémonial de l'audience (robe, estrade pour "dominer" l'assemblée..) impressionne forcément le quidam venu se faire juger, et qu'il est donc d'une simplicité déroutante de faire preuve de mépris, de couper la parole dès que le prévenu essaie d'en placer une, de se moquer de sa défense à 2 sous, de faire preuve d'autoritarisme etc...

On peut faire application des règles de procédure tout en gardant toujours à l'esprit qui l'on a en face de nous.

J'ai hélas pu rencontrer certains magistrats assez odieux à l'audience, ouvertement méprisant envers le prévenu, et faisant ressentir dès la première question posée que son jugement était déjà fait..

C'était en majorité des magistrats assez âgés, sans doute usés par la routine des audiences et n'acceptant aucune remise en question.

Il y a des cons partout, la magistrature ne fait pas exception.

Mais du coup, du haut de ma jeunesse, je grince des dents quand j'entends régulièrement les politiques brocarder les "jeunes juges" (cf.Outreau) et l'incompétence qui doit forcément en résulter selon eux.

Je crois que Me Eolas avait pu écrire dans un autre billet que les pouvoirs du juge unique en correctionnelle lui faisait bien + peur que ceux du juge d'instruction qui focalise pourtant l'attention de tous.

Il a entièrement raison. La très grande majorité des délits passent aujourd'hui en juge unique (sauf ceux passant par la 3è voie), le juge se voyant accorder de très gros pouvoirs grâce à cause des lois sur la récidive.

Ainsi que le disait le philosophe arachnéen, "A grands pouvoirs grandes responsabilités". Je rajouterais "collégialité" mais ce n'est définitivement pas dans l'air du temps..

233. Le mardi 24 mars 2009 à 14:46 par La Biscotte

Dante Timélos a écrit :Ses propos ont sans doute été rapidement jugés, aussi vite, peut-être, que ceux d'un prévenu dans le box quand le juge consacre dix minutes à son affaire...

Voilà un coup bas ! Croyez bien que, d'une manière générale, les magistrats aimeraient bien pouvoir passer + de temps sur les dossiers, mais avec une moyenne de 30 dossiers par audience, c'est impossible sauf à juger en nocturne.

Cela dit, je n'ai jamais empêché un prévenu de parler lors de sa défense, quitte à faire des renvois d'office pour les dossiers suivants (ça apprendra au Parquet à mieux audiencer..)

234. Le mardi 24 mars 2009 à 15:21 par france

Et bien !! instructif tout ça.

235. Le mardi 24 mars 2009 à 15:30 par Wyrm

"(c'est là le but de la loi HADŒPI : faire cesser le sentiment d'impunité des téléchargeurs pirates)"

Est-ce que le fait que la loi en préparation ne sanctionne pas particulièrement le piratage mais la non sécurisation de connexion internet d'adresse IP ne tombe pas à côté de la plaque?

En ordre de grandeur (je précise ça pour éviter les foudres de l'ACAF), cela revient à imposer à tout le monde de savoir désamorcer une bombe pour faire cesser le "sentiment d'impunité" des terroristes. Je développe...
(NB: si l'ACAF me sanctionne pour les bombes, j'avouerai que le déminage est peut-être plus simple que la sécurité informatique... bien que certainement plus nuisible pour la santé. Sinon, j'ajouterai pour ma défense qu'il y a à peu près autant de lien entre "poser une bomber et la désamorcer" qu'entre "télécharger illégalement et sécuriser sa ligne".)

Sécuriser une connexion réseau, et plus encore son adresse IP, sont des défis pour des professionnels... Peut-on donc légitimement donner une obligation de résultat de cet ordre à un particulier? Et étant donné que nous sommes encore dans le flou quant aux "outils de sécurisation" promis, l'obligation de moyens est elle-même très discutable... car il y de nombreux doutes tant sur leur efficacité que sur leur "honnêteté".
(Je précise que je suis, sincèrement et sans ironie, curieux de savoir ça: peut-on, de manière générale, légalement contraindre un particulier à un résultat difficile à atteindre pour un professionnel?)

236. Le mardi 24 mars 2009 à 15:45 par Wyrm

Je précise que je ne me pose pas tout à fait la même question que Personne@82: même si la non sécurisation n'est pas un délit, on veut nous le faire passer comme une obligation légale... en sanctionnant son non-respect par une peine administrative dans le seul but de contourner des décisions préalables "d'entités supérieures" (dont le conseil constitutionnel).
Est-ce que ça devient acceptable pour autant?

A noter que vous avez, il y a peu de temps, très bien expliqué ce projet de loi. Mais votre petite remarque sur son objectif, bien qu'exact (les rapporteurs ont effectivement dans l'idée de rendre le "piratage" risqué dans l'esprit des gens), ramène le débat à la surface...
- est-ce qu'on ne se trompe pas d'objectif en punissant le propriétaire de la connexion sans chercher à savoir s'il est le "pirate"?
- est-ce qu'on peut contraindre Mme Michu à sécuriser son adresse IP comme une pro?
- est-ce que les fondements du droit sont respectés? (notament eu égard au principe du contradictoire et de l'équitabilité)

Je suis conscient qu'une série de commentaires sur un post de blog n'est pas le lieu idéal pour un tel débat... mais j'ai quand même cette petite question qui me trotte dans la tête... "Mme Michu doit-elle devenir experte en sécurité des réseaux?"

237. Le mardi 24 mars 2009 à 16:07 par Guillaume

@ Wyrm 236

Ne vous inquiétez pas pour Mme Michu, on lui vendra une solution toute faite de sécurisation labélisée qui la mettra à l'abris de sanction éventuelles. Comme Mme Michu ne connait rien en sécurité des réseaux et qu'on lui fera une belle opération de marketing elle achètera les yeux fermés et on lui fichera une paix royale. (Notez que l'efficacité même de la solution de sécurisation n'est pas importante tant qu'elle sera labélisée)

238. Le mardi 24 mars 2009 à 16:25 par Lazarre

Cher Maître,

Merci encore une fois pour cette analyse, qui comme beaucoup d'autres méritait une place plus visible encore dans le débat public, pour l'éclairage qu'elle donne sur des mécanismes qui échappent souvent au citoyen non-averti (qui pense effectivement que le moindre délinquant est un méchant digne d'affronter James Bond et Navarro en même temps).

Je me permettrai humblement de rajouter un pendant d'analyse stratégique à votre analyse juridique (après tout, le fonctionnement d'un politique est d'utiliser la loi pour promouvoir un certain modèle de société). Contrairement à ce qui inquiète certains, l'HADOEPI n'a pas pour objectif de décourager à coups d'emails les grands penseurs de l'échange de fichiers libres. Cette loi vise les français qui téléchargent des contenus protégés de façon occasionnelle, simplement "parce que c'est facile", et qui en dehors de ce fait n'ont aucunement un profil de pirate informatique ou de voleur d'une sorte ou d'une autre.

Une étude a révélé que 25 à 35% des Français pratiquent ce genre de téléchargements répréhensibles. Face à cette population, l'arsenal légal actuel est inadapté, ce car il conduirait à classer comme criminels une frange de la population si grande qu'il serait inepte (et même dangereux) de vouloir appliquer sur eux la loi dans sa forme actuelle (mettre 25% des Français en prison ? Quand en plus ce sont essentiellement des personnes de moins de 35 ans ? Autant pousser l'âge de la retraite à 75 ans tout de suite, parce qu'il va y avoir un sacré creux démographique).

Avec l'HADOEPI, l'Etat va disposer d'un outil permettant de faire sortir de ce panel un grand nombre de téléchargeurs, les plus occasionnels, qui se décourageront rapidement après avoir reçu le mail d'avertissement, et le courrier recommandé. Ainsi, les forces de l'ordre pourront se concentrer avec toute la sévérité légale sur les "pirates" les plus durs : ceux qui agissent par idéologie, et ceux qui participent de la mise à disposition de contenus protégés.

Il faudra certainement quelques mois à un an pour évaluer si cette nouvelle loi réussit sur ce point. Personnellement je reste assez dubitatif, non pas sur le caractère dissuasif des courriers personnalisés (qui rempliront leur fonction sur le public ciblé), mais sur la réelle capacité des autorités à mettre cette loi en application, et à identifier les usagers se livrant au téléchargement. Et quand on considère les moyens mis en oeuvre pour ce résultat, et la limitation apportée à l'utilisation d'internet qui peut en résulter, j'ai l'impression que ce dispositif est avant tout une gigantesque usine à gaz.

239. Le mardi 24 mars 2009 à 16:35 par indi

J'espère que vous avez lu le magnifique article du Figaro sur les commissions d'office et l'Aide Juridictionnelle.

Je m'abstiens de tout commentaire pour le moment

240. Le mardi 24 mars 2009 à 16:49 par lapocompris

A propos de ce magnifique article du Figaro, je cite "L'avocat commis d'office, d'origine asiatique, ne parle qu'à peine le français… ".
Soit l'article du Figaro est teinté d'un préjugé condescendant soit les examens pour devenir avocat sont des formalités.

241. Le mardi 24 mars 2009 à 17:32 par Bruno Kant

@232, par La Biscotte

A mon avis, il n'y a pas d'âge pour être ponctuellement indisponible ou durablement abject, les exemples ne manquent pas. Je n’en citerais qu'un, parce que comique, celui de ce magistrat assommé par la police à coup de flash ball alors qu'il était ivre et qu'il les menaçait avec une épée de franc-maçon (probablement un coupe papier). Il me semble que devant le CSM, il a plaidé qu'il vivait un divorce difficile... Je ne pense pas qu'on puisse en tirer de quelconques généralités susceptibles de jeter l'opprobre sur l'ensemble des magistrats ou le personnel de la justice.

les politiques brocarder les "jeunes juges" (cf.Outreau) et l'incompétence qui ... les pouvoirs du juge unique en correctionnelle lui faisait bien + peur que ceux du juge d'instruction qui focalise pourtant l'attention de tous.

La collégialité, c’est bien beau, mais une telle décision serait encore plus forte que celle d'un juge unique. Nicolas en personne nous a fait une belle démonstration avec la poupée vaudou: la cour d'appel a nuancé le jugement rendu en première instance, par un collège! S’agissant de collégialité et d’Outreau, la mayonnaise a tout de même pris autour d’une soixantaine de magistrats. A mon avis, en désignant l’un des juges et l’un des procureurs, les politiques ont surtout fait le nécessaire pour qu’on oublie d’autres pans et difficultés mises en lumière par cette même affaire (la qualité des investigations, la médecine légale, la contribution des psychologues ou même de certains tiers, …).

de très gros pouvoirs à cause des lois sur la récidive

Il y a différentes manières de soulager les plus anxieux? On s’incline et on se résigne face à la volonté et aux choix des politiques et de ceux qui les soutiennent. Fillippis a toussé trois fois et on supprime le JI. Il y a aussi eu de longs débats sur le thème de la rétention de sureté, alimentés au gré des faits divers… et quelques mois plus tard, on nous confirmait que la psychiatrie en France est elle aussi en crise, depuis longtemps. Par moments, tout dans ce pays semble être en crise, à la recherche de rustines, de lampistes aussi, à punir ou à exclure, pour l’exemple ou pour la satisfaction du plus grand nombre ou de plus puissants (je pense à des lobbies et à ceux qui leurs tendent le porte voix). La récente crise financière n'arrangera rien.

Il y a des cons partout, la magistrature ne fait pas exception.

Tant que ce n’est que l’un de vos voisins et qu’il n’a pas de relations ou de pouvoirs particuliers, ce n’est pas bien grave.

242. Le mardi 24 mars 2009 à 17:43 par Le Beurre

@La Biscotte

{{"Voilà un coup bas ! Croyez bien que, d'une manière générale, les magistrats aimeraient bien pouvoir passer + de temps sur les dossiers, mais avec une moyenne de 30 dossiers par audience, c'est impossible sauf à juger en nocturne.

Cela dit, je n'ai jamais empêché un prévenu de parler lors de sa défense, quitte à faire des renvois d'office pour les dossiers suivants (ça apprendra au Parquet à mieux audiencer..)"}}

Ah nos juges garant des libertes individuelles...cheres a notre coeur de citoyen..

C'est le procureur de la Republique qui determine le nombre de dossier par audience !! hey ben bravo! C'est incroyable !! Parlons en de l'egalite des armes avec le prevenu!

Chaque citoyen a un droit a un proces equitable (article 6 de la Convention) et on ne peut considerer qu'un proces penal durant 10 mn sans que les agents de police et les temoins ne sont cites a l'audience soit equitable, sans que le prevenu sans avocat ait acces au dossier etc.

Bien entendu le procureur de la Republique veut que le moins de temps possible soit consacre aux "cross examination" des temoins..ils preferent que l'on lise des proces verbaux de police plutot qu'on interroge les policiers/ temoins directement...ben voyons!

La responsabilite du proces equitable c'est au juge correctionnel president de l'audience qu'elle incombe.

De meme des juges condamnent des personnes a des mandats de depot tout en sachant que la maison d'arret locale est superpeuplee et que une detention dans cette maison d'arret est un traitement degradant (violation de l'article 3 de la Convention).

On ne peut pas fuir sa reponsabilite en envoyant un citoyen dans un lieu ou il seras soumis a des traitements degradants ..

meme si l'impunite penale et la justification (c'est la faute du proc', il y a pas d'argent etc) permet de ne pas se sentir..coupable.

Pensez vous que cet homme qui a manifestement eu ses droits fondamentaux violes en detention policiere de garde a vue a eu un proces equitable quand il n'a pu se defendre sur ces nullites ? Je ne le pense toujours pas...

243. Le mardi 24 mars 2009 à 18:13 par La Biscotte

@Le Beurre : euh.. comment dire ?

Si le prévenu (ou son avocat) veuille faire citer des témoins, ils en ont tout à fait le droit. En pratique, ils n'en usent presque jamais.

10 mn passées sur un dossier, c'est valable surtout pour les dossiers très simples qui ne méritent pas vraiment qu'on s'y attarde + (ex : la conduite en état alcoolique, le prévenu reconnaît les faits).

Et encore, entre l'interrogatoire de base du prévenu (identité, situation sociale/professionnelle, rappel des faits, pourquoi vous buvez etc.., réquisitions du MP et dernières déclarations du prévenu ss avocat), on arrive + au 1/4 d'heure.. Honnêtement, pour ce genre de dossier c'est tt à fait suffisant.

Il ne faut pas faire de généralité, une affaire un peu + complexe ou un dossier où le prévenu nie les faits prendront forcément + de temps que 10 mn.

Faudrait voir à arrêter un peu de citer la CEDH à tout bout de champ, sans connaître la pratique réelle des audiences correctionnelles..

Dans 80 à 90% des dossiers, les faits sont reconnus par les prévenus, on n'a donc pas vraiment besoin de déranger les policiers et autres témoins pour refaire l'histoire. Et encore une fois, libre au prévenu de faire citer qui il veut !

Quant à envoyer en prison des gens dans des prisons insalubres, je suis d'accord avec vous que nos prisons sont une véritable honte. Mais il ne faut pas en rendre responsable les magistrats ! Chacun son travail, à ce titre là, on pourrait aussi demander aux médecins d'arrêter de soigner les gens dans des hôpitaux surpeuplés et vétustes ?

Il ne s'agit pas de "fuir sa responsabilité" mais de mettre chacun devant SES responsabilités.

Quant à l'homme dont vous faisiez allusion, j'imagine qu'il s'agissait de mon prévenu qui avait plaidé tardivement la nullité de la procédure ? Dans ce cas, pour info, il reprochait aux policiers de ne pas lui avoir notifié immédiatement ses droits en garde-à-vue mais seulement plusieurs après, une fois dégrisé... (ce qui est parfaitement légal, à quoi bon notifier des droits à quelqu'un qui, avec + de 3 grammes dans le sang, n'est pas vraiment en état de les comprendre ?)

244. Le mardi 24 mars 2009 à 18:56 par De minimis

@ La Biscotte en 243 C'est marrant que vous souleviez ce point, ça tombe pile sur ce sur quoi j'ai planché ce matin.

Sans aucun lien, il est dommage que les personnes dont le droit n'est pas la profession considèrent les juristes comme de fuligineux alchimistes. Des sorciers, en somme, qui se réunissent en d'imposants sanctuaires pour déblatérer en langue hermétique et sceller le sort du pauvre justiciable. C'est marrant, j'avais jamais vu la Justice rendue au nom du Peuple français comme un Sabbat de sorcières. D'un coup mes révisions deviennent plus... "Poudlardesques"!

245. Le mardi 24 mars 2009 à 21:23 par Bruno

@240, par lapocompris "soit les examens pour devenir avocat sont des formalités."

On s'inscrit où? La page précédente est pas mal aussi:

Affaire Fofana : elle - une avocate - est partisane de la publicité des débats, au motif que le «huis clos a aussi pour but de protéger la police, qui ne répondrait pas publiquement d'une enquête ratée».

Je crois que souvenir qu'une équipe recherchait des activistes en Corse...

246. Le mardi 24 mars 2009 à 23:27 par L'argent du beurre

@244 : La Biscotte

"Faudrait voir à arrêter un peu de citer la CEDH à tout bout de champ, sans connaître la pratique réelle des audiences correctionnelles.."

Quelle bonne idee! On peut demander que la France fasse un avenant stipulant que la Convention de sauvegarde des Droits de l'Homme et des Libertes Fondamentales et la jurisprudence de la Cour europeene des droits de l'homme ne s'appliquent plus aux juridictions correctionelles...(ben oui par ce que l'on est un superbooke vous voyez le probleme).

Deja les articles 5-3 et 5-4 de la Convention n'existent pas dans le code de procedure penal francais..c'est plus pratique non ;) ? Le 5-1-c depuis 1982 n'est pas applique dans les enquetes preliminaires et de flagrance.

Et les articles 6-1, 6-2, 6-3 a) b) d) ne sont pas appliques (on n'a pas le temps vous savez et puis on va pas "deranger" les agents de police)..

Ou on peut laisser au procureur le soin de faire des compositions penales, mediations penales et CRPC pour chaque personne qui le souhaite ait droit a un proces equitable.

Pour les prisons : "Mais il ne faut pas en rendre responsable les magistrats !" Ben oui ce n'est pas comme ce sont eux qui les ont envoye la bas, suis je bete...

Dans les etats de droits, les prisons surpeuples sont vidés (avec des remises de peine) pour que les autres detenus puissent entrer car on ne peut concevoir que des citoyens soient soumis a des traitements degradants de la nature de ceux subis en France.

Mais finalement les traitements degradants cela fonctionne bien.."dans 80 à 90% des dossiers, les faits sont reconnus par les prévenus".. c'est mieux que l'inquisition dis donc!! Je tire mon chapeau au repentir des mis en cause qui ont confondu commissariat de police et confessional.

Tout cela fait vraiment froid dans le dos...

"Chacun son travail, à ce titre là, on pourrait aussi demander aux médecins d'arrêter de soigner les gens dans des hôpitaux surpeuplés et vétustes ?" heu mmh

Si chaque patient sort avec une maladie nocosomiale de l'hopital de la Sante de Framboisy, il est du devoir du medecin de ne plus pratiquer dans cet hopital et d'alerter la DASS et la presse.

Pour votre prevenu comme les milliers d'autres qui ont droit a des gardes a vue EXTRA longue, avec des aveux EXTRA pratiques et des proces EXTRA court...je me demande comment ils percoivent la legitimite de la peine...mais je peux vous dire que les notions de proces equitable ou de respect de la diginite humaine, elles leur ont pas echappe in extenso and in concreto ;)

Peut etre faudrait t'il que les juges prettent serment de respecter la Convention de sauvegarde des Droits de l'Homme et des Libertes Fondamentales avant d'entrer en fonction ?

247. Le mercredi 25 mars 2009 à 10:14 par Wyrm

@237, Guillaume: c'est justement pour ça que j'ai rappelé les deux types de doutes... et j'avais oublié l'absence d'interopérabilité... (Même si Mme Michu sera très probablement sur l'OS majoritaire du marché)

Serons-nous donc obligés d'installer un logiciel sur lequel on émet des doutes? En termes d'efficacité passe encore puisque ça validera la défense... mais sur l'honnêteté et la fiabilité en termes de failles de sécurité? Devra-t-on risquer d'ouvrir des failles dans notre déjà maigre sécurité informatique en installant des outils dont nous ne saurons rien? Sans compter un point simple et élémentaire... Si ce logiciel doit s'installer sur le poste de l'utilisateur... il ne le "défendra" que durant son activité... Si Mme Michu se fait pirater pendant qu'elle fait une promenade, rien ne la défendra.

Que dire aussi de cette labélisation? Sera-t-elle permanente pour un logiciel donné ou pourra-t-elle être retirée si l'Autorité décide qu'il n'est plus assez fiable? Mme Michu pourra-t-elle alors être pénalisée pour avoir continué à utiliser en toute bonne foi un logiciel "périmé"?

Bref, nous ne savons encore rien de ces logiciels de "sécurisation" dont les charges ne sont pas encore établies (contrairement au processus de la riposte... il y a des priorités dans la vie), c'est pourquoi j'attends avec impatience d'en savoir plus là-dessus... Mais évidemment, c'est très, très loin des préoccupations des législateurs... ou de leurs "muses".

248. Le mercredi 25 mars 2009 à 11:57 par proc hure

A l'argent du beurre en 246

oui, dans 90 % des dossiers il n'y a aucune contestation sur la culpabilité.

ça a l'air de vous étonner, mais peut-être pensez-vous que de coupables il n'y en a nulle part, que la société est coupable, que les gendarmes sont coupables, que les juges sont de dangereux personnages...

Et à ceux qui déplorent qu'aucun témoin ne soit jamais cité devant le tribunal je rappelle dans les traces de LA biscotte qu'il suffit que le prévenu en fasse citer pour que le juge soit contraint de les entendre.

Alors à tous les contempteurs de la justice républicaine, prenez la robe (d'avocat) et montrez-nous, sans tarder, ce que vous savez faire.

Je suis impatient que vous puissiez contribuer, comme nous tous, à l'oeuvre de justice.

249. Le mercredi 25 mars 2009 à 12:08 par La Biscotte

@Proc hure : merci, moi j'avais baissé les bras...

250. Le mercredi 25 mars 2009 à 13:23 par Bruno Kant

@Proc hure, en 248 "dans 90 % des dossiers il n'y a aucune contestation sur la culpabilité."

On peut donc en déduire qu'il y a contestation dans 10% ces cas mais que globalement, c'est efficace. D'après ce que je sais, parmi ces 90% de coupables, il y aura des contestations mais qui porteront plutôt sur les peines prononcées.

Par ailleurs et d'après les statistiques justice, avant d'attendre de chiffre de 90% aux audiences, il s'effectue un sacré tri en amont si on se fie au nombre de PV enregistrés et au nombre d'affaires effectivement traitées dans les tribunaux. Cela peut expliquer certains mécontentements, notamment de victimes qui ne seront jamais entendues (faute de preuves, de ci ou de cela).

D'autre part, je trouve regrettable qu'on se focalise à chaque fois sur ces affaires et chiffres qui relèvent plutôt de la délinquance alors que la justice intervient aussi dans d'autres domaines (famille, enfance, commerce, marchés publics, ...). Là, nous aurions probablement des informations plus nuancées que simplement "90% de coupables"?

Comme je l'écrivais plus haut, seul environ 20% des enfants placés le sont en raison de maltraitance ou de défaillances parentales graves.

251. Le mercredi 25 mars 2009 à 19:07 par ariane

@ Dante Timelos, Petruk

je persiste à dire que dans l'immense majorité des cas, il est préférable de s'exprimer en garde à vue.

Petruk, si la police vous arrête pour meurtre alors que vous vous savez innocent, dites le tout de suite aux policiers. Je crois qu'il est important de rappeller que, contrairement à ce que vous semblez pensez (comme beaucoup d'autres d'ailleurs) que les policiers ou gendarmes (ne les oublions pas!) ne sont pas obsédés par prouver votre culpabilité, quand même. Si vous leur donnez un alibi, ils vont aller VERIFIER, et non pas dire au témoin de dire si ou ça... par ailleurs, quand vous dites Petruk qu'il vaut mieux attendre que les versions des témoins se soient un peu décantées "quitte à passer un mauvais moment", je dis :

1) que plus les témoins sont entendus rapidement, plus leurs déclarations sont fiables en principe (souvenirs plus frais, absence de concertation avec la personne mise en cause ou la famille de cette personne...)

2) le mauvais moment en question peut être un séjour en détention provisoire, le temps d'aller entendre les témoins (soit dit en passant, s'il y a instruction, ces témoins ont de fortes chances d'être entendus sur commission rogatoire, donc normalement par les enquêteurs qui vous ont placé en garde à vue...); je crois que si vous pouvez vous éviter un seul jour de détention provisoire, faites le!!!!!

252. Le mercredi 25 mars 2009 à 22:11 par Jolie histoire...

Et dommage pour lui : si le menteur avait été avocat, cela ne lui aurait rien couté, tant il est connu qu'on peut dire n'importe quoi à un procès quand on l'est, sans être inquiété... foi d'avocat !

253. Le jeudi 26 mars 2009 à 12:54 par L argent du beurre

@proc hure (248);

Oui nous avons bien note que le respect de la Convention de sauvegarde des Droits de l'Homme et des Libertes Fondamentales importe peu pour les procureurs mais cela nous le savions deja car il sont charge des poursuites et non une autorite judiciaire charge du respect de la Convention;

"ça a l'air de vous étonner, mais peut-être pensez-vous que de coupables il n'y en a nulle part, que la société est coupable, que les gendarmes sont coupables, que les juges sont de dangereux personnages...":

Wow c'est la position de la France suite aux violations de la Convention dans les juridictions correctionelles et en Gu.yane (cf dernier rapport CNDS mars 2009) ? J'adorreeeee...et la petite touche d'attaque perso a "la francaise". Bravo!! Genial!!

"oui, dans 90 % des dossiers il n'y a aucune contestation sur la culpabilité."

On voit que les procureurs sont vraiment habitues a prouver la culpabilite des prevenus/accuses au dela de tout doute. Quel talent!! mais plutot dans 100% des cas ils sont tous presumes coupables et menteurs non ?

"Alors à tous les contempteurs de la justice républicaine, prenez la robe (d'avocat) et montrez-nous, sans tarder, ce que vous savez faire."

Je vous remercie pour votre proposition personnelle de "vient te battre" mais il n'est pas ethique de se lancer dans une "bataille" non equitable et ou il n y a pas d'egalite des armes. Ce serait legitimer la bataille...Desole...

@La Biscotte (249);

"@Proc hure : merci, moi j'avais baissé les bras..."

Je vous conseille cet e-book du conseil de l'europe sur le proces equitable (art 6-1 Convention) :

et celui la sur l'article 5 de la Convention :

Bonne lecture!

254. Le lundi 6 avril 2009 à 14:06 par ClaireM3T

''Quant à la "vérité", personne n'y croit et surtout pas la police et le procureur qui bâtissent l'accusation. Alors... '' Je crois que votre interlocuteur n'ait vraiment raison sur ce coup-là.

Et si vous pensez que l'impunité est ce qui évite de la retenue chez le délinquant de base, est-ce cette même impunité qui évite aux procureurs aux juges et à la police d'appuyer leurs accusations sur une réalité des faits.

Il y a beaucoup de mensonge dans la justice française. Car les acteurs de la justice ne prennent aucun risque à condamner sur des mensonges.

Mes logiciels, comme mes clients, sont libres. Ce blog est délibéré sous Firefox et promulgué par Dotclear.

Tous les billets de ce blog sont la propriété exclusive du maître de ces lieux. Toute reproduction (hormis une brève citation en précisant la source et l'auteur) sans l'autorisation expresse de leur auteur est interdite. Toutefois, dans le cas de reproduction à des fins pédagogiques (formation professionnelle ou enseignement), la reproduction de l'intégralité d'un billet est autorisée d'emblée, à condition bien sûr d'en préciser la source.

Vous avez trouvé ce blog grâce à

Blog hébergé par Clever-cloud.com, la force du Chouchen, la résistance du granit, la flexibilité du korrigan.

Domaine par Gandi.net, cherchez pas, y'a pas mieux.